You are on page 1of 168

A course material on

MATHEMATICS – II

By

Mr. S. SINNAPPARAJ M.Sc.,B.Ed.,M.Phil.,

Assistant Professor

DEPARTMENT OF MATHEMATICS

BALAJI INSTITUTE OF ENGINEERING AND TECHNOLOGY

OMR,THANDALAM,THIRUPORUR,

CHENNAI- 603110
QUALITY CERTIFICATE

This is to certify that the course material MA6251 MATHEMATICS - II


being preparedby me meets the knowledge requirement of the university
curriculum.

Signature of Author

This is to certify that the course material being prepared by Mr.S.Sinnapparaj


is of adequate quality. He has referred more than five books among them minimum
one is from aboard author.

Signature of HD

Based on the above certifications, I authorize this course material as the


subject referencefor the students concerned.

Signature of Principal
SYLLABUS
MA6251 MATHEMATICS – II LTPC
3104
OBJECTIVES:
 To make the student acquire sound knowledge of techniques in solving ordinary differential
equations that model engineering problems.
 To acquaint the student with the concepts of vector calculus, needed for problems in all
engineering disciplines.
 To develop an understanding of the standard techniques of complex variable theory so as to
enable the student to apply them with confidence, in application areas such as heat conduction,
elasticity, fluid dynamics and flow the of electric current.
 To make the student appreciate the purpose of using transforms to create a new domain in
which it is easier to handle the problem that is being investigated.

UNIT I VECTOR CALCULUS 9+3

Gradient, divergence and curl – Directional derivative – Irrotational and solenoidal vector fields –
Vector integration – Green’s theorem in a plane, Gauss divergence theorem and Stokes’ theorem
(excluding proofs) – Simple applications involving cubes and rectangular parallelopipeds.

UNIT II ORDINARY DIFFERENTIAL EQUATIONS 9+3

Higher order linear differential equations with constant coefficients – Method of variation of
parameters – Cauchy’s and Legendre’s linear equations – Simultaneous first order linear equations with
constant coefficients.

UNIT III LAPLACE TRANSFORM 9+3

Laplace transform – Sufficient condition for existence – Transform of elementary functions – Basic
properties – Transforms of derivatives and integrals of functions - Derivatives and integrals of transforms
- Transforms of unit step function and impulse functions – Transform of periodic functions.Inverse
Laplace transform -Statement of Convolution theorem – Initial and final value theorems –Solution of
linear ODE of second order with constant coefficients using Laplace transformation techniques.

UNIT IV ANALYTIC FUNCTIONS 9+3

Functions of a complex variable – Analytic functions: Necessary conditions – Cauchy-Riemann


equations and sufficient conditions (excluding proofs) – Harmonic and orthogonal properties of
analytic function – Harmonic conjugate – Construction of analytic functions – Conformal mapping:
w =z+k, kz, 1/z, z2, ez and bilinear transformation.

UNIT V COMPLEX INTEGRATION 9+3

Complex integration – Statement and applications of Cauchy’s integral theorem and Cauchy’s integral
formula – Taylor’s and Laurent’s series expansions – Singular points – Residues – Cauchy’s residue
theorem – Evaluation of real definite integrals as contour integrals around unit circle and semi-
circle(excluding poles on the real axis).
TOTAL: 60 PERIODS

TEXT BOOKS:

1. Bali N. P and Manish Goyal, “A Text book of Engineering Mathematics”, Eighth Edition, Laxmi
Publications Pvt Ltd.,(2011).
2. Grewal. B.S, “Higher Engineering Mathematics”, 41st Edition, Khanna Publications, Delhi,
(2011).

REFERENCES:

1. Dass, H.K., and Er. Rajnish Verma,” Higher Engineering Mathematics”, S. Chand Private Ltd., (2011)
2. Glyn James, “Advanced Modern Engineering Mathematics”, 3rd Edition, Pearson Education,
(2012).
3. Peter V. O’Neil,” Advanced Engineering Mathematics”, 7th Edition, Cengage learning, (2012).
4. Ramana B.V, “Higher Engineering Mathematics”, Tata McGraw Hill Publishing Company, New
Delhi, (2008).
CONTENTS

S.NO. TOPICS PAGE NO.

1 VECTOR CALCULUS
1.1 Directional derivative 1

1.2 Gradient, divergent and curl 2

1.3 Irrotational and solenoidal vector fields 3

1.4 Vector integration 9

1.5 Application problems 13

1.6 Green’s theorem in a plane 14


1.7 Stokes’ theorem 19
1.8 Gauss divergent theorem 25

2 ORDINARY DIFFERENTIAL EQUATIONS


2.1 Higher order linear equations with constant coefficients 27
2.2 Method of variation of parameters 39

2.3 Cauchy’s and Legendre’s linear equations 46


2.4 Simultaneous first order linear equations with constant 50
coefficients

3 LAPLACE TRANSFORM
3.1 Laplace transform 52

3.2 Sufficient conditions for existence 53

3.3 Transform of elementary functions 54


3.4 Basic properties 55

3.5 Transforms of derivatives and integrals of functions 62


3.6 Initial and final value theorems 64

3.7 Solutions of linear ODE of second order with constant 69


coefficients
4 ANALYTIC FUNCTIONS
4.1 Analytic functions: necessary conditions 75

4.2 Cauchy –Riemann equations and sufficient conditions 76

4.3 Construction of analytic functions 77

4.4 Properties of analytic functions 82


4.5 Harmonic conjucate 87

4.6 Conformal mapping: w=z+k,kz,1/z,z2,ez and Bilinear 92


transformation

5 COMPLEX INTEGRATION

5.1 Statement and applications of Cauchy’s integral theorem and 101


Cauchy’s integral formula

5.2 Taylor’s and Laurent’s series expansions 105

5.3 Singular points 110

5.4 Residues: Cauchy’s residue theorem 113

5.5 Evaluvation of real definite integrals as contour integrals around 116


unit circle and semi-circle

UNITWISE FORMULAE

PREVIOUS YEAR UNIVERSITY QUESTIONS


MA6251 MATHEMATICS - II

CHAPTER - 1
VECTOR CALCULUS

1.1 Directional derivative


The derivative of a point function (scalar or vector) in a particular
direction is called its directional derivative along the direction.
The directional derivative of a scalar point function φ in a given
direction is the rate of change of φ in the direction. It is given by the
component of grad φ in that direction.
The directional derivative of a scalar point function

→ ∇φ . a
φ (x,y,z) in the direction of a is given by →
.
a
Directional derivative of φ is maximum in the direction of ∇φ .
Hence the maximum directional derivative is ∇φ or gradφ
Unit normal vector to the surface
If φ (x, y, z) be a scalar function, then φ (x, y, z) = c represents
A surface and the unit normal vector to the surface φ is given by
∇φ
∇φ
Equation of the tangent plane and normal to the surface

Suppose a is the position vector of the point ( x0 , y0 , z0 )
→ → → →
On the surface φ (x, y, z) = c. If r = x i + y j + z k is the position vector of

any point (x,y,z) on the tangent plane to the surface at a , then the

equation of the tangent plane to the surface φ at a given point a on it is
→ →
given by  r − a .gradφ = 0
 

If r is the position vector of any point on the normal to the surface

at the point a on it. The vector equation of the normal at a given point

→ →
a on the surface φ is  r − a  × gradφ = 0
 
The Cartesian form of the normal at ( x0 , y0 , z0 ) on the surface

φ (x,y,z) = c is
x − x0 y − yo z − z0
= =
∂φ ∂φ ∂φ
∂x ∂y ∂z
1.2 Divergence of a vector

If F ( x, y, z ) is a continuously differentiable vector point function in

a given region of space, then the divergences of F is defined by
→ → →
→ → ∂F →∂F →∂F

∇. F = div F = i + j +k
∂x ∂y ∂z
BIET 1 DEPARTMENT OF MATHEMATICS
MA6251 MATHEMATICS - II


∂F →
∑ i ∂x
→ → → → → → → →
If F = F1 i + F2 j + F3 k ,then div F = ∇.( F1 i + F2 j + F3 k )
→ ∂F ∂F ∂F
i.e., div F = 1 + 2 + 3
∂x ∂y ∂z

1.3 Solenoidal Vector


→ → →
A vector F is said to be solenoidal if div F = 0 (ie) ∇. F = 0

Curl of vector function



If F ( x, y, z ) is a differentiable vector point function defined at each

point (x, y, z), then the curl of F is defined by
→ →
curl F = ∇ × F
→ → →
→ ∂F → ∂F → ∂F
= i× + j× + k×
∂x ∂y ∂z

∂F

= ∑i×
∂x
→ → → → → → → →
If F = F1 i + F2 j + F3 k ,then curl F = ∇ × ( F1 i + F2 j + F3 k )

i j k
→ ∂ ∂ ∂
curl F =
∂x ∂y ∂z
F1 F2 F3
→  ∂F ∂F  →  ∂F ∂F  →  ∂F ∂F 
= i  3 − 2  − j 3 − 1  + k 2 − 1 
 ∂y ∂z   ∂x ∂z   ∂x ∂y 
→ →
Curl F is also said to be rotation F

Irrotational Vector
→ →
A vector F is called irrotational if Curl F = 0

(ie) if ∇ × F = 0
Scalar Potential

If F is an irrotational vector, then there exists a scalar function φ
→ →
Such that F = ∇φ . Such a scalar function is called scalar potential of F

Properties of Gradient

1. If f and g are two scalar point function that ∇( f ± g ) = ∇f ± ∇g (or)


grad ( f ± g ) = gradf ± gradg
→ ∂ → ∂ → ∂ 
Solution: ∇( f ± g ) =  i + j + k ( f ± g )
 ∂x ∂y ∂z 

BIET 2 DEPARTMENT OF MATHEMATICS


MA6251 MATHEMATICS - II

→ ∂ → ∂ → ∂ 
=  i ( f ± g ) + j ( f ± g ) + k ( f ± g )
 ∂x ∂y ∂z 

∂f →
∂g →
∂f →
∂g →
∂f →
∂g
= i ±i + j ± j +k ±k
∂x ∂x ∂y ∂y ∂z ∂z
 
 → ∂f → ∂f → ∂f   → ∂g → ∂g → ∂g 
=  i + j + k  ±  i + j +k
 ∂x ∂y ∂z   ∂x ∂y ∂z 
 
= ∇f ± ∇g

2. If f and g are two scalar point functions then ∇( fg ) = f∇g + g∇f (or)
grad ( fg ) = fgradg + ggradf
→ ∂ → ∂ → ∂ 
Solution: ∇( fg ) =  i + j + k ( fg )
 ∂x ∂y ∂z 
→ ∂ → ∂ → ∂ 
=  i ( fg ) + j ( fg ) + k ( fg )
 ∂x ∂y ∂z 

 ∂g ∂f  → ∂g ∂f  → ∂g ∂f 
= i f + g  + j  f + g  + k  f +g 
 ∂x ∂x   ∂y ∂y   ∂z ∂z 
 → ∂g → ∂g → ∂g   → ∂f → ∂f → ∂f 
= f  i + j +k  + g  i + j + k 
 ∂x ∂y ∂z   ∂x ∂y ∂z 
= f∇g + g∇f

 f  g∇f − f∇g
3. If f and g are two scalar point function then ∇  = where
g g2
g≠0
f  → ∂ → ∂ → ∂  f 
Solution: ∇  =  i + j + k  
g  ∂x ∂y ∂z  g 
→ ∂  f 
= ∑ i  
∂x  g 
 ∂f ∂g 
g
→ −f 
= ∑i  ∂x ∂x 
 g2 
 
 
1  →
∂f →
∂g 
= 2  g∑ i − f∑i 
g  ∂x ∂x 
= 2 [g∇f − f∇g ]
1
g

→ → → → → →
4. If r = x i + y j + z k such that r = r ,prove that ∇r n = nr n − 2 r

→ ∂ → ∂ → ∂  n
Solution: ∇r n =  i + j + k r
 ∂x ∂y ∂z 
 → ∂r n → ∂r n → ∂r n 
=  i + j +k 
 ∂x ∂y ∂z 
BIET 3 DEPARTMENT OF MATHEMATICS
MA6251 MATHEMATICS - II


∂r → n −1 ∂r → n −1 ∂r
= i nr n −1 + j nr + k nr
∂x ∂y ∂z
→ x → y → z 
= nr n −1  i + j + k 
 r r r
nr n −1  → → → 
= x i + y j+ z k 
r  
nr n −1 →
= r
r

5. Find a unit normal to the surface x 2 y + 2 xz = 4 at (2,-2, 3)


Solution: Given that φ = x 2 y + 2 xz

→ ∂ → ∂ → ∂  2
∇φ =  i + j + k ( x y + 2 xz )
 ∂x ∂y ∂z 

( )
→ → →
= i (2 xy + 2 z ) + j x 2 + k (2 x )
At (2,-2, 3)

→ → →
∇φ = i (− 8 + 6) + j (4) + k (4)
→ → →
= − 2 i + 4 j+ 4k

∇φ = 4 + 16 + 16 = 36 = 6
Unit normal to the given surface at (2,-2,3)

→ →
∇φ − 2 i + 4 j + 4 k
=
∇φ 6
1  → → →
= − i + 2 j+ 2 k 
3 

6. Find the directional derivative of φ = x 2 yz + 4 xz 2 + xyz at (1,2,3) in the


→ → →
direction of 2 i + j − k
Solution: Given φ = x 2 yz + 4 xz 2 + xyz

→ ∂ → ∂ → ∂  2
∇φ =  i + j + k ( x yz + 4 xz 2 + xyz )
 ∂x ∂y ∂z 

( ) ( ) ( )
→ → →
= 2 xyz + 4 z 2 + yz i + x 2 z + xz j + x 2 y + 8 xz + xy k
At (1, 2, 3)

→ → →
∇φ = 54 i + 6 j + 28 k
→ → → →
Given: a = 2 i + j − k

∴ a = 4 +1+1 = 6

BIET 4 DEPARTMENT OF MATHEMATICS


MA6251 MATHEMATICS - II


a
∴ D. = ∇φ . →
a
→ → →
 → →
 2 i + 2 j− k

=  54 i + 6 j + 28 k .
  6
=
1
[108 + 6 − 28] = [86]
1
6 6

7. Find the angle between the surface x 2 + y 2 + z 2 = 5 and


x 2 + y 2 + z 2 − 2 x = 5 at (0,1,2)
Solution: Let φ1 = x 2 + y 2 + z 2 and φ2 = x 2 + y 2 + z 2 − 2 x
∂φ1 ∂φ ∂φ
= 2 x, 1 = 2 y , 1 = 2 z
∂x ∂y ∂z
∂φ2 ∂φ ∂φ
= 2 x − 2, 2 = 2 y, 2 = 2 z
∂x ∂y ∂z
→ → →
∇φ1 = 2 x i + 2 y j + 2 z k
→ → →
∇φ2 = (2 x − 2) i + 2 y j + 2 z k
At (o,1,2)
→ →
∇φ1 = 2 j + 4 k
→ → →
∇φ2 = −2 i + 2 j + 4 k
 → →  → → →
 2 j + 4 k . − 2 i + 2 j + 4 k 
∇φ1.∇φ2   
Cos θ = =
∇φ1 ∇φ2 16 + 4 4 + 4 + 6
4 + 16 20
cosθ = =
20 24 20 24
 20 
θ = cos −1  
 20 24 
 20 
= cos −1  

 24 

8. Find the angle between the surfaces x log z = y 2 − 1 and x 2 y = 2 − z at the


point (1,1,1)
Solution: let φ1 = y 2 − x log z and φ2 = x 2 y + z
∂φ1 ∂φ ∂φ x
= − log z , 1 = 2 y, 1 = −
∂x ∂y ∂z z
∂φ2 ∂φ ∂φ
= 2 xy, 2 = x 2 , 2 = 1
∂x ∂y ∂z
→ →
k→
∇φ1 = (− log z ) i + 2 y j − k
z
→ →
∇ φ2 = 2 j − k

BIET 5 DEPARTMENT OF MATHEMATICS


MA6251 MATHEMATICS - II

∇φ1.∇φ2 2 −1 1
Cos θ = = =
∇φ1 ∇φ2 4 +1 4 +1+1 5 6
 1 
θ = cos −1  
 5 6

9. Find ∇ 2 r n ( )
( )
Solution: ∇ 2 r n = ∇.∇ r n ( )
∂ n → ∂ n →∂ n
( ) ( ) ( )

= i r + j r +k r
∂x ∂y ∂z

∂r → ∂r → ∂r
= i nr n −1 + j nr n −1 + k nr n −1
∂x ∂y ∂z
→ → → →
r = x i + y j+ z k

r = r = x2 + y 2 + z 2

r 2 = x2 + y 2 + z 2
∂r ∂r x
2r = 2 x ⇒ =
∂x ∂x r
∂r ∂r y
2r = 2y ⇒ =
∂y ∂y r
∂r ∂r z
2r = 2 z ⇒ =
∂z ∂z r
( ) → x → y → z 
∴ ∇ 2 r n = nr n −1  i + j + k 
 r r r
 → → →
= nr n − 2  x i + y j + z k 
 
n − 2 → 
= nr  r 
 
→ → →
 
Since ∇ φ u  = ∇φ . u + φdiv u
 
( )

 
∇ 2 r n = ∇ nr n − 2 r 
 
( )
→ →
 
= nr n − 2  ∇. r  + ∇ nr n − 2 . r
 
→  ∂
→ → ∂ → ∂  → → → 
∇. r =  i + j + k  x i + y j + z k 
 ∂x ∂y ∂z  
=1+1+1 = 3

( ) ( )

∇ 2 r n = 3nr n − 2 + n∇ r n − 2 . r
( )
= 3nr n − 2 + n(n − 2) r n − 4 .r 2
( )
= 3nr n − 2 + n(n − 2) r n − 2

( ) [ ]
∇ 2 r n = r n − 2 n 2 + n = n(n + 1)r n − 2
BIET 6 DEPARTMENT OF MATHEMATICS
MA6251 MATHEMATICS - II

→ → → → → →
10. If r = x i + y j + z k and r = r .Prove that r n r is solenoidal if n = −3 and

r n r is irrotational for all vectors of n.
→ → → →
Solution: r n r = r n x i + r n y j + r n k
 → ∂ n ∂ n ∂ n
div  r n r  = ( )
r x + r y + ( ) ( )
r z …………………(1)
  ∂x ∂y ∂z
Now r 2 = x 2 + y 2 + z 2
Differentiating partially w.r.to x,

∂r ∂r x
2r = 2x ⇒ =
∂x ∂x r
∂r ∂r y
Similarly, 2r = 2y ⇒ =
∂y ∂y r
∂r ∂r z
2r = 2 z ⇒ =
∂z ∂z r
∂ n ∂ ∂r
Now
∂x
(r x ) = x (r n ). + r n
∂r ∂x
x
= x.n r n −1 + r n
r
∂ n
∂y
( r y ) = nr n − 2 y 2 + r n

∂ n
∂z
(r z ) = nr n − 2 z 2 + r n
From (1) we have
div r n r  = nr n − 2 (x 2 + y 2 + z 2 ) + 3r n
 →
 
= nr n + 3r n
= (n + 3)r n

 →
The vector r n r is solenoidal if div r n r  = 0
 
⇒ (n + 3)r n = 0
⇒ n+3=0
⇒ n = −3

∴ r n r is solenoidal only if n = -3
→ → →
i j k
 →
 ∂ ∂ ∂
Now curl  r n r  =
  ∂x ∂y ∂z
rnx rn y rnz
 ∂ ∂
∑ i  ∂y (r z ) − ∂z (r y )

n  n
=
 
 → ∂r ∂r 
= ∑ i  nr n −1 z − nr n −1 y 
 ∂y ∂z 

BIET 7 DEPARTMENT OF MATHEMATICS


MA6251 MATHEMATICS - II


 y z
= ∑ i  nr n −1

r
z − nr n −1 y 
r

∑ i (nr )

n−2
= yz − nr n − 2 yz
=0
→ → → →
Curl ( r n r ) = 0 i + 0 j + 0 k =0


Curl ( r n r ) = 0 for all values of n

Hence r n r is irrotational for all values of n.

( )
→ → → →
11. Prove that F = y 2 cos x + z 3 i + (2 y sin x − 4) j + 3xz 2 k is irrotational and
find its scalar potential
Solution:

→ → →
i j k
→ ∂ ∂ ∂
curl  F  =
  ∂x ∂y ∂z
y cos x + z 3
2
2 y sin x − 4 3xz 2

[ ]
→ → →
= i [0 − 0] − j 3z 2 − 3 z 2 + k [2 y cos x − 2 y cos x ] = 0

∴ F is irrotational.


To Find φ such that F = gradφ
∂φ → ∂φ → ∂φ
∴ ( y 2 cos x + z 2 ) i + (2 y sin x − 4 ) j + 3xz 2 k = i
→ → → →
+ j +k
∂x ∂y ∂z
Integrating the equation partially w.r.to x,y,z respectively
φ = y 2 sin x + xz 3 + f1 ( y, z )
φ = y 2 sin x − 4 y + f 2 ( x, z )
φ = xz 3 + f3 ( x, y )
∴φ = y 2 sin x + xz 3 − 4 y + C , is scalar potential

→ → → → → →
12. Prove that div A× B  = B .(curl A) − A.(curl B)
 
→ → → →
 
Proof : div A× B  = ∇.( A× B)
 

∂ → →
= ∑ i  A× B 
∂x  
→ →
→ → ∂ B  → ∂ A →

= ∑ i  A×  +∑ i × B 
 
∂x   ∂x 
   
→ →
→ ∂ B → → ∂ A →
= −∑ i   
× A + ∑ i  × B 
 ∂x   ∂x 
   
BIET 8 DEPARTMENT OF MATHEMATICS
MA6251 MATHEMATICS - II

 → → →  → → →
∂B ∂ A
= −  ∑ i × . A+  ∑ i × .B
 ∂x   ∂x 
   
→ → → →
   
= −  curl B . A+  curl A . B
   

→ → →
   
13.Prove that curl  curl F  = ∇ ∇ • F  − ∇ 2 F
   
Solution:

→ →
   
curl  curl F  = ∇ ×  ∇ × F 
   
→ → → → → →
    → →→
By using a×  b× c  =  a . c  b −  a . b  c
     
→ →
 
=  ∇. F ∇ − (∇.∇ ) F
 
→ →
 
=  ∇. F ∇ − ∇ 2 F
 

1.4 VECTOR INTEGRATION

Line, surface and Volume Integrals


Problems based on line Integral
Example 1:
( )
→ → → → →
If F = 3x 2 + 6 y i − 14 yz j + 20 xz 2 k Evaluate ∫ F .dr from (0,0,0) to
C

(1,1,1) along the curve x = t , y = t , z = t 2 3

Solution: The end points are (0, 0, 0) and (1, 1, 1)


These points correspond to t = 0 and t = 1
∴ dx = dt , dy = 2t , dz = 3t 2

( )

∫ F .dr = ∫ 3x + 6 y dx − 14 yzdy + 20 xz dz
2 2

C C

∫ (3t ) ( )
1
= 2
+ 6t 2 dt − 14t 5 (2tdt ) + 20t 7 3t 2 dt
0

= ∫ (9t 2 − 28t 6 + 60t 9 )dt


1

(
= 3t 3−4t 7 + 6t10 0 )
1

= [(3 − 4 + 6 ) − 0] = 5

Example 2:
→ → → →
Show that F = x 2 i + y 2 j + z 2 k is a conservative vector field.

BIET 9 DEPARTMENT OF MATHEMATICS


MA6251 MATHEMATICS - II

→ →
Solution: If F is conservative then ∇ × F = 0
→ → →
i j k
→ ∂ ∂ ∂ → → →
Now ∇ × F = = 0 i + 0 j+ 0 k = 0
∂x ∂y ∂z
x2 y2 z2

∴ F is a conservative vector field.

Surface Integrals
Definition: Consider a surface S. Let n denote the unit outward normal to the

surface S. Let R be the projection of the surface x on the XY plane. Let f be
a vector valued defined in some region containing the surface S. Then the
→ ∧
→ → ∧ f .n
surface integral of f is defined to be ∫∫ f . nds = ∫∫
S R


dx.dy
n. k

Example 1;
→ ∧ → → → →
Evaluate ∫∫
S
f . nds where F = z i + x j − y 2 z k and S is the surface of

the cylinder x + y 2 = 1 included in the first octant between the planes z = 0


2

and z = 2.
→ → → →
Solution: Given F = z i + x j − y 2 z k

φ = x2 + y2 − 1

→ →
∇φ = 2 x i + 2 y j
∇φ = 4 x 2 + 4 y 2
=2 x 2 + y 2
=2
∧ ∇φ
The unit normal n to the surface =
∇φ
2 xi + 2 yj
= = xi + yj
2
→ ∧
 → → →
  → →
F . n =  z i + x j − y 2 z k . x i + y j  = xz + xy
  

BIET 10 DEPARTMENT OF MATHEMATICS


MA6251 MATHEMATICS - II

BIET 11 DEPARTMENT OF MATHEMATICS


MA6251 MATHEMATICS - II

BIET 12 DEPARTMENT OF MATHEMATICS


MA6251 MATHEMATICS - II

BIET 13 DEPARTMENT OF MATHEMATICS


MA6251 MATHEMATICS - II

1.5 INTEGRAL THEOREMS

(I)Gauss’s divergence theorem


(II)Stoke’s theorem
(III)Green’s theorem in the plane

1.6 Green’s Theorem

Statement:

If M(x,y) and N(x,y) are continuous functions with continuous


partial derivatives in a region R of the xy plane bounded by a simple closed
curve C, then
 ∂N ∂M 
∫c Mdx + ndy = ∫∫R  ∂x − ∂y dxdy , where C is the curve described in the
positive direction.

BIET 14 DEPARTMENT OF MATHEMATICS


MA6251 MATHEMATICS - II

Verify Green’s theorem in a plane for the integral ∫ (x − 2 y )dx + xdy


c

taken around the circle x + y = 4


2 2

Solution: Green’s theorem gives


 ∂N ∂M 
∫c Mdx + Ndy = ∫∫R  ∂x − ∂y dxdy

Consider ∫ (x − 2 y )dx + xdy
c

M = x – 2y N=x
∂M ∂N
= −2, =1
∂y ∂x
 ∂N ∂M 
∴ ∫∫  − dxdy
R 
∂x ∂y 
∫∫ (1 + 2)dxdy = 3∫∫ dxdy
R R

= 3[Area of the circle]


= 3 πr 2
=3. π .4
= 12π ……………………(1)
Now ∫ Mdx + Ndy
We know that the parametric equation of the circle x 2 + y 2 = 4
x = 2 cos θ y = 2 sin θ
dx = −2 sin θdθ , dy = 2 cos θdθ
∴ Mdx + Ndy = ( x − 2 x )dx + xdy
= (2 cos θ − 4 sin θ )(− 2 sin θdθ ) + 2 cos θ (2 cos θ )dθ
= − 2 cos θ sin θ + 8 sin 2 θ + 4 cos 2 θdθ
Where θ various from 0 to 2π

∴ ∫ Mdx + Ndy = ∫ (− 2 cosθ sin θ + 4 sin θ + 4 )dθ
2

C 0

  1 − cos 2θ  
= ∫  − sin 2θ + 4
0
2
 + 4 dθ
 

= ∫ (− sin 2θ + 6 − 2 cos 2θ )dθ
0

 cos 2θ 2 sin 2θ 
= + 6θ −
 2 2  0
1 1
= + 12π − = 12π …………………….(2)
2 2
∴ From (1) and (2)

BIET 15 DEPARTMENT OF MATHEMATICS


MA6251 MATHEMATICS - II

 ∂N ∂M 
∫ Mdx + Ndy = ∫∫  ∂x
c R

∂y
dxdy

Hence Green’s Theorem is verified.

Example 2
Using Green’s theorems find the area of a circle of radius r.
Solution: By Green’s theorem we know that
1
Area enclosed by C = ∫ xdy − ydx
2C
The parametric equation of a circle of radius r is x = r cos θ , y = r sin θ
Where 0 ≤ θ ≤ 2π

1
∴ Area of the circle = ∫ r cos θ (r cos θ ) − r sin θ (− r sin θ )dθ
2 0

∫ (r )
1
= 2
cos 2 θ + r 2 sin 2 θ dθ
2 0

1 2
r dθ
2 ∫0
=

r [θ ]0 = πr 2
1 2 2π
=
2

Example 3:
Evaluate ∫ [(sin x − y )dx − cos xdy] where c is the triangle with
c

π π
vertices (0,0) ,(,0) and ( ,1)
2 2
y−0 x−0
Solution: Equation of OB is =
1− 0 π
−0
2
2x
⇒y=
π

BIET 16 DEPARTMENT OF MATHEMATICS


MA6251 MATHEMATICS - II

 ∂N ∂M 
By Green’s theorem ∫ Mdx + Ndy = ∫∫  ∂x
c R

∂y
dxdy

∂M
Here M = sin x − y, = −1
∂y
∂N
N = − cos x, = sin x
∂x
∴ ∫ [(sin x − y )dx − cos xdy ] = ∫∫ (sin x + 1)dxdy
C R

πy π
In the region R, x varies from x = to and y varies from y = 0 to y = 1
2 2
π
1 2
∴ ∫ (sin x − y )dx − cos xdy = ∫ π∫ (sin x + 1)dxdy
C 0 y
2
1 π
= ∫ [− cos x + x]π
0
2
y dy
2

 πy π πy 
1
= ∫ cos + − dy
0 
2 2 2
1
2 πy π πy 2 
=  sin + y − 
π 2 2 4 0
2 π π 2 π
= + − = +
π 2 4 π 2

Example 4
Verify Green’s theorem in the plane for
( )
∫ 3x − 8 y dx + (4 y − 6 xy )dy where C is the boundary of the region defined
2 2

by X = 0 , y= 0, x + y =1

Solution: We have to prove that

BIET 17 DEPARTMENT OF MATHEMATICS


MA6251 MATHEMATICS - II

 ∂N ∂M

∫ Mdx + Ndy = ∫∫  ∂x
c R
− dxdy
∂y

M = 3x − 8 y , N = 4 y − 6 xy
2 2

∂M ∂N
= −16 y, = −6 y
∂y ∂x

By Green’s theorem in the plane


 ∂N ∂M 
∫c Mdx + Ndy = ∫∫R  ∂x − ∂y dxdy

1 1− x
= ∫ ∫ (10 y )dydx
0 0
1− x
 y2 
1
= 10 ∫  
0 
2 0
1
= 5∫ (1 − x ) dx
2

 (1 − x )3 
1
5
= 5  =
 − 3 0 3
Consider ∫ Mdx + Ndy = ∫ + ∫ + ∫
c OA AB BO

Along OA, y=0 , x varies from 0 to 1

[ ]
1
1
∴ ∫ Mdx + Ndy = ∫ 3 x 2 dx = x 3 0 =1
OA 0

Along AB, y = 1 - x ⇒ dy = −dx and x varies from 1 to 0 .

[ ]
0
∴ ∫ Mdx + Ndy = ∫ 3x 2 − 8(1 − x ) − 4(1 − x ) + 6 x(1 − x ) dx
2

AB 1

 3x 2 8(1 − x )3 4(1 − x )2
0

=  − − + 3x 2 − 2 x 3 
 3 −3 −2 1
8 8
= + 2 −1− 3 + 2 =
3 3

BIET 18 DEPARTMENT OF MATHEMATICS


MA6251 MATHEMATICS - II

1.7 STOKE’S THEOREM


If S is an open surface bounded by a simple closed curve C and if a vector

function F is continuous and has continuous partial derivatives in S and on
→ → → → →
C, then ∫∫ curl F . n ds = ∫ F .d r where n is the unit vector normal to the
c

surface (ie) The surface integral of the normal component of curl F is equal

to the integral of the tangential component of F taken around C.

Example 1
→ → → →
Verify Stoke’s theorem for F = (2 x − y ) i − yz 2 j − y 2 z k where S is the upper
half of the sphere x 2 + y 2 + z 2 = 1 and C is the circular boundary on z = 0
plane.
→ → → →
Solution: By Stoke’s theorem ∫ F .d r = ∫∫ curl F . n ds
c s
→ → → →
F = (2 x − y ) i − yz 2 j − y 2 z k
→ → →
i j k
→ ∂ ∂ ∂
curl F =
∂x ∂y ∂z
2 x − y − yz 2 − y2z
→ → → →
= i [− 2 yz + 2 yz ] − j (0 − 0 ) + k (0 + 1) = k
→ →
Here n = k since C is the circular boundary on z = 0 plane
→ →
∴ ∫∫ = area of the circle curl F . n ds = ∫∫ dxdy
S S

= π (1) 2 = π ……….(1)

→ → → →
ON z = 0, ∫ F .d r = ∫∫ curl F . n ds
c s

On C, x = cos θ , y = sin θ
dx = − sin θdθ , dy = cos θdθ
θ varies from 0 to 2π

BIET 19 DEPARTMENT OF MATHEMATICS


MA6251 MATHEMATICS - II

→ → 2π
∴ ∫ F .d r =
c
∫ (2 cosθ − sin θ )(− sin θ )dθ
0
2π 2π
= − ∫ (2 cosθ sin θ )dθ + ∫ sin θdθ
2

0 0
2π 2π
 1 − cos 2θ 
=− ∫ (sin 2θ )dθ + ∫ 
0 0
 dθ
 2 
2π 2π
 cos 2θ  1 sin 2θ 
= −  + θ −
 2 0 2 2  0
1 1
= − + + π = π ……………(2)
2 2

∴ From (1) and (2)

→ → → →

∫ F .d r = ∫∫ curl F . n ds
c s

Hence stoke’s theorem is verified

Example 2
→ → → →
Verify stoke’s theorem for F = ( y − z + 2 ) i + ( yz + 4 ) j − xz k where s is
the surface of the cube x = 0, x = 2, y = 0, y = 2, z = 0 and z = 2 above the xy –
plane.
Solution:
By Stoke’s theorem
→ → → →

∫ F .d r = ∫∫ curl F . n ds
c s
→ → →
i j k
→ → → → → ∂ ∂ ∂
Given F = ( y − z + 2 ) i + ( yz + 4 ) j − xz k curl F =
∂x ∂y ∂z
y−z+2 yz + 4 − xz
→ → →
= i [0 − y ] − j [− z + 1] + k [0 − 1]
→ → →
= − y i + j [z − 1] − k

BIET 20 DEPARTMENT OF MATHEMATICS


MA6251 MATHEMATICS - II

BIET 21 DEPARTMENT OF MATHEMATICS


MA6251 MATHEMATICS - II

Hence Stoke’s theorem is verified.


Example 3:
→ → → →
Verify Stoke’s theorem for F = y i + z j + x k where S is the upper half
surface of the sphere x 2 + y 2 + z 2 = 1 and C is its boundary.
Solution: By stoke’s theorem

BIET 22 DEPARTMENT OF MATHEMATICS


MA6251 MATHEMATICS - II

BIET 23 DEPARTMENT OF MATHEMATICS


MA6251 MATHEMATICS - II

BIET 24 DEPARTMENT OF MATHEMATICS


MA6251 MATHEMATICS - II

→ → → →

∫ F .d r = ∫∫ curl F . n ds
c s

1.8 Gauss Divergence theorem


Statement:
The surface integral of the normal component of a vector
function F over a closed surface S enclosing volume V is equal to the volume
integral of the divergence of F taken throughout the volume V ,
→ ∧ →

∫∫ F . n ds = ∫∫∫ ∇. Fdv
S V

∫∫ x dydz + x ydzdx + x 2 zdxdy over the surface bounded by z = 0 ,z


3 2
Evaluate
= h, x 2 + y 2 = a 2
Solution:

BIET 25 DEPARTMENT OF MATHEMATICS


MA6251 MATHEMATICS - II

π
2
3 1 π 3π
∫ cos
4
θdθ = =
0
4 2 2 16

→ → 3a 4
∫∫S F . n ds = 2

BIET 26 DEPARTMENT OF MATHEMATICS


MA6251 MATHEMATICS - II

CHAPTER - 2

ORDINARY DIFFERENTIAL EQUATIONS

The study of a differential equation in applied mathematics consists of three phases.

(i) Formation of differential equation from the given physical situation, called
modeling.
(ii) Solutions of this differential equation, evaluating the arbitrary constants
from the given conditions, and
(iii) Physical interpretation of the solution.

2.1 HIGHER ORDER LINEAR DIFFERENTIAL EQUATIONS WITH


CONSTANT COEFFICIENTS.

General form of a linear differential equation of the nth order with constant
coefficients is

dny d n −1 y d n−2 y
+ K + K + .............. + K n y = X ………….. (1)
dx n −1 dx n − 2
1 2
dx n
Where K 1 , K 2,........................ K n are constants.

The symbol D stands for the operation of differential


dy d 2y d3y
(i.e.,) Dy = , similarly D y = 2 , D y = 3 , etc...
2 3

dx dx dx
The equation (1) above can be written in the symbolic form

(D n + K 1 D n −1 + .......... + K n ) y = X i.e., f(D)y = X


Where f (D) = D n + K 1 D n −1 + ........... + K n

Note
1
1. X = ∫ Xdx
D
1
2. X = e ax ∫ Xe − ax dx
D−a
1
3. X = e −ax ∫ Xe ax dx
D+a

(i) The general form of the differential equation of second order is

BIET 27 DEPARTMENT OF MATHEMATICS


MA6251 MATHEMATICS - II

d2y dy
2
+ P + Qy = R …………………(1)
dx dx

Where P and Q are constants and R is a function of x or constant.

(ii)Differential operators:

The symbol D stands for the operation of differential

dy d2y
(i.e.,) Dy = , D2 y = 2
dx dx
1
Stands for the operation of integration
D

1
Stands for the operation of integration twice.
D2

(1) can be written in the operator form


D 2 y + PDy + Qy = R (Or) ( D 2 + PD + Q) y = R

(iv) Complete solution = Complementary function + Particular Integral

PROBLEMS
1. Solve (D 2 −5 D + 6) y = 0

Solution: Given (D 2 −5 D + 6) y = 0
The auxiliary equation is m 2 −5m + 6 = 0`
i.e., m = 2,3
∴ C.F = Ae 2 x + Be 3 x
The general solution is given by
y = Ae 2 x + Be 3 x

d2y dy
2. Solve 2
− 6 + 3y = 0
dx dx
Solution: Given (D − 6 D + 3 y ) = 0
2

The auxiliary equation is m 2 − 6m + 13 = 0


6 ± 36 − 52
i.e., m =
2
= 3 ± 2i
Hence the solution is y = e 3 x ( A cos 2 x + B sin 2 x)

3. Solve (D 2 +1) = 0 given y(0) =0, y’’(0) = 1

BIET 28 DEPARTMENT OF MATHEMATICS


MA6251 MATHEMATICS - II

Solution: Given (D 2 +1) = 0


A.E is m 2 + 1 = 0
M = ±i
Y = A cosx + B sinx
Y(x) = A cosx + B sinx
Y(0) = A =0
Y’(0) =B =1
∴ A = 0, B = 1
i.e., y = (0) cosx + sinx
y = sinx

3. Solve ( D 2 − 4 D + 13) y = e 2 x
Solution: Given ( D 2 − 4 D + 13) y = e 2 x
The auxiliary equation is m 2 − 4m + 13 = 0
4 ± 16 − 52 4 ± − 36
m= = = 2 ± 3i
2 2
∴ C.F = e 2 x ( A cos 3x + B sin 3x )
1
P.I. = 2 e2x
D − 4 D + 13
1
= e2x
4 − 8 + 13
1
= e2x
9
∴ y = C.F +P.I.
y = e 2 x ( A cos 3x + B sin 3 x ) + e 2 x
1
9

5. Find the Particular integral of y’’- 3y’ + 2y = e x −e 2 x

Solution: Given y’’- 3y’ + 2y = e x −e 2 x

( D 2 − 3D + 2) y = e x − e 2 x
1
P.I 1 = 2 ex
D − 3D + 2
1
= ex
1− 3 + 4
1
= ex
0
1
=x ex
2D − 3
1 x
=x e
2−3
= − xe x

BIET 29 DEPARTMENT OF MATHEMATICS


MA6251 MATHEMATICS - II

P.I 2 =
1
(
− e2x )
D − 3D + 2
2

1
=- e2x
4−6+2
1
=-x e2x
2D − 3
1 2x
=-x e
4−3
= - xe 2 x

∴ P.I. = P.I 1 + P.I 2


= − xe x + (- xe 2 x )

= -x( e x + e 2 x )

d2y dy
6. Solve 2
− 4 + 5 y = −2 cosh x
dx dx
d2y dy
Solution: Given 2 − 4 + 5 y = −2 cosh x
dx dx
The A.E is m −4m + 5 = 0
2

− 4 ± 16 − 20
m= = −2 ± i
2
C.F = e −2 x ( A cos x + B sin x )
e x + e−x 
P.I =
1
( − 2 cosh x ) = −2
1
 
D 2 + 4D + 5 D 2 + 4D + 5  2 
−1 −1
= 2 ex + 2 e −x
D + 4D + 5 D + 4D + 5
−e x
e x
= −
1+ 4 + 5 1− 4 + 5
− e x e−x
= −
10 2
∴ y = C.F + P.I
e x e−x
= e −2 x ( A cos x + B sin x ) - −
10 2

sin ax(or )
1 1
Problems based on P.I = cos ax
f ( D) f ( D)
⇒ Replace D 2 by − a 2

d2y dy
7. Solve 2
+ 3 + 2 y = sin 3x
dx dx

BIET 30 DEPARTMENT OF MATHEMATICS


MA6251 MATHEMATICS - II

d2y dy
Solution: Given 2
+ 3 + 2 y = sin 3x
dx dx
The A.E is m +3m + 2 = 0
2

(m+1)(m+2) = 0
M = -1, m = -2
C.F = Ae − x + Be −2 x
1
P.I = 2 sin 3 x
D + 3D + 2
1
= sin 3 x (Replace D 2 by − a 2 )
− 3 + 3D + 2
2

1 1 (3D + 7)
= sin 3x = sin 3x
3D − 7 3D − 7 (3D + 7)
3D + 7
= sin 3x
(3D) 2 − (7) 2
3D + 7
= sin 3x
9 D 2 − 49
3D + 7
= sin 3x
9(−3 2 ) − 49
3D + 7
= sin 3x
− 130
= −
1
(3D(sin 3x) + 7 sin 3x )
130
= −
1
(9 cos 3x + 7 sin 3x )
130

∴ y = C.F + P.I
Y = Ae − x + Be −2 x −
1
(9 cos 3x + 7 sin 3x )
130

8. Find the P.I of (D 2 +1) = sin x


Solution: Given (D 2 +1) = sin x
1
P.I. = 2 sin x
D +1
1
= sin x
−1+1
1
=x sin x
2D
x 1
= sin x
2D
x
= ∫ sin xdx
2
x cos x
P.I =-
2

BIET 31 DEPARTMENT OF MATHEMATICS


MA6251 MATHEMATICS - II

9. Find the particular integral of (D 2 +1) y = sin 2 x sin x


Solution: Given (D 2 +1) y = sin 2 x sin x

=- (cos 3x − cos x )
1
2
1 1
= - cos 3x + cos x
2 2
1  1 
P.I 1 = 2  − cos 3x 
D +1  2 
1 1
=− cos 3 x
2 − 9 +1
1
= cos 3x
16

1 1 
P.I 2 =  cos x 
D +1 2
2

1 1
= cos x
2 −1+1
1 1
= x cos x
2 2D
x
= ∫ cos xdx
4
x
= sin x
4
1 x
∴P.I = cos 3x + sin x
16 4
Problems based on R.H.S = e ax + cos ax(or )e ax + cos ax

10. Solve (D 2 −4 D + 4) y = e 2 x + cos 2 x


Solution: Given (D 2 −4 D + 4) y = e 2 x + cos 2 x
The Auxiliary equation is m 2 −4m + 4 = 0
(m – 2 ) 2 = 0
m = 2 ,2
2x
C.F = (Ax +B)e
1
P.I 1 = 2 e2x
D − 4D + 4
1
= e2x
4−8+ 4
1
= e2x
0
1
=x e2x
2D − 4
1 2x
=x e
0

BIET 32 DEPARTMENT OF MATHEMATICS


MA6251 MATHEMATICS - II

1
= x 2 e2x
2

1
cos 2 x
P.I 2 = D − 4 D + 4
2

1
= cos 2 x
− 2 − 4D + 4
2

1
= cos 2 x
− 4D
−1  1 
=  cos 2 x 
4 D 
− 1 sin 2 x sin 2 x
= =−
4 2 8

∴ y = C.F + P.I
sin 2 x
y = (Ax +B)e 2 x −
8

Problems based on R.H.S = x

Note: The following are important


• (1 + x) −1 = 1 − x + x 2 − x 3 + .......
• (1 − x) −1 = 1 + x + x 2 + x 3 + .................
• (1 + x) −2 = 1 − 2 x + 3x 2 − 4 x 3 + ..............
• (1 − x) −2 = 1 + 2 x + 3x 2 + 4 x 3 + ..............

11. Find the Particular Integral of (D 2 +1) y = x


Solution: Given (D 2 +1) y = x
A.E is (m 2 −1) = 0
m = ±1
−x
C.F = Ae + Be x
1
P.I = 2 x
D −1
−1
= x
1− D2
[
= − 1− D2 ]
−1
x
[
= − 1 + D + (D 2 ) + ......... x
2
]
= − [x + 0 + 0 + 0...........]
=-x

(
12. Solve: D 4 − 2 D 3 + D 2 y = x 3 )

BIET 33 DEPARTMENT OF MATHEMATICS


MA6251 MATHEMATICS - II

( )
Solution: Given D 4 − 2 D 3 + D 2 y = x 3
The A.E is m 4 −2m 3 + m 2 = 0
m 2 (m 2 − 2m + 1) = 0
m 2 (m − 1) 2 = 0
m = 0,0 , m = 1,1
C.F = (A + Bx)e 0 x +(C + Dx)e x

1
P.I = x3
D − 2D + D
4 3 2

1
[
D 1 + (D 2 − 2 D )
= 2 x3
]
1
D
[
= 2 1 + (D 2 − 2 D ) x 3
−1
]
1
[ ( ) ( 2
) (
3
)
= 2 1 − D 2 − 2 D + D 2 − 2 D − D 2 − 2 D + ...............
D
]
1
[
= 2 1 + 2 D + 3D 2 + 4 D 3 + D 4 x 3
D
]
1
[
= 2 x 3 + 6 x 2 + 18 x + 24
D
]
1  x 4 6 x 3 18 x 3 
=  + + + 24 x 
D 4 3 2 
x 5 6 x 4 18 x 3 24 x 2
= + + +
20 12 6 2
5 4
x x
= + + 3x 3 + 12 x 2
20 2

∴ y = C.F + P.I
x5 x4
y = (A + Bx)e 0 x +(C + Dx)e x + + + 3x 3 + 12 x 2
20 2

Problems based on R.H.S = e ax x


1 1
P.I = e ax x = e ax x
f ( D) f ( D + a)

13. Obtain the particular integral of ( D 2 − 2 D + 5) y = e x cos 2 x


Solution: Given ( D 2 − 2 D + 5) y = e x cos 2 x

1
P.I = e x cos 2 x
D − 2D + 5
2

 
 cos 2 x(Re placeDbyD = 1)
1
= ex 
 (D + 1) − 2(D + 1) + 5 
2

BIET 34 DEPARTMENT OF MATHEMATICS


MA6251 MATHEMATICS - II

 1 
= ex  2 cos 2 x
 D + 2 D + 1 − 2 D − 2 + 5 
 1 
= ex  2 cos 2 x
 D + 4 
1
= ex cos 2 x
−4+4
1
= ex x cos 2 x
2D
xe x
2 ∫
= cos 2 xdx

xe x sin 2 x
P.I =
4

14. Solve ( D + 2) 2 y = e −2 x sin x


Solution: Given ( D + 2) 2 y = e −2 x sin x
A.E is (m 2 +1) = 0
m = -2, -2
C.F. = (Ax + B)e −2 x
1
P.I = e − 2 x sin x
(D + 2 ) 2

1
= e −2 x sin x
(
D −2+2 )2

1
= e − 2 x 2 sin x
D
1
= e −2 x sin x
−1
−2 x
= -e sin x

∴ y = C.F + P.I
y = (Ax + B)e −2 x - e −2 x sin x

Problems based on f(x) = x n sin ax(or ) x n cos ax


To find P.I when f(x) = x n sin ax(or ) x n cos ax
1
P.I = x n sin ax(or ) x n cos ax
f ( D)
1 1  d 1 
( xV ) = x V + V
f ( D) f ( D)  dD f ( D) 
1 1  f ' (D ) 1 
i.e., ( xV ) = x V − V
f ( D) f ( D)  f ( D) f ( D) 
1 1 f ' ( D)
xV = x V− V
f ( D) f ( D) [ f ( D)]2

BIET 35 DEPARTMENT OF MATHEMATICS


MA6251 MATHEMATICS - II

15. Solve ( D 2 + 4 D + 3) y = e − x sin x + xe 3 x


Solution: The auxiliary equation is m 2 +4m + 3 = 0
m= -1,-3
C.F =A e + Be −3 x
−x

P.I 1 =
1
(
e − x sin x )
( D + 4 D + 3)
2

1
=
[(D − 1) + 4(D − 1) + 3
2
]
sin x

1
= e −x 2
(
D + 2D )
sin x

= e −x
(1 + 2 D ) sin x
− 1 + 4D 2
e−x
= [2 cos x + sin x]
−5
1
P.I 2 = e 3 x
(
D + 3) + 4( D + 3) + 3
2
x
)
1
= e 3x 2
(
D + 10 D + 24
x
)
−1
 D 2 + 10 D 
e3x
= 1 +  x
24
 24 
e  5D 
3x
= 1 − x
24  12 
e3x  5
= x − 
24  12 
General solution is y = C.F + P.I
e−x
[2 cos x + sin x] + e  x − 5 
3x
y = A e − x + Be −3 x −
5 24  12 

d2y dy
16 Solve 2
+ 2 + y = x cos x
dx dx
Solution: A.E : m 2 +2m + 1 = 0
m = -1,-1
C.F = ( A + Bx)e − x
1
P.I = ( x cos x)
( D + 1) 2
 2( D + 1) 
= x − 2 
1
(cos x )
 ( D + 1)  ( D + 1) 2

BIET 36 DEPARTMENT OF MATHEMATICS


MA6251 MATHEMATICS - II

 2 
= x −
1
(cos x )

(
(D + 1) D + 2 D + 1
2
)
 2 
= x −
1
(cos x )
 D + 1 (− 1 + 2 D + 1)

 2  sin x
= x −
 D + 1 2
x sin x
=
2
x sin x sin x
= −
2 D +1
x sin x (D − 1)sin x
= −
2 D2 −1
x sin x cos x − sin x
= +
2 2

x sin x cos x − sin x


The solution is y = ( A + Bx)e − x + +
2 2

( )
17. Solve D 2 + 1 y = sin 2 x
Solution: A.E : m 2 +1 = 0
m = ±i
C.F =A cosx +B sinx
1
P.I = 2 sin 2 x
D +1
1  1 − cos 2 x 
= 2  
D +1 2 
1 1 1 
=  2 e0x − 2 cos 2 x 
2 D +1 D +1 
1 1 
= 1 + cos 2 x 
2 3 
1 1
= + cos 2 x
2 6
1 1
∴ y = A cosx +B sinx + + cos 2 x
2 6

d2y
18. Solve − y = x sin x + (1 + x )e x
dx 2
Solution: A.E : m 2 −1 = 0
m = ±1
C.F = A e − x + Be x
P.I 1 =
1
(xV ) =  x − f ' ( D)  1 (V )
f ( D)  f ( D)  f ( D)

BIET 37 DEPARTMENT OF MATHEMATICS


MA6251 MATHEMATICS - II

 2D 
= x − 2  2
1
(sin x )
 D − 1 (D − 1)
 2 D  sin x
= x − 2 
 D − 1 − 2
 x sin x 2 cos x 
= − +
2(D 2 − 1)
2 

x sin x cos x
=− −
2 2
P.I 2 = 2
1
(
1+ x2 ex )
D −1
1
(1 + x 2 )
= ex
[
(D + 1) − 1
2
]
1
= ex 2 (1 + x 2 )
( D + 2D )
e (2 x
x 3
− 3x 2 + 9 x )
=
12
e x (2 x 3 − 3x 2 + 9 x )
∴y = A e − x + Be x +
12

d2y
19. Solve − y = xe x sin x
dx 2
Solution: A.E : m 2 −1 = 0
m = ±1
C.F = A e − x + Be x
1
xe x sin x
P.I =
D −1
2
( )
1
(x sin x )
= ex
[
(D + 1)2 − 1 ]
= ex 2
1
(x sin x )
D + 2D ( )
 1 2D + 2 
= e x x 2 sin x −
 (D + 2 D )
sin x 
(2 D − 1) 2

sin αx = sin x, α = 1, putD = −α = −1
2 2


= e x x
1
sin x −
(2 D + 2) sin x 

 2D − 1 (2 D − 1)2 

= e x − x
(1 + 2 D ) sin x − (2 D + 2)sin x 
 (1 − 4 D ) (4 D 2 − 4 D + 1)
2

Put D 2 = −1
 (1 + 2 D )  (2 D + 2)(3 − 4 D )  
= e x − x sin x +   sin x 
 5  9 − 16 D 2
 

BIET 38 DEPARTMENT OF MATHEMATICS


MA6251 MATHEMATICS - II

− x 
[sin x + 2 cos x] +  − 8D − 2 D2 + 6  sin x
2
= ex
 5  9 − 16 D  
= ex
− x
(sin x + 2 cos x ) + (14 − 2 D ) sin x 
 5 25 
− x
P.I = e x  (sin x + 2 cos x ) + (14 sin x − 2 cos x )
 5 25 
Complete Solution is
y = A e − x + Be x +
− x
ex (sin x + 2 cos x ) + (14 sin x − 2 cos x )
 5 25 

2.2 METHOD OF VARIATION OF PARAMETERS


This method is very useful in finding the general solution of the second order
equation.

d2y dy
2
+ a1 + a 2 y = X [Where X is a function of x] ……………….(1)
dx dx
The complementary function of (1)
C.F = c 1 f 1 + c 2 f 2
Where c 1 , c 2 are constants and f 1 andf 2 are functions of x
Then P.I = Pf 1 +Qf 2

f2 X
P= −∫ dx
f 1 f − f1' f 2
2
1

f1 X
Q= ∫ f1 f 2 − f1' f 2
'
dx

∴ y = c1 f 1 + c 2 f 2 + P.I

( )
1. Solve D 2 + 4 y = sec 2 x
Solution: The A.E is m 2 +4 = 0
m = ± 2i
C.F = C 1 cos 2 x + C 2 sin 2 x
f 1 = cos 2 x f 2 = sin 2 x
f 1' = −2 sin 2 x f '2 = 2 cos 2 x
f 1 f 2' − f 1' f 2 = 2 cos 2 2 x + 2 sin 2 2 x
[
= 2 cos 2 2 x + sin 2 2 x]
= 2 [1]
=2
f2 X
P= −∫ dx
f 1 f 2 − f1' f 2
1

BIET 39 DEPARTMENT OF MATHEMATICS


MA6251 MATHEMATICS - II

sin 2 x sec 2 x
= −∫ dx
2
1 1
= − ∫ sin 2 x dx
2 cos 2 x
1 − 2 sin 2 x
= ∫ dx
4 cos 2 x
1
= log(cos 2 x)
4
f1 X
Q= ∫ dx
f1 f 2 − f1' f 2
'

cos 2 x sec 2 x
= ∫ 2
dx
1 1
= ∫ cos 2 x dx
2 cos 2 x
1
= ∫ dx
2
1
= x
2
P.I = Pf 1 +Qf 2
1 1
= log(cos 2 x) (cos2x) + x sin2x
4 2

d2y
2. Solve by the method of variation of parameters + y = x sin x
dx 2
Solution: The A.E is m 2 +1 = 0
m = ±i
C.F = C 1 cos x + C 2 sin x
Here f1 = cos x f 2 = sin x
f1 = − sin x
'
f 2 = cos x
'

f1 f 2 − f 1 f 2 = cos x + sin 2 x = 1
' ' 2

f2 X
P= −∫ dx
f 1 f − f1' f 2
2
1

sin x( x sin x)
= −∫ dx
1
= − ∫ x sin 2 xdx

= −∫x
(1 − cos 2 x ) dx
2
1
(x − x cos 2 x )dx
2∫
= −
1 1
= − ∫ xdx + ∫ x cos 2 xdx
2 2

BIET 40 DEPARTMENT OF MATHEMATICS


MA6251 MATHEMATICS - II

1  x 2  1   sin x   − cos 2 x 
=−   +  x  − (1) 
2 2  2  2   4 
x2 x 1
=− + sin 2 x + cos 2 x
4 4 8
f1 X
Q = ∫ f1 f 2' − f1' f 2
dx

(cos x) x(sin x)
= ∫ 1
dx

= ∫ x sin x cos xdx


sin 2 x
= ∫x dx
2
1
2∫
= x sin 2 xdx

1   − cos 2 x   − sin 2 x 
=  x  − (1) 
2  2   4 
x 1
= − cos 2 x + sin 2 x
4 8

P.I = Pf 1 +Qf 2
− x2 x 1  − x 1 
=  + sin 2 x + cos 2 x  cos x +  cos 2 x + sin 2 x  sin x
 4 4 8   4 8 

4. Solve (D 2 −4 D + 4) y = e 2 x by the method of variation of parameters.


Solution: The A.E is m 2 − 4m + 4 = 0
(m − 2)2 = 0
m = 2,2
C.F = ( Ax + B )e 2 x
= Axe 2 x + Be 2 x
f1 = xe 2 x f 2 = e2x
f1' = xe 2 x 2 + e 2 x , f 2' = 2e 2 x
f1 f 2' − f 2 f1' = 2 x(e 2 x ) − e 2 x (2 xe 2 x + e 2 x )
2

= 2x (e 2 x ) − 2 x(e 2 x ) − (e 2 x )
2 2 2

= (e 2 x ) [2 x − 2 x − 1]
2

= − (e 2 x )
2

= − e4x
f X
P= − ∫ 2
dx
f f − f f2
1 2
1
1
'

2x 2x
e e
= −∫ dx
e −4 x

BIET 41 DEPARTMENT OF MATHEMATICS


MA6251 MATHEMATICS - II

= ∫ dx = x
f1 X
Q= ∫f 1 f − f1 f 2
2
' '
dx

xe 2 x e 2 x
= ∫ − e 4 x dx
= ∫ − xdx
x2
=−
2
x 2 2x x 2 2x
P.I = x 2 e 2 x − e = e
2 2

y = C.F + P.I
x 2 2x
2x
= (Ax +B) e + e
2
5. Use the method of variation of parameters to solve (D 2 + 1)y = sec x
Solution: Given (D 2 + 1)y = sec x
The A.E is m 2 + 1 = 0
m = ±i
C.F = c1 cos x + c 2 sin x
= c1 f1 + c 2 f 2
f1 = cos x, f 2 = sin x
f1' = − sin x, f 2' cos x
f1 f 2' − f 1' f 2 = cos 2 x + sin 2 x = 1

f2 X
P= − ∫ dx
f 1 f − f1' f 2
2
1

sin x sec x
= −∫ dx
1
sin x
= −∫ dx
cos x
= − ∫ tan xdx
= log (cos x)
f1 X
Q= ∫ dx
f1 f 2 − f1' f 2
'

cos x sec x
= ∫
1
dx

= ∫ dx
=x
P.I = Pf 1 + Qf 2
= log(cos x) cos x + x sin x
y = c1 cos x + c 2 sin x + log(cos x) cos x + x sin x

BIET 42 DEPARTMENT OF MATHEMATICS


MA6251 MATHEMATICS - II

6. Solve (D 2 + a 2 )y = tan ax by the method of variation of parameters.


Solution: Given (D 2 + a 2 )y = tan ax
The A.E is m 2 + a 2 = 0
m = ± ai
C.F = c1 cos ax + c 2 sin ax
f1 = cos ax, f 2 = sin ax
f1' = − a sin x, f 2' = a cos ax
f1 f 2' − f 2 f1' = a cos ax cos ax − sin ax(− a sin ax)
= a cos 2 ax + a sin 2 ax
= a(cos 2 ax + sin 2 ax)
=a
P.I = Pf 1 + Qf 2

f2 X
P= − ∫ dx
f 1 f − f1' f 2
2
1

sin ax tan ax
= −∫ dx
a
1 sin 2 ax
=− ∫ dx
a cos ax
− 1 1 − cos 2 ax
a ∫ cos ax
= dx
−1
(sec ax − cos ax )dx
a ∫
=

−1 1 sin ax 
= log (sec ax + tan ax ) −
a  a a 
−1
= 2 [log(sec ax + tan ax ) − sin ax ]
a
= 2 [sin ax − log(sec ax + tan ax )]
1
a

f1 X
Q= ∫f 1 f − f1 f 2
2
' '
dx

cos ax tan ax
= ∫ a
dx
1
a∫
= sin axdx
1
= − 2 cos ax
a
∴ P.I = Pf1 + Qf 2

= 2 cos ax[sin ax − log(sec ax + tan ax )] − 2 sin ax[cos ax ]


1 1
a a

BIET 43 DEPARTMENT OF MATHEMATICS


MA6251 MATHEMATICS - II

=−
1
[cos ax log(sec ax + tan ax )]
a2
y = C.F + P.I
= c1 cos ax + c 2 sin ax − 2 [cos ax log(sec ax + tan ax )]
1
a

d2y
7. Solve + y = tan x by the method of variation of parameters.
dx 2
Solution: The A.E is m 2 + 1 = 0
m = ±i
C.F = c1 cos x + c 2 sin x
Here f1 = cos x, f 2 = sin x
f1' = − sin x, f 2' = cos x
f1 f 2' − f 2 f1' = cos 2 x + sin 2 x = 1
f2 X
P= − ∫ dx
f 1 f 2 − f1' f 2
1

sin x tan x
= −∫ dx
1
sin 2 x
= −∫ dx
cos x
1 − cos 2 x
= −∫ dx
cos x
= − ∫ (sec x − cos x )dx
= − log(sec x + tan x) + sin x
f1 X
Q= ∫ dx
f1 f 2 − f1' f 2
'

cos x tan x
= ∫ 1 dx

= ∫ sin dx
= − cos x
∴ P.I = Pf1 + Qf 2
= − cos x log(sec x + tan x)
y = C.F + P.I
= c1 cos x + c 2 sin x − cos x log(sec x + tan x)

4 ' 4
8. Solve by method of variation of parameters y '' − y + 2 y = x2 +1
x x
4 ' 4
Solution: Given y '' − y + 2 y = x2 +1
x x
i.e., x y − 4 xy ' + 4 y = x 4 + x 2
2 ''

[ ]
i.e., x 2 D 2 − 4 xD + 4 y = x 4 + x 2 ………….(1)
Put x = e z

BIET 44 DEPARTMENT OF MATHEMATICS


MA6251 MATHEMATICS - II

Logx = log e z
=z
So that XD = D '
x 2 D = D ' (D ' − 1)
[ ]
(1) ⇒ D ' (D ' − 1) − 4 D ' + 4 y = (e z ) + (e z )
4 2

[D '2
]
− 5D ' + 4 y = e 4 z + e
2z

A.E is m 2 − 5m + 4 = 0
(m − 4)(m − 1) = 0
m = 1,4
∴ C.F = c1e 4 z + c 2 e z
Here f1 = e 4 z , f 2 = e z
f1' = e 4 z , f 2' = e z
f1 f 2' − f 2 f1' = e 5 z − 4e 5 z = −3e 5 z
P.I = Pf1 + Qf 2
f2 X
P= − ∫ dx
f 1 f 2 − f1' f 2
1

= −∫
[
e z e4z + e2z
dx
]
− 3e 5 z
e5z + e3z
=∫ dx
3e 5 z
1
[
= ∫ 1 + e − 2 z dz
3
]
1 e −2 z 
= z + 
3 −2 
1 1
= z − e −2 z
3 6
f1 Z
Q= ∫ f1 f 2' − f 1' f 2
dz

e 4 z (e 4 z + e 3 z )
= ∫ − 3e 5 z dz
1 e8z + e 6 z
=− ∫ dz
3 e5z
1
(
= − ∫ e 3 z + e z dz
3
)
1  e3z 
= −  + ez 
3 3 
1 1
= − e3z − e z
9 3
1 1   1 1 
∴ P.I =  z − e − 2 z e 4 z +  − e 3 z − e z e z
3 6   9 3 

BIET 45 DEPARTMENT OF MATHEMATICS


MA6251 MATHEMATICS - II

1 1 1 1
= ze 4 z − e 2 z − e 4 z − e 2 z
3 6 9 3

y = C.F + P.I
1 1 1 1
= c1e 4 z + c 2 e z + ze 4 z − e 2 z − e 4 z − e 2 z
3 6 9 3

3
4 1
6
( )
= c1 (e z ) 4 + c 2 e z + z (e z ) − (e z ) − e z − (e z )
1 2 1
9
4 1
3
2

= c1 (e z ) + c 2 (e z ) +
4 4 2
4 x x x
log x − −
3 9 2

2.3 DIFFERENTIAL EQUATIONS FOR THE VARIABLE COEFFICIENTS


(CAUCHY’S HOMOGENEOUS LINEAR EQUATION)

Consider homogeneous linear differential equation as:

n −1
dny n −1 d y
an n
+ a 1 x x −1
+ ........ + a n y = X ....................(1)
dx dx
(Here a’s are constants and X be a function of X) is called Cauchy’s
homogeneous linear equation.
dny d n −1 y dy
a n x n n + a n −1 n −1 + .............. + a1 x + a 0 y = f ( x)
dx dx dx
is the homogeneous linear equation with variable coefficients. It is also known
as Euler’s equation.
Equation (1) can be transformed into a linear equation of constant Coefficients
by the transformation.
x = e z , (or ) z = log x
Then
dy dy dz 1 dy
Dy = = =
dx dz dx x dz
 d d
 D = dx , θ = dz 
d d
x = xD = =θ
dx dz
Similarly
d 2 y d  dy  d  1 dy 
=  =  
dx 2 dx  dx  dx  x dz 
1 dy 1 d  dy  1 dy 1 d  dy  dz
=− 2 +  =− 2 +  
x dz x dx  dz  x dz x dz  dz  dx
d2y 1 dy 1 d 2 y
= − +
dx 2 x 2 dz x 2 dz 2
d 2 y d 2 y dy
∴ x2 2 = 2 −
dx dz dz

BIET 46 DEPARTMENT OF MATHEMATICS


MA6251 MATHEMATICS - II

( )
∴ x 2 D 2 = θ 2 − θ = θ (θ − 1)
Similarly,
x 3 D 3 = θ (θ − 1)(θ − 2)

x 4 D 4 = θ (θ − 1)(θ − 2)(θ − 3)
and soon.

∴ xD = θ
x 2 D 2 = θ (θ − 1)
x 3 D 3 = θ (θ − 1)(θ − 2)

and so on.

d2y dy
1. Solve x 2 2
+ x + y = 4 sin(log x)
dx dx
Solution: Consider the transformation

x = e z , (or ) z = log x
∴ xD = θ
x 2 D 2 = θ (θ − 1)
(x 2 D 2 + xD + 1)y = 4 sin(log x)
(θ 2 + 1)y = 4 sin(z )
R.H.S = 0 : (θ 2 + 1)y = 0
A.E : m 2 + 1 = 0, m = ±i
C.F = A cosz + B sinz
C.F = A cos (log x) +B sin (log x)
4(sin z )
1
P.I = 2
θ +1
 z cos z 
= 4 −  = −2 z cos z
 2 
P.I = − 2 log x cos(log x)
∴ Complete solution is:
y = A cos (log x) +B sin (log x) − 2 log x cos(log x)
y = ( A − 2 log x ) cos(log x) − 2 log x cos(log x)

d2y dy
2. Solve x 2 2
+ 4 x + 2 y = x log x
dx dx
Solution: Given (x 2 D 2 + 4 xD + 2)y = x log x ……………(1)
Consider: x = e z , (or ) z = log x
∴ xD = θ
x 2 D 2 = θ (θ − 1)
(1) : (θ (θ − 1) + 4θ + 2) y = e z z

BIET 47 DEPARTMENT OF MATHEMATICS


MA6251 MATHEMATICS - II


+ 3θ + 2 y = ze z
2
)
A.E : m 2 + 3m + 2 = 0
M = - 2, -1
−2 −1
C.F = Ae −2 z + Be − z = Ae log x + Be log x
A B
C.F = 2 +
x x

)( )
1
ez z
P.I = 2
(
θ + 3θ + 2
1
=e z z
(θ + 1) + 3(θ + 1) + 2
2

1
= ez 2
θ + 5θ + 6 ( z
)
−1
ez   θ 2 + 5θ  
1 +  
=
6   6  z
  
ez  5  ez  5
= 1 − θ  z =  z − 
6  6  6  6
log x
e  5
=  log x − 
6  6
x 5
=  log x − 
6 6
Complete solution is y = C.F +P.I
A B x 5
= 2 + +  log x − 
x x 6 6

3. Solve ( ( x 2 D 2 − 3xD + 4) y = x 2 , giventhat y(1) = 1 and y’(1) = 0


( )
Solution: Given x 2 D 2 − 3xD + 4 y = x 2 …………………(1)
Take x = e z , (or ) z = log x
∴ xD = θ
x 2 D 2 = θ (θ − 1)
(
(1) : θ 2 − 4θ + 4 y = e 2 z )
A.E : m − 4m + 4 = 0, m = 2,2
2

C.F = ( A + Bz )e 2 z = ( A + B log x )x 2
P.I =
1
e2z = e2z ( ) 1
(1)
(θ − 2) 2
(θ + 2 − 2)2
= e 2 z 2 (1)
1
θ
z2
P.I = e 2 z
2

BIET 48 DEPARTMENT OF MATHEMATICS


MA6251 MATHEMATICS - II

x 2 (log x )
2
= …………………(2)
2
x 2 (log x )
2
Complete solution is : y = ( A + B log x )x 2 +
2
Apply conditions: y(1) =1,y’(1) = 0 in (2)
A = 1, B = -2
x 2 (log x )
2
Complete solution is y = (1 − 2 log x )x + 2

2.4 EQUATION REDUCIBLE TO THE HOMOGENEOUS LINEAR


FORM (LEGENDRE LINEAR EQUATION)

It is of the form:

n
d n −1 y
(a + bx )n d y
+ A1 (a + bx )
n −1
+ ........... + An y = f ( x) …………..(
dx n dx n −1
1)
A1 , A2,......................., An are some constants
It can be reduced to linear differential equation with constant
Coefficients,
by taking: a + bx = e z (or ) z = log(a + bx)
d d
Consider = D, = θ , gives
dx dz
(a + bx ) = b ⇒ (a + bx )Dy = bθ ( y )
dy dy
dx dz
Similarly (a + bx ) D 2 y = b 2θ (θ − 1) y …………(2)
2

(a + bx )3 D 3 y = b 3θ (θ − 1)(θ − 2) and so on
Substitute (2) in (1) gives: the linear differential equation of
constant Coefficients.

Solve (2 x + 3) y ' '−(2 x + 3) y '−12 y = 6 x


2

Solution: This is Legendre’s linear equation:


(2 x + 3)2 y' '−(2 x + 3) y '−12 y = 6 x ……………….(1)
Put z = log (2x + 3) , e z = 2 x + 3
(2 x + 3)D = 2θ
(
(2 x + 3)2 D 2 = 4 θ 2 − θ ,θ = d
dz
)
(
Put in (1) : 4θ − 6θ − 12 y = 3e z − 9
2
)
R.H.S =0
A.E : 4m 2 − 6m − 12 = 0
3 + 57 3 − 57
m1 = , m2 =
4 4
C.F = Ae m1z + Be m2 z
C.F = A(4 x + 3) m1 + B(2 x + 3) m2

BIET 49 DEPARTMENT OF MATHEMATICS


MA6251 MATHEMATICS - II

3e z
= − (2 x + 3)
3
P.I 1 =
4θ − 6θ − 12
2
14
θz
9e
P.I 2 =
4θ − 6θ − 12
2

9 3
=− =−
12 4
Solution is
(3+ )
y = A(2 x + 3) + B(2 x + 3) (2 x + 3) − 3
 3− 57  3
4  −
57 / 4

14 4

SIMULTANEOUS FIRST ORDER LINEAR EQUATIONS WITH


CONSTANT COEFFICIENTS

dx dy
1. Solve the simultaneous equations, + 2 x + 3 y = 2e 2t , + 3x + 2 y = 0
dt dt
dx dy
Solution: Given + 2 x + 3 y = 2e 2t , + 3x + 2 y = 0
dt dt
d
Using the operator D =
dt
(D + 2)x + 3 y = 2e ………………(1)
2t

3 x + (D + 2 ) y = 0 ……………….(2)
Solving (1) and (2) eliminate (x) :
3 × (1) − (2) × ( D + 2) ⇒ (D 2 + 4 D − 5)y = −6e 2t ..............(3)
A.E : m 2 + 4m − 5 = 0
m = 1,-5
C.F = Ae t + Be −5t
− 6e 2t 6
P.I = 2 = − e 2t
D + 4D − 5 7
6
y = Ae t + Be −5t − e 2t
7
put in (1) : x = − [(D + 2) y ]
1
3
8
x = − Ae t + Be −5t + e 2t
7
∴ solution is :

8
x = − Ae t + Be −5t + e 2t
7
6
y = Ae t + Be −5t − e 2t
7
dx dy
2. Solve + y = sin t , + x = cos t , giventhat t=0, x = 1, y =0
dt dt
Solution: Dx + y = sin t ……………..(1)
x + Dy = cost ……………… (2)
Eliminate x : (1) – (2)D ⇒ y − D 2 y = sin t + sin t

BIET 50 DEPARTMENT OF MATHEMATICS


MA6251 MATHEMATICS - II

(D )
− 1 y = −2 sin t...............(3)
2

m − 1 = 0, m = ±1
2

C.F = Ae t + Be − t
sin t sin t
P.I = − 2 2 = (−2) = sin t
D −1 −1−1
y = Ae t + Be − t + sint
(2) : x = cost –D(y)
x = cost -
d
dt
(
Ae t + Be −t + sin t )
x = cos t − Ae t + Be −t − cos t
x = − Ae t + Be − t
Now using the conditions given, we can find A and B
t = 0, x = 1 ⇒ 1 = − A + B
t = 0, y = 0 ⇒ 0 = A + B
1 1
B= ,A= −
2 2
Solution is
1 1
x = e t + e −t = cosh t
2 2

1 1
y = − e t + e −t + sin t = sin t − sinh t
2 2

dx dy
3. Solve + 2 y = − sin t , − 2 x = cos t
dt dt
Solution: Dx + 2y = -sin t ……………..(1)
- 2x +Dy = cos t ……………...(2)
(1) × 2 + (2) × D ⇒ 4 y + D 2 y = −2 sin t + D(cos t )
⇒ (D 2 + 4) = −3 sin t
m 2 + 4 = 0, m = ±i 2
C.F = A cos 2t + B sin 2t
3 sin t − 3 sin t
P.I = − 2 = = − sin t
D + 4 −1+ 4
y = A cos 2t + B sin 2t - sin t
(2) : x = [Dy − cos t ]
1
2
1d 
x =  ( A cos 2t + B sin 2t − sin t ) − cos t 
2  dt 
x = A cos2t +Bsin2t – cost
Solution is :

x = A cos2t +Bsin2t – cost


y = A cos 2t + B sin 2t - sin t

BIET 51 DEPARTMENT OF MATHEMATICS


MA6251 MATHEMATICS - II

CHAPTER - 3
LAPLACE TRANSFORM
3.1 Definition:

Let f(t) be a function of t defined for t>0. Then the laplace transform of f(t),
defined by L[f(t)] or F(s), is defined by

Provided the integral exists. Here s is the parameter which may be a real or
complex number.

LAPLACE TRANSFORM OF SOME ELEMENTARY FUNCTIONS:

1.

2.

3.

4.
5.
6.

7.
8.
9.

SOME IMPORTANT PROPERTIES OF LAPLACE TRANSFORM:

1. Linear property

2. First shifting theorem

BIET 52 DEPARTMENT OF MATHEMATICS


MA6251 MATHEMATICS - II

3. Second shifting theorem

4. Change of scale property

SOME STANDARD RESULTS IN LAPLACE TRANSFORM:

1.

2.

3.

4.

5.

6.

7.

8.

9.

10.

11.

BIET 53 DEPARTMENT OF MATHEMATICS


MA6251 MATHEMATICS - II

12.

13.

14.

15.
cos bt

16.

LAPLACE TRANSFORM OF DERIVATIVES:

1.
i.
ii. and etc.,
2. .

3.

4.

BIET 54 DEPARTMENT OF MATHEMATICS


MA6251 MATHEMATICS - II

3.2 PROBLLEMS:

Problem 1 State the conditions under which Laplace transform of f  t  exists


Solution:
(i) f  t  must be piecewise continuous in the given closed interval  a, b  where a  0 and
(ii) f  t  should be of exponential order.

Problem 2 Find (i) L t 3/ 2  (ii) L e at cos bt 


Solution:
(i) We know that
  n  1
L t n  
s n 1
3  3 3
   1  
L t n / 3    3  
2 2 2
1 s 5/ 2    n  1  n  n  
s2
3 1 
   1
2 2 

s5 / 2
3 1 1
.  
2 2 2

S 5/ 2
3 
 5/ 2  1/ 2    
4s
ii)
L e  at cos bt    L  cos bt   s s  a
 s 
 2
 s  b  s  s  a
2

 sa 
 
  s  a   b 
2 2

Problem 3 Find L sin 8t cos 4t  cos3 4t  5


Solution:
L sin 8t cos 4t  cos3 4t  5  L sin 8t cos 4t   L cos3 4t   L 5
 sin12t  sin 4t   sin( A  B)  sin( A  B) 
L sin 8t  cos 4t   L    sin A cos B  
 2 2

BIET 55 DEPARTMENT OF MATHEMATICS


MA6251 MATHEMATICS - II


1
2
L sin12t   L  sin 4t 
1  12 4 
  2  2 
2  s  144 s  16 
 cos12t  3cos 4t   cos 3  3cos  
L cos3 4t   L    cos3   
 4  4
 L  cos12t   3L  cos 4t 
1
4
1 s 3s 
  2  2
4  s  144 s  16 
1 5
L 5  5L 1  5    .
s s
1  12 4  1 s 3s  5
L sin 8t cos 4t  cos3 4t  5   2  2   2  2  .
2  s  144 s  16  4  s  144 s  16  s

0 ; when 0  t  2
Problem 4 Find L  f (t ) where f  t    .
3 ; when t2
Solution:

W.K.T L  f  t     e  st f  t  dt
0
2 
 e  st
f  t  dt   e  st f  t  dt
0 2
2 
  e  st 0.dt   e  st 3dt
0 2
 
 e st 
 3 e dt  3 
 st

2  s 2
 e  e 2 s  3e 2 s
 3  s .
 s 

Problem 5 If L  f  t   F  s  show that L  f  at   F   .


1 s
a a
(OR)
State and prove change of scale property.
Solution:

W.K.T L  f  t    e  st f  t  dt
0

BIET 56 DEPARTMENT OF MATHEMATICS


MA6251 MATHEMATICS - II


L  f  at    e  st f  at  dt
0

Put at  x when t  0 , x  0
adt  dx when t   , x  
  x
 s 
L  f  at    e
dx
a
f  x
0
a
 s
1  t
  e  a  f  t  dt  x is a dummy variable 
a0
1 s
 F  .
a a

cos at
Problem 6 Does Laplace transform of exist? Justify
t
Solution:

1  f (t ) 
If L  f (t )  F ( s ) and f (t ) has a limit as t  0 then L     F  s  ds .
t  t  S
cos at 1
Here tLim
0  
t 0
 cos at 
L   does not exist.
 t 


Problem 7 Using Laplace transform evaluate  te 3t sin 2t dt
0

Solution:

W.K.T L  f  t    e st f  t  dt
0

  e3t t sin 2t dt  L  t sin 2t   s 3
0

 d   d  2 
   L  sin 2t       2 
 ds  s 3  ds  s  4   s 3
  
  4 s  
 
   s 2  4 2 
    s 3
  
4s   12
    .
   s 2  4  
2
169
    s 3

BIET 57 DEPARTMENT OF MATHEMATICS


MA6251 MATHEMATICS - II

 t sin u 
Problem 8 Find L   du 
 0
u 
Solution:
t  1
By Transform of integrals, L   f  x  dx   L  f  t 
0  s
 sin u  1  sin t  1
t 
1

1
L  du   L     L sin t  ds   2 ds
0 u  s  t  ss s s s 1
1  1  
  tan 1 s     tan 1 s 
s s s2 
1
 cot 1 s
s

Problem 9 Find the Laplace transform of the unit step function.


Solution:
The unit step function (Heaviside’s) is defined as
0 ; t  a
U a (t )   , where a  0
1 ; t  a

W.K.T L  f (t )   e  st f (t ) dt
0

L U a (t )   e  st U a  t  dt
0
a 
  e  st  0  dt   e  st 1 dt
0 a

  e  st dt
a

 e st   e   e  as  e  as
    s
 s  a  s 
e as
Thus L U a  t  
s

1
Problem 10 Find the inverse Laplace transform of
(s  a)n
Solution:
L t n  
n!
s n 1

BIET 58 DEPARTMENT OF MATHEMATICS


MA6251 MATHEMATICS - II

L  t n 1  
 n  1!
sn
 (n  1)!
L  e at t n 1    n  
 n  1!
 s  s s  a ( s  a )
n

  n  1 ! 
e  at t n1  L1  n 
 ( s  a) 
 1 
e  at t n 1   n  1 ! L1  n 
  s  a  
 1  1
 L1  n 
 e at t n 1
  s  a    n  1 !

1
Problem 11 Find the inverse Laplace Transform of
s  s  a2 
2

Solution:
1 
t
W.K.T L1  F  s     L1  F ( s )  dt
s  0
 1  t  1 
L1     L1  2 dt
 s  s  a   0  s  a 
2 2 2

 a 
t
1
  L1  2 dt
 s  a 
2
0
a
t
1
a 0
 sin at dt

t
1  cos at 
 
a  a  0
1
  2  cos at  1
a
1
 2 1  cos at  .
a

 s 
Problem 12 Find L1  
  s  2  
2

Solution:
 s  d
L1  2
 L1  sF ( s )  L1  F ( s ) 
  s  2   dt

BIET 59 DEPARTMENT OF MATHEMATICS


MA6251 MATHEMATICS - II

1 n!
Where F ( s )  , L t n   n 1
 s  2
2
s
 1  1
L1  F  s    L1  2 
, L t   2
 ( s  2)  s
1
L1  F  s    e2t L1  2   e2t t
s 
 s  d 2 t
L1  2
 e t   t  2e 2t   e 2t
 ( s  2)  dt
 s  2 t
L1  2
 e 1  2t 
 ( s  2) 

 
 s2 1 .
Problem 13 Find L
  s 2  4 s  5 2 
 
Solution:
   
s  2 L  s2 
L 
1 1
 2 
  s  4s  5  
 
2
  s  2   1
2 2
  
 
s
 e 2 t L1   .......(1)
  s 2  1 2 
 
  
1  s  s
L t L  1
ds
  s  1 
2 2
s  s  1
2 2

 
du
 t L1  let u  s 2  1, du  2 sds
2u 2
t  1 
 L1  
2  u 

t 1  1 
 L  
2  s 2  12  s
t 1  1 
L  
2  s2 1 
t
 sin t........(2)
2
Using (2) in (1)
 
s2   e 2t . t sin t  1 te 2t sin t.
L1 
  s 2  4s  52  2 2
 

BIET 60 DEPARTMENT OF MATHEMATICS


MA6251 MATHEMATICS - II

100
Problem 14 Find the inverse Laplace transform of
s  s  100 
2

Solution:
100 A Bs
Consider   2
s  s  100  s s  100
2

100  A  s 2  100    Bs  C  ( s)
Put s  0, 100  A 100 
A 1
s  1, 100  A(101)  B  C
B  C  1
Equating s 2 term
0  A B
 B  1
 B  C  1 i.e., 1  C  1
C0
 100  1 s 
 L1  2   L1   2
 s ( s  100)   s s  100 
1  s 
 L1    L1  2
s  s  100 
 1  cos10t

dx dy
Problem 15 Solve  2 y  cos 2t and  2 x  sin 2t given x (0)  1; y (0)  0
dy dt
Solution:
x  2 y  cos 2t
y  2 x  sin 2t given x (0)  1; y (0)  0
Taking Laplace Transform we get
s
 sL  x   x (0)   2 L  y   L cos 2t   2
s 4
s
 sL  x   2 L  y   2  1..........(1)
s 4
2
 sL  y   y (0)   2 L  x   L sin 2t   2
s 4
2
2 L  x   sL  y   2 ........(2)
s 4
1  2  s   2  gives,
2
  s2  4 L  y  
s 4
2

BIET 61 DEPARTMENT OF MATHEMATICS


MA6251 MATHEMATICS - II

 2 
 y   L1  2
 s  4 
  sin 2t
dy
2 x  sin 2t 
dt
 sin 2t  2 cos 2t
1
 x  cos 2t  sin 2t
2

Transforms of derivatives and integrals of functions

t
sin t
Problem 1 Find the Laplace transform of e t  dt
0
t
Solution:
 t sin t  1  sin t 
L  dt   L  
0 t  s  t 

 sin t 
L   L  sin t  ds

 t  s

1
 ds
s
s 1
2

  tan 1 ( s ) 


  tan 1  s 
2
 co t 1  s 
 t sin t  1
L  dt   co t 1  s 
0 t  s
 t sin t   1 
L e  t  dt    co t 1  s  
 0 t  s  s  s 1
co t  s  1
1

 .
s 1

Problem 2 Find  te 2t sin 3t dt using Laplace transforms.
0

3
Solution: L  sin 3t  
s 9 2

d  1  6s
L t sin 3t     2  2
ds  s  9   s  9  2

BIET 62 DEPARTMENT OF MATHEMATICS


MA6251 MATHEMATICS - II

 e  t sin 3t dt  L t sin 3t   by definition 


 st

6s

s  9
2 2


6s
i.e.,  te  st sin 3t dt 
s  9
2 2
0


12
 te
2 t
Putting s  2 we get sin 3t dt 
0
169

t
sin 5t
Problem 3 Find the Laplace transform of t  te4t cos 3t dt  dt
0
t
Solution:
 t 4 t  d  4t
t

L t  e cos 3t dt    L   e cos 3t dt 
 0  ds  0 
d 1 
    L  e 4t cos 3t   
ds  s 
d 1 
    L  cos 3t   s s  4 
ds  s 
d 1  s  
   2  
ds  s  s  9  s  s  4 
d 1 s4 

ds  s ( s  4) 2  9 


d 1  s  4 
 
ds  s ( s  8s  25) 
2

d  s4 

ds  s  8s 2  25s 
 3

  s 3  8s 2  25s  1   s  4   3s 3  16 s  25  
  
   
2

 s 3
 8 s 2
 25 s 
 3 
 s  8s 2  25s  3s 3  16s 2  25s  12 s 2  64 s  100 

   
2

 s 3
 8 s 2
 25 s 
 
 2 s 3  20 s 2  64s  100 

  s 3  8s 2  25s 2 
 

BIET 63 DEPARTMENT OF MATHEMATICS


MA6251 MATHEMATICS - II

 3 
 s  10 s 2  32 s  50 
2
  s 3  8s 2  25s  2 
 

 sin 5t 
L   L  sin 5t  ds
 t  s

5
 ds
s
s  25
2


 1  s 
 5. tan 1   
 5  5  s

  s 
  tan 1   
  5  s
 s
  tan 1  
2 5
s
 co t 1  
5
 t
sin 5t   t 4 t   sin 5t 
 L t  e 4t cos 3t dt    L t  e cos 3t dt   L 
 0 t   0   t 
2  s 3  10 s 2  32 s  50  s
  cot 1  
s  8s  25s  5
3 2 2

Problem 4 Find L t 2e 2t cos 2t  .


Solution:
2
2 d
L t e cos 2t    1
 2 2t
 2
L e 2t cos 2 t 
ds
d  s 
2

 2  2  
ds  s  4  s  s  2 

d2 s  2 
  
  s  2   4  
2
ds 2
d2  s  2 
 2 2
ds  s  4s  8 

d   s  4s  8  1   s  2  2 s  4  
 2 

ds   s 2  4s  8 
2

 

BIET 64 DEPARTMENT OF MATHEMATICS


MA6251 MATHEMATICS - II

 
d  s 2  4s  8  2s 2  4 s  4s  8 

ds   s 2  4s  8  
2

 
 
d   s 2  4s 

ds   s 2  4 s  8  2 
 


s 2
 4s  8   2 s  4    s 2  4s  2  s 2  4 s  8   2 s  4 

s  4s  8
2 4


s 2
 4 s  8   2s  4    s 2  4s   2s  4 

s  4s  8
2 3

2s 3  8s 2  16 s  4 s 2  16s  32  4 s3  8s 2  16 s 2  32 s

s  4s  8 
2 3

2 s 3  12s 2  32
 .
s  4s  8
2 3

Problem 5 Verify the initial and final value theorems for the function
f  t   1  e t  sin t  cos t 
Solution: Given f  t   1  e t  sin t  cos t 
L  f  t   L 1  e  t sin t  e  t cos t
 L 1  L  e  t sin t   L  e t cos t 
1 1 s 1
  
s  s  1  1  s  12  1
2

1 s2
 
s  s  12  1
Initial value theorem:
t 0 f  t   s  sF  s 
Lt Lt

LHS tLt0 1  e  t  sin t  cos t    1  1  2


RHS  sLt sF  s 
1 s2 
 Lts s   
 s  s  1  1 
2

 s 2  2s 
 sLt 1  2 
 s  2s  2 

BIET 65 DEPARTMENT OF MATHEMATICS


MA6251 MATHEMATICS - II

 2s 2  4 s  2 
 Lts  2 
 s  2s  2 
 2  4 / s  2 / s2 
 s  
Lt
2 
2
 1 2 / s  2 / s 
LHS  RHS
Hence initial value theorem is verified.
Final value theorem:
t  f  t   s 0 sF  s 
Lt Lt

LHS tLt f  t 
tLt 1  e  t sin t  e t cos t   1  e 
 0
RHS  sLt0 sF  s 
1 1 s 1 
 Lts0 s    
 s  s  1  1  s  1  1 
2 2

 s 2  2s 
 sLt0 1   1
  s  1  1 
2

LHS  RHS
Hence final value theorem is verified.

  s 2  1 
Problem 6 Find L1 log  2   .
  s 
Solution:
1
L1  F  s     L1  F   s   .......(1)
t
 s 1
2
F  s   log  2 
 s 
log  s 2  1  log  s 2  
d
Fs 
ds
2s 2s
 2  2
s 1 s
 2s 2s 
L1  F   s    L1  2  2
 s 1 s 
 s 1
 2 L1  2  
 s 1 s 
 2  cos t  1
  s 2  1  1
L1 log  2     2  cos t  1
  s  t

BIET 66 DEPARTMENT OF MATHEMATICS


MA6251 MATHEMATICS - II

2 1  cos t 
 .
t
s3
Problem 7 Find the inverse Laplace transform of
 s  1  s 2  2s  3
s 3 A Bs  C
Solution:   2 ....(1)
 s  1  s  2s  3 s  1 s  2s  3
2

s  3  A  s 2  2 s  3    Bs  c  s  1
Put s  1
2=2A
A=1
Equating the coefficients of s 2
0 = A+B  B = -1
Put s = 0
3 = 3A+C
C=0
s3 1 s
1    2
 s  1  s  2s  3
2
s  1 s  2s  3
1 s
 
s  1  s  12  2
1 s 1 1
  
s  1  s  1  2  s  1 2  2
2

 s 3   s  1  1  
1  1  1
L1    L  L1
   L  
  s  1  s  2 s  3 
 s  1 
  s  1  2    s  1  2 
2 2 2

 s  t 1  1 
 e  t  e t L1  2 e L  2
 s  2   s  2 
2

 e  t  e  t cos 2t  e  t sin 2t
 e  t 1  cos 2t  sin 2t  .

 s 1 
Problem 8 Find L1  s log  
  2
 s 1 
Solution:
  s 1  
L1  s log    2  f t 
  s 1 
 s 1 
 L  f  t    s log  2
 s 1 
 s log  s  1  s log  s  1  2

BIET 67 DEPARTMENT OF MATHEMATICS


MA6251 MATHEMATICS - II

L t f  t   
d
 s log  s  1  s log  s  1  2 
ds 
 s s 
   log  s  1   log  s  1 
 s 1 s 1 
  s  1  s  s  1  s  s  1 
  log   
  s 1  s2 1 
 s 1   s  s  s  s 
2 2
 log   
 s 1   s2 1 
 s 1 2s
 log   2
 s 1  s 1
  s  1  1  s 
tf  t   L1  log    2L  2 
  s 1   s  1
  s  1 
 L1 log     2 cosh t....(1)
  s  1 
  s  1 
To find L1 log  
  s 1 
  s  1 
Let f  t   L1 log  
  s 1 
 s 1
L  f  t    log  
 s 1 
L t f  t    log  s  1  log  s  1 
d
ds
1 1 2
   2
s 1 s  1 s 1
 1 
 t f  t   2 L1  2   2sinh t
 s  1
2sinh t
f t   .....(2)
t
Using (2) in (1)
2sinh t
tf  t    2 cosh t
t
2sinh t 2 cosh t
f t   
t2 t
 sinh t  t cosh t 
 2  .

2
t

BIET 68 DEPARTMENT OF MATHEMATICS


MA6251 MATHEMATICS - II

Solutions of linear ODE of second order with constant coefficients


 
 s  1
Problem 9 Using convolution theorem find L
  s 2  a2 2 
 
Solution:
L1  F  s  G  s    L1  F  s   * L1 G  s  
 
L1 
s   L1  s  * L1  1 
  s2  a2  
2  s 2  a 2   s 2  a 2 
 
 s  1 1  a 
 L1  2 * L  2
 s  a  a  s  a 
2 2

1
 cos at* sin at
a
1
  cos at*sin at 
a
t
1
  cos a u sin a  t  u  du
a0
t
1
  sin  at  au  cos a u du
a0
1 sin  at  au  au   sin  at  au  au 
t
  du
a0 2
t
1
sin at  sin a  t  2u   du
2a 0 

cos a  t  2u  
t
1 
  sin at  u  
2a  2 a 0
1  cos at cos at 
  t sin at  
2a  2a 2a 
t sin at
 .
2a

1
Problem 10 Find the Laplace inverse of using convolution theorem.
 s  1  s 2  9 
Solution:
L1  F  s  .G  s    L1  F  s   * L1 G  s  
 1   1 1 
L1    L1
 . 
  s  1  s  9     s  1  s  9  
2 2

BIET 69 DEPARTMENT OF MATHEMATICS


MA6251 MATHEMATICS - II

 1  1  1 
 L1  *L  2 
  s  1    s  9  
1
 et * sin 3t
3
t
1
  e u sin 3  t  u   du
30
t
1 u
e sin  3t  3u  du
3 0

t
1 u
e  sin 3t cos 3u  cos 3t sin 3u  du
3 0

t t
1 1
 sin 3t  e u cos 3u du  cos 3t  e  u sin 3u du
3 0
3 0
t t
sin 3t  e  u  cos 3t  e u 
    cos 3u  3sin 3u       sin 3u  3cos 3u  
3  10 0 3  10 0
sin 3t  e  u 1 
    cos 3t  3sin 3t    1 
3  10 10 
sin 3t  e u 1 
    sin 3t  3cos 3t    3 
3  10 10 

 s2 
Problem 11 Find L1   using convolution theorem
  s  a  s  b  
2 2 2 2

Solution:
L1  F  s  .G  s    L1  F  s   * L1 G  s  
 s s   s  1  s 
L1  2 . 2 2
 L1  2 *L  2
s  a s b   s  a   s  b 
2 2 2

1  sin  a  b  u  bt  sin  a  b  u  bt  


t

    
2  ab a b  0

1  sin  at  bt  bt  sin  at  bt  bt  sin bt sin bt 


     
2 a b ab a b a b 
1  sin at sin at sin bt sin bt 
   
2  a  b a  b a  b a  b 
1  2a sin at 2b sin bt 
  2  2 2 
2  a  b2 a b 

BIET 70 DEPARTMENT OF MATHEMATICS


MA6251 MATHEMATICS - II

1  2a sin at  2b sin bt 

2  a2  b2 
a sin at  b sin bt
 .
a2  b2

Problem 12 Using convolution theorem find the inverse Laplace transform of


1
.
s  a2 
2 2

Solution:
L1  F  s  .G  s    L1  F  s   * L1 G  s  
 
L 
1 1   L1  1  * L1  1 
  s2  a2  
2  s 2  a 2   s 2  a 2 
 
sin at sin at
 *
a a
t
1
 2  sin au sin a  t  u  du
a 0
t
1
cos  2au  at   cos at  du  2sin A sin B  cos  A  B   cos  A  B  
2a 2 0 

1  sin  2au  at 
t

 2   cos at  u 
2a  2a 0
1  sin at   sin at  
 2  t cos at   
2a  2a  2a  
1  2sin at 
 2 
 t cos at 
2a  2a 
1
 3 sin at  at cos at 
2a

Problem 13 Solve the equation y  9 y  cos 2t; y  0   1and y  / 2   1


Solution:
Given y   9 y  cos 2t
L  y   t   9 y  t    L  cos 2t 
L  y   t    9 L  y  t    L  cos 2t 
s
 s 2 L  y  t    sy  0   y   0    9 L  y  t    2
s 4

BIET 71 DEPARTMENT OF MATHEMATICS


MA6251 MATHEMATICS - II

As y  0  is not given, it will be assumed as a constant, which will be evaluated at the


end.  y  0   A.
s
L  y  t    s 2  9   s  A 
s 42

s
L  y  t    s 2  9   s A
s 4 2

s s A
L  y  t    2  2  2 .
 s  4 s  9 s  9 s  9
2

s As  B Cs  D
Consider   2
s 2
 4  s  9 
2
s2  4 s 9
s   As  B   s 2  9    Cs  B   s 2  4 
 As 3  9 As  Bs 2  9 B  Cs 3  4  s  1 s 2  4
Equating coefficient of s 3 A + C = 0 …….(1)
2
Equating coefficient of s B + D = 0 …….(2)
Equating coefficient of s 9A+ 4C = 1 …….(3)
Equating coefficient of constant 9B +4D = 0 …….(4)
Solving (1) & (3)
4 A  4C  0
9 A  4C  1
5 A  1
1
A
5
1
C  0
5
1
C
5
Solving (2) & (4)
9B  9D  0
9 B  AD  0
D0
B  0& D  0 .
s 1 s s
 2  
 s  4  s  9 5 s  4 5  s  9
2 2 2

1 s s  s A
 L  y  t     2  2  2  2
5 s  4 s  9 s  9 s  4
1 1 A
 y  t   cos 2t  cos 3t  cos 3t  sin 3t
5 5 3
1 4 A
 cos 2t  cos 3t  sin 3t
5 5 3

BIET 72 DEPARTMENT OF MATHEMATICS


MA6251 MATHEMATICS - II

 
Given y    1
2
1 A
1   
5 5
12
A
5
1 4 4
 y  t   cos 2t  cos 3t  sin 3t
5 5 5

d 2 y 3dy
Problem 14 Using Laplace transform solve   2 y  4 given that
dx 2 dx
y  0   2, y  0   3
Solution:
L  y   t    3L  y  t    2 L  y  t    L  4 
4
s 2 L  y  t    sy  0   y  0   3sL  y  t    3 y  0   2 L  y  t   
5
s 2
 3s  2  L  y  t    2 s  3  6 
4
s
s 2
 3s  2  L  y  t   
4
s
 2s  3

2 s 2  3s  4
L  f  t   
s  s 2  3s  2 
2 s 2  3s  4
L  f  t   
s  s  1 s  2 
2 s 2  3s  4 A B C
  
s  s  1 s  2  s s  1 s  2
2s 2  3s  4  A  s  1 s  2   Bs  s  2   Cs  s  1
Put s = 0 4 = 2A => A = 2
s=1 3   B  B  3
s2 6  2c  C  3
2 3 3
 L  y  t     
s s 1 s  2
y  t   2  3e  3e 2t
t

dx dy
Problem 15 Solve  y  sin t; x  cos t with x  2 and y  0 when t  0
dy dt
Solution:
Given x  t   y  t   sin t

BIET 73 DEPARTMENT OF MATHEMATICS


MA6251 MATHEMATICS - II

x  t   y  t   cos t
L  x  t    L  y  t    L  sin t 
1
sL  x  t    x  0   L  y  t   
s 1
2

1
sL  x  t    L  y  t     2.........(1)
s 1
2

L  x  t    L  y  t    L  cos 2t 
1
L  x  t    sL  y  t     y  0   ....(2)
s 1
2

Solving (1) & (2)


1  s2
1  s  L  y  t   2  s 2  1
2

1  s 2  L  y  t   2s s22 11  s


2 2

2s 2  3
L  y  t    2
 s  11  s 2 
s2  3 As  B Cs  D
 2 
 s  11  s  s  1 1  s 2
2 2

s 2  3   As  B  1  s 2    Cs  D   s 2  1
Equating s 3 on both sides
0  A C put s  0
Ac 3 BD
A0 C 0
Equating s 2 on both sides
1  B  D D2
B 1
Equation son both sides 0 = A + B
 1   1 
 y  t   L1  2   2 L1  2 
 s  1  s  1
 sin t  2sinh t
To find x  t  we have x  t   y  t   cos t , x  t   cos t  y   t  , y  t   sin t  2sinh t
dy
 cos t  2 cosh t
dt
x  t   cos t  cos t  2 cosh t
 2 cosh t
Hence x  t   2 cosh t
y  t   sin t  2sinh t

BIET 74 DEPARTMENT OF MATHEMATICS


MA6251 MATHEMATICS - II

CHAPTER - 4

ANALYTIC FUNCTIONS

4.1 INTRODUCTION
In this Chapter we continue the study about the complex numbers. When x and y are real variables,z=x+iy is called
a complex variable. Let u ( x, y )andv ( x, y ) be two function of the variables x and y. Then
w = f ( z ) = u ( x, y ) + iv(x, y ) is a function of the complex variable z = x + iy .
If w gives a unique value corresponding to a value of z then w is called a single valued function of z and if it
gives more than one value corresponding to a given value of z then it is called a multiple valued function of z.

Definition
A single valued function f ( z ) is said to have a limit w o as z → z 0 if
(i) f ( z ) is defined in a neighborhood of z 0
(ii) and if for every ∈> 0, we can find a positive number δ such that f ( z ) − w0 <∈ ∀z given by 0

< z − z0 < δ .
We Write Lim f ( z ) = w0.
z → z0
Analytic Function:

Definition
A function f ( z ) is said to be continuous at z = z 0 if f ( z 0 ) is defined and Lim f ( z ) = f ( z 0 ).
z → z0

Definition
f ( z + ∆z ) − f ( z )
A function f ( z ) is said to be differentiable at a fixed point z if Lim exists.
∆z
∆z → z0
This limit is called the derivative of f ( z ) at the point z and it is denoted by f ( z ) .

Note
Here z + ∆z is a neighbourhood point of z with small distance ∆z , can be chosen in any side of z. This is
an important aspect in the above definition.

Definition
A function f ( z ) is said to be analytic at a point z = a in a region R if
(i) f ( z ) is differentiable at z = a
(ii) f ( z ) is differentiable at all points for some neighbourhood of z = a .

Definition
A function f ( z ) is said to be analytic in a region R if f ( z ) is analytic at all points in the region R.
Note
Instead of the term analytic in R, the terms holomorphic I R and regular in R are also used.

BIET 75 DEPARTMENT OF MATHEMATICS


MA6251 MATHEMATICS - II

4.2 CAUCHY-RIEMANN EQUATIONS


We shall now obtain the necessary conditions for a complex function f ( z ) = u ( x, y ) + iv ( x, y )
to be analytic.

Let f ( z ) = u + iv be analytic in a region R in z plane.


⇒ f ( z ) is differentiable in R.
f ( z + ∆z ) − f ( z )
⇒ f ' ( z ) = Lim exists in R.
∆z
∆z → 0 … (1)
Let z = x + iy
∴ z = ∆x + i∆y
∴ z + ∆z = ( x + iy ) + (∆x + i∆y )
= ( x + ∆x ) + i ( y + ∆y )
Now f ( z ) = u ( x, y ) + iv ( x, y )
∴ f ( z + ∆z ) = u ( x + ∆x, y + ∆y ) + iv( x + ∆x, y + ∆y ) …. (2)
Case (i): Choose z + ∆z along the horizontal line through z. Then ∆y = 0
∴ ∆z = ∆x

∴ ∆z → 0 ⇒ ∆x → 0
Now From (1) and (2),

f ' ( z ) = Lim
[u (x + ∆x, y + ∆y ) + iv(x + ∆x, y + ∆y )] − [u (x, y ) + iv(x, y )]
∆x + i∆y
∆z → 0

u (x + ∆x, y ) + iv(x + ∆x, y ) − u ( x, y ) − iv( x, y )


= Lim
∆x
∆x → 0

=Lim
[u(x + ∆x, y ) − u (x, y )] + i[v(x + ∆x, y ) − v(x, y )]
∆x
∆x → 0

u (x + ∆x, y ) − u (x, y ) v( x + ∆x, y ) − v( x, y )


= Lim +i Lim
∆x ∆x
∆x → 0 ∆x → 0

f ' (z ) =
du dv
+i …(3)s
dx dx
Case (ii)
Choose z + ∆z along the vertical line through z. Then ∆x = 0

BIET 76 DEPARTMENT OF MATHEMATICS


MA6251 MATHEMATICS - II

⇒ ∆z = i∆y

∴ ∆z → 0 ⇒ ∆y → 0

From (1) and (2),

f ' ( z ) = Lim
[u(x + ∆x, y + ∆y ) + iv(x + ∆x, y + ∆y )] − [u (x, y ) + iv(x, y )]
∆x + i∆y
∆z → 0

u (x, y + ∆y ) + iv( x, y + ∆y ) − u ( x, y ) − iv( x, y )


= Lim
i∆y
∆y → 0

=
1
Lim
[u(x, y + ∆y ) − u (x, y )] + i[v(x, y + ∆y ) − v(x, y )]
i ∆y
∆y → 0

1 u (x, y + ∆y ) − u ( x, y ) v( x, y + ∆y ) − v( x, y )
= Lim + Lim
i ∆y ∆y
∆y → 0 ∆y → 0
1 du dv
= +i
i dy dy

f ' (z ) = − i
du dv
+i … (4)
dy dy

From (3) and (4), we get


du dv du dv
+i =−i +i
dx dx dy dy
Equating the real and imaginary parts, we get
du dv du dv
= = =−
dx dy dy dx
The above equations are known as Cauchy-Riamann equations (or) C-R equations.

SUFFICIENT CONDITION FOR A FUNCTION f ( z ) TO BE ANALYTIC


Continuous single valued function f ( z ) = u + iv is analytic in a region R if
du du dv dv
(i) the four partial derivatives , , , exists and all are continuous
dx dy dx dy

BIET 77 DEPARTMENT OF MATHEMATICS


MA6251 MATHEMATICS - II

du dv du dv
(ii) = , =−
dx dy dy dx
Proof
By Taylor’s series
2
 d d  1 d d 
f ( x + h, y + k ) = f ( x, y ) +  h + k  f ( x, y ) +  h + k  f ( x, y ) + K … (1)
 dx dy  2!  dx dy 
Let z = x + iy ⇒ ∆z = ∆x + i∆y
z + ∆z = u ( x + iy ) + (∆x + i∆y ) = ( x + ∆x ) + i ( y + ∆y )
f ( z + ∆z ) = u ( x + ∆x, y + ∆y ) + iv( x + ∆x, y + ∆y )
2
 d  1 d 
= u ( x, y ) +  ( )  u ( x, y )
d d

 dxx + ∆y 
u x , y +  ∆
 dxx + ∆y
 dy  2!  dy 
  d  
+ K + i v( x, y ) +  ∆x + ∆y v( x, y ) + L{by (1)}
d
  dx dy  

du  dv 
= u ( x, y ) + ∆x + i v( x, y ) + ∆x + ∆y  + L
du dv
+ ∆y
dx dy  dx dy 
 du dv   du dv 
= u ( x, y ) + iv( x, y ) + ∆x + i  + ∆y + i 
 dx dx   dx dy 
[Omitting the higher powers of ∆x, ∆y ]
 du dv   dv du 
= f ( z ) + ∆x + i  + ∆y − + i  [by CR equations]
 dx dx   dx dx 
 du dv   dv du 
= f ( z ) + ∆x + i  + ∆y i 2 + i  [Since i 2 =-1]
 dx dx   dx dx 
 du dv   du dv 
= f ( z ) + ∆x + i  + i∆y +i 
 dx dx   dx dx 
 du dv 
= f (z ) +  + i (∆x + i∆y )
 dx dx 
 du dv 
f ( z + ∆z ) = f ( z ) +  + i ∆z
 dx dx 
 du dv 
⇒ f ( z + ∆z ) − f (z ) =  + i ∆z
 dx dx 
f ( z + ∆z ) − f ( z ) du dv
⇒ = +i
∆z dx dx
f ( z + ∆z ) − f (z ) du dv
⇒ Lim = Lim +i
∆z dx dx
∆z → 0 ∆z → 0

⇒ f ' (z ) =
du dv dv du
+i = −i (by C-R equations)
dx dx dy dy

BIET 78 DEPARTMENT OF MATHEMATICS


MA6251 MATHEMATICS - II

⇒ f ' (z ) exists and


du du dv dv
, , , are all continuous
dx dy dx dy

Example: 1 Test whether the function f ( z ) = z 3 + z is analytic or not.


Solution: Let f ( z ) = z 3 + z
⇒ u + iv = ( x + iy ) + (x + iy )
3

= x 3 + 3 x 2 (iy ) + 3x(iy ) + (iy )3 + x + iy


2

[since (a + b ) = a 3 + 3a 2 b + 3ab 2 + b 3 ]
3

= x 3 + i3 x 2 y − 3xy 2 − iy 3 + x + iy
[Since i 2 =-1, i 3 = −i ]
= (x 3 − 3xy 2 + x ) + i (3x 2 y − y 3 + y )
Equating the real and imaginary parts, we get

u = x 3 − 3xy 2 + x, v = 3x 2 y − y 3 + y

du dv
⇒ = 3x 2 − 3 y 2 + 1; = 6 xy − 0 + 0 = 6 xy
dx dx
= 0 − 3x(2 y ) + 0; = 3x 2 (1) − 3 y 2 + 1
du dv
dy dy

= −6 xy; = 3x 2 − 3 y 2 + 1
du dv du − dv
⇒ = ; =
dx dy dy dx
⇒ C-R equations are satisfied.
⇒ f ( z ) = z 3 + z is analytic.

Example: 2 Define an analytic function. Determine whether the function w = 2 xy + i x 2 − y 2 ( )


is analytic]

Solution: A function f ( z ) is said to be an analytic function at z = a in a region R if


(i) f ( z ) is differentiable at z = a
(ii) f ( z ) is differentiable at all points for some neighbourhood of z = a

A function f ( z ) is analytic in a reigion R if f ( z ) is analytic at all points in the region R.

Now w = 2 xy + i (x 2 − y 2 )
⇒ u + iv = 2 xy + i (x 2 − y 2 )

BIET 79 DEPARTMENT OF MATHEMATICS


MA6251 MATHEMATICS - II

Equating the real and imaginary parts, we get


u = 2 xy; v = x2 − y2
du dv
⇒ = 2 y; = 2x
dx dx
du dv
= 2x; = 2y
dy dx
du dv du − dv
⇒ ≠ ; ≠ ;
dx dy dy dx
⇒ C-R equations are not satisfied.
⇒ w is not analytic.

Example:3 Find the constants a,b and c if f ( z ) = x + ay + i (bx + cy ) is analytic

Solution: Let f ( z ) = x + ay + i (bx + cy )


⇒ u + iv = ( x + ay ) + i (bx + cy )

Equating the real and imaginary parts, we get

u = x + ay; v = bx + cy;

du dv
⇒ = 1; =b
dx dx

du dv
= a; =c
dy dy
Since f ( z ) is analytic, we have

du dv du dv
= ; =−
dx dy dy dx
⇒ c = 1 and a = −b

⇒ c = 1 and a = −b, b may be any value.

Example4: Verify whether f ( z ) = sinh z is analytic using C-R equations.

Solution: Let f ( z ) = sinh z


⇒ u + iv = sinh ( x + iy )

= sin i( x + iy )
1
[since sin iθ = i sinh θ ]
i

BIET 80 DEPARTMENT OF MATHEMATICS


MA6251 MATHEMATICS - II

= sin (ix + i 2 y )
1
i
= sin (ix − y )
1
[Since i 2 =-1]
i
= {sin ix cos y − cos ix sin y}
1
i
[since sin (a − b ) = sin a cos b − cos a sin b ]
= {i sinh x cos y − cosh x sin y}
1
i
[since sin iθ = i sinh θ , cos iθ = i cosh θ ]
1
= sinh x cos y − cosh x sin y
i
= sinh x cos y + i cosh x sin y [since 1 i = −i ]

Equating the real and imaginary parts, we get

u = sinh x cos y; v = sin y cos x


du dv
= cosh x cos y; = sin y sinh x
dx dx
du dv
= − sin y. sin x; = cos y cosh x
dy dy
du dv du dv
⇒ = ; =−
dx dy dy dx
⇒ C-R equations are satisfied.
⇒ f ( z ) = sinh z is analytic.

f ( z ) = (x − y ) + 2i( x + y )
2
Example:5 Find the analytic region of
f ( z ) = (x − y ) + 2i( x + y )
2
Solution: Let
u + iv = ( x − y ) + i 2( x + y )
2
(i.e.)
u = ( x − y ) and v = 2( x + y )
2

u x = 2( x − y ) vx = 2
u y = −2( x − y ) vy = 2
The C-R equations will satisfy only if x − y = 1 . Hence the function is analytic only on the line
x − y = 1.

Example6: verify if the function e −2 x cos 2 y can be real


/imaginary part of an analytic function

Solution: Let f = e −2 x cos 2 y

BIET 81 DEPARTMENT OF MATHEMATICS


MA6251 MATHEMATICS - II

f x = −2e −2 x cos 2 y
f xx = 4e −2 x cos 2 y
f y = −2e −2 x sin 2 y
f yy = −4e −2 x cos 2 y

f xx + f yy = 0 ∴ f is harmonic
In a simply connected domain, every harmonic function is the real part or the imaginary part
of some analytic function.
∴ Given is a real or imaginary part of an analytic function.

4.3 PROPERTIES OF ANALYTIC FUNCTIONS

Property: 1
Show that an analytic function with constant real part is constant. [Anna UQ]

Solution: Let f ( z ) = u + iv be analytic


du dv du dv
= ; =−
dx dy dy dx
Given that u =constant = c1 (say)

du du
⇒ = 0, =0
dx dy
dv dv
⇒ = 0, =0 [by (1) ]
dy dx
⇒ v is independent of x and y
⇒ v is constant
⇒ v = c 2 , c 2 is constant (say)
⇒ f ( z ) = u + iv = c1 + ic 2 is a constant.

Property: 2
Show that an analytic function with constant modulus is constant.

Solution: Let f ( z ) = u + iv be analytic

du dv du dv
= ; =−
dx dy dy dx
Given that f ( z ) = Constant.
⇒ u 2 + v 2 = c1 , c1 is a constant.
⇒ u 2 + v 2 = c 21
Differentiate partially with respect to x, y we get

BIET 82 DEPARTMENT OF MATHEMATICS


MA6251 MATHEMATICS - II

du dv du dv
2u + 2v = 0, 2u + 2v = 0,
dx dx dy dy
du dv du dv
⇒u +v = 0, u +v = 0,
dx dx dy dy

du dv dv du
⇒u +v = 0, −u +v = 0, [by (1)]
dx dx dx dx
du dv du dv
⇒u +v = 0, v −u = 0,
dx dx dx dx
= u 2 − v 2 = −(u 2 + v 2 )
u v
Now
v −u
= −c1 ≠ 0
2

∴ The above equations have trivial solution.


du dv
∴ = 0, =0
dx dx
dv du
⇒ = 0, =0 [by (1)]
dy dy
du du dv dv
Hence = = 0, = =0
dx dy dx dy
⇒ u, v is independent of x and y
⇒ u and v are constants
⇒ f ( z ) = u + iv is a constant.

Property: 3
Show that an analytic function with constant imaginary part is constant.

Solution: Let f ( z ) = u + iv be analytic


du dv du dv
⇒ = , =− … (1)
dx dy dy dx
Given that v = Constant = c1 (say )
dv dv
⇒ = 0, = 0,
dx dy
du du
⇒ = 0, = 0, [by (1)]
dy dx
⇒ u is independent of x and y
⇒ u = Constant = c 2 (say )
⇒ f ( z ) = u + iv = c 2 + ic 2 = Constant.

Property: 4
If f ( z ) is a regular function prove that

BIET 83 DEPARTMENT OF MATHEMATICS


MA6251 MATHEMATICS - II

 d2 d2 
f (z ) = 4 f ' (z )
2 2
 2 + 2 
 dx dy 
[MU April 2002, April 2000, AU Nov 2003, April 2004]
d 2
d2  d2
( ) f (z )
2 2
Solution:  2
+ 2 
f z = 4
 dx dy  dzd z
[By Property 6]

[ ]
2
f (z ). f ( z ) [Since zz = z ]
d 2
=4
dzd z

=4
d2
dzd z
[
f ( z ). f z ( )] [Since () ()
f z = f z ]

=4
d d 
[
  f ( z ). f z
dz  dz 
( )]
=4
d
dz
[
f ( z ). f ' z ( )]
()
= 4 f ' z . f ' (z )
= 4 f ' ( z ). f ' ( z ) = 4 f ' ( z )
2

Property: 5
If f ( z ) is an analytic function prove that
 d2 d2 
 2 + 2  log f ( z ) = 0 (Anna UQ)
 dx dy 
 d2 d2 
Solution:  2 + 2  log f ( z ) = 4
d2
dzdz
log f ( z ) f ( z ) 2
1
[ ]
 dx dy 

( )
[Since z = zz
1
2
]

=4
d2 1
dzdz 2
log f ( z ). f z [ ( )] [Since log a x = x log a ]

=2
d2
dzdz
[
log f ( z ) + log f z ( )] [Since log mn = log m + log n ]

=2
d d 
[
  log f ( z ) + log f z
dz  dz 
( )]
d  
f ' ( z ) = 2(0 ) = 0
1
= 2 0 +
dz  f z  ()

Property: 6
If f ( z ) is an analytic function, prove that

BIET 84 DEPARTMENT OF MATHEMATICS


MA6251 MATHEMATICS - II

 d2 d2 
 2 + 2  f ( z ) = p 2 f ( z ) f ' (z )
p p−2 2

 dx dy 
 d2 d2  d2
Solution:  2 + 2  f ( z ) = 4 f (z )
p p

 dx dy  dz dz

∂2
( )
p
=4  f ( z ). f ( z ) 12  [Since z = zz
1
2
]
∂ z∂ z  

=4
∂2
f ( z ). f z () p 2  (a m ) = a mn 
n

∂ z∂ z
 Since
 () 
f (z ) = f z 
∂ ∂ p 2
=4  ( f (z )) .( f ( z )) 
p 2

∂z  ∂z 

= 4 ( f ( z ))
∂z
p 2 p

2
f z
p 2 −1
( ( ))
f' z ()

( ( )) ()
p−2
d  
= 2p  f (z ) . f z
p 2 2 .f ' z 
dz  

() ( ) dd
p −2
= 2 pf z f (z )
p 2
2 .f ' z
z

() ( ) 2p ( f (z ))
p−2
f ' (z )
p
−1
= 2 pf z 2 .f ' z . 2

( ( )) ()
p−2 p−2
= p2 f z 2 .f ' z 2 f ' (z )

( () ) ()
p −2
= p 2 f z . f ' (z ) . f ' ( z ). f z [Since a m .b m = (ab ) ]
2 m

( f ' (z ) f ' (z ))
p −2
= p 2  f ( z ) f ' ( z ) 2 
1

 
= p 2 f (z ) . f ' (z ) .
p− 2 2

POLAR FORM OF CAUCHY RIEMANN EQUATIONS


Let z = re iθ and f ( z ) = p(r , θ ) + iQ (r , θ )
Then p (r , θ ) + iQ (r , θ ) = f re iθ ( )
Differentiating (1) partially w.r.t to r, we get
∂P ∂Q
∂r
+i
∂r
( )
= f ' re iθ e iθ
Differentiating (1) partially w.r. to θ , we get
∂P ∂Q
∂θ
+i
∂θ
( )
= f ' re iθ .r.e iθ

[ ( ) ]
= ri f ' re iθ e iθ

BIET 85 DEPARTMENT OF MATHEMATICS


MA6251 MATHEMATICS - II

 ∂p ∂Q 
= ri  + i
 ∂r ∂r 
∂p 2 ∂Q
= ir +i r
∂r ∂r
∂p ∂Q
= ir −r [Since i 2 =-1]
∂r ∂r
Equating the real and imaginary parts, we get
∂p ∂Q ∂Q ∂p
= −r ; =r
∂θ ∂r ∂θ ∂r
∂Q − 1 ∂p ∂p 1 ∂Q
⇒ = ; =
∂r r ∂θ ∂r r ∂θ
The above equations are called Cauchy Riemann equations in polar form.

Show that f ( z ) = z n is differentiable and hence find its derivative.

Solution: Let f ( z ) = z n
⇒ P + iQ = re iθ [ ] n

= r n e inθ = r n [cos nθ + i sin nθ ]


= r n cos nθ + ir n sin nθ
Equating the real and imaginary parts, we get
P = r n cos nθ ; Q = r n sin nθ
∂P ∂Q
= nr n −1 cos nθ ; = nr n −1 sin nθ
∂r ∂r
∂P ∂Q
= r n (− n sin n θ ); = r n .n cos nθ
∂θ ∂θ
∂P nr n cos nθ  ∂Q  1
⇒ = nr n −1 cos nθ = = 
∂r r  ∂r  r
∂Q nr n sin nθ  ∂Q  1
= nr n −1 sin nθ = = − 
∂r r  ∂r  r
⇒ C-R equations are satisfied
⇒ f ( z ) = z n is differentiable
 ∂P ∂Q 
Now f ' ( z ) = e −iθ  + i
 ∂r ∂r 
[ ]
= e −iθ nr n −1 cos nθ + inr n −1 sin nθ
=e − iθ
( − iθ
)
= e (cos nθ + i sin nθ )
− iθ
=e nr n −1 .e n −1
= e − iθ +inθ .nr n −1 [Since a m .a n = a m + n ]
= nr n −1e iθ (n −1)

BIET 86 DEPARTMENT OF MATHEMATICS


MA6251 MATHEMATICS - II

[ ]
= n re iθ
n −1
[Since (ab ) = a m b m ]
m

= nz n −1
⇒ f ' ( z ) = nz n −1
Example: 3 Show that r n = a sec nθ , r n = b cos ecnθ interested orthogonally where is an integer,
a and b are constants.

Solution: Given that


r n = a secnθ ; r n = b cos ecnθ
rn rn
⇒ = a; =b
sec nθ cos ecnθ
⇒ r n cos nθ = a; r n sin nθ = b,
Consider f ( z ) = r n cos nθ + ir n sin nθ
Then P = r n cos nθ ; Q = r n sin nθ
∂P ∂Q
= nr n −1 cos nθ ; = nr n −1 sin nθ
∂r ∂r
∂P ∂Q
= r n (− n sin n θ ); = r n .n cos nθ
∂θ dθ
∂P 1 ∂Q ∂Q − 1 ∂P
⇒ = ; = except at r = 0
∂r r ∂θ ∂r r ∂θ
⇒ f ( z ) is analytic except at r = 0
Also we know that if f ( z ) = u + iv is analytic then the family of curves u ( x, y ) = c1 , v( x, y ) = c 2
Insects orthogonally.
∴ r n cos nθ = a, r n sin nθ = b, Intersects orthogonally.
⇒ r n = a secnθ , r n = b cos ecnθ Intersects orthogonally.

4.5 HARMONIC FUNCTIONS

Definition
∂ 2φ ∂ 2φ
An expression of the form + = 0 is called the Laplace equation in two dimensions.
∂x 2 ∂y
Definition
Any function having continuous second order partial derivatives which satisfies the Laplace is
called harmonic function.
Definition
Any two harmonic functions u and v such that f ( z ) = u + iv is analytic are called conjugate harmonic
functions.

Note: If u and v are conjugate harmonic functions then u is conjugate harmonic to v is conjugate
harmonic to u .

BIET 87 DEPARTMENT OF MATHEMATICS


MA6251 MATHEMATICS - II

Property: 13
If f ( z ) = u + iv is an analytic function then u is a harmonic function.
Solution: Let f ( z ) = u + iv be analytic
du dv du − dv
⇒ = , =
dx dy dy dx
∂ v ∂ v ∂  ∂v  ∂  ∂v 
2 2
Now 2 + 2 =   +  
∂x ∂y ∂x  ∂x  ∂y  ∂y 
∂  − ∂u  ∂  ∂u 
=  +  
∂x  ∂y  ∂y  ∂x 
∂ 2u ∂ 2u
+ =0 =
∂x∂y ∂y∂x
∴ v is a harmonic function.

−y
Example: 1 prove that u = x 2 − y 2 , v = are harmonic
x − y2
2

but u + iv is not a regular function.


Solution: Let u = x 2 − y 2
∂u ∂u
⇒ = 2 x; = −2 y
∂x ∂y
∂ 2u ∂ 2u
= 2;
⇒ = −2
∂x 2 ∂y 2
∂ 2u ∂ 2u
∴ 2 + 2 = 2−2 = 0
∂x ∂y
∴ u is harmonic
−y
Let v = 2
x + y2
[(
∂v − x 2 + y 2 0 − y (2 x )
=
) ]
∂x x2 + y2
2
( )
− (− 2 xy ) 2 xy
= =
(x 2
) (x + y )
+y 2 2 2 2 2

∂ v (x
2
+ y ) (2 y ) − (2 xy )2(x + y )2 x
2 2 2 2 2
=
∂x 2 (x + y ) 2 2 4

=
(x + y )([ x + y )2 y − 8x y ]
2 2 2 2 2

(x + y ) 2 2 3

2 x 2 y + 2 y 3 − 8x 2 y
=
(x 2
+ y2 ) 3

BIET 88 DEPARTMENT OF MATHEMATICS


MA6251 MATHEMATICS - II

2 y 3 − 6x 2 y
=
(x 2
) + y2
3

∂v − [(x + y )(1) − y (2 y )]
2 2
=
∂y (x + y ) 2 2 2

− [x + y − 2 y ]
2 2 2
=
(x + y ) 2 2 2

− [x − y ] 2 2
=
(x + y )
2 2 2

y2 − x2
=
(x 2
) + y2
2

∂ v (x
2
+ y ) (2 y ) − ( y − x )2(x + y )(2 y )
2 2 2 2 2 2 2
=
∂x 2 (x + y ) 2 2 4

=
(x + y )([ x + y )(2 y ) − 4 y(y − x )]
2 2 2 2 2 2

(x + y ) 2 2 4

2 x 2 y + 2 y 3 − 4 y 3 + 4 xy 2
=
(x 2
+ y2 ) 3

6x 2 y − 2 y 3
=
(x + y )2 2 3

=
(2 y − 6 x y )
3 2

(x + y ) 2 2 3

∂ 2v ∂ 2v 2 y 3 − 6x 2 y 2 y 3 − 6x 2 y
+ = − =0
( )
∂x 2 ∂y 2 x2 + y2
3
(
x2 + y2
3
) ( )
∴ v is harmonic
∂u ∂u
But = 2x; = −2 y
∂x ∂y
∂v 2 xy ∂v y 2 − x2
= ; = ;
∂x (
x2 + y2
2
∂y x2 + y2
2
) ( )
∂u ∂v ∂u − ∂v
⇒ ≠ ; ≠
∂x ∂y ∂y ∂x
⇒ f ( z ) = u + iv is not analytic.

CONSTRUCTION OF CONJUGATE HARMONIC FUNCTIONS


Method 1
Suppose u is given
∂u ∂u
Then , are known.
∂x ∂y

BIET 89 DEPARTMENT OF MATHEMATICS


MA6251 MATHEMATICS - II

Now by total derivative,


∂v ∂v
∂v = .∂x + ∂y
∂x ∂y
− ∂u ∂u  + ∂u ∂v ∂u − ∂v 
= ∂x + .∂y Since = ; = 
∂y ∂x  ∂x ∂y ∂y ∂x 
Integrating on both sides. we get
 − ∂u ∂u 
∫ ∂v = ∫  ∂y ∂x + ∂x .∂y 
 − ∂u ∂u 
⇒v= ∫ 
 ∂y
∂x + .∂y 
∂x 
Method: 2
Suppose v is given
∂u ∂u
Than , are known
∂x ∂y
Now by total derivative,
∂u ∂u
∂u = .∂x + ∂y
∂x ∂y
∂v ∂v  ∂u ∂v ∂u − ∂v 
= .∂x − ∂y Since = ; = 
∂x ∂y  ∂x ∂y ∂y ∂x 
Integrating on both sides. we get
 ∂v ∂v 
∫ ∂u = ∫  ∂y ∂x + ∂x .∂y 
 ∂v ∂v 
⇒u= ∫  ∂x + .∂y 
 ∂y ∂x 

Example: 1 Find the real part of the analytic function whose imaginary part
[ ( )
is e − x (2 xy cos y ) + y 2 − x 2 sin y ]
[ (
Solution: Given that v = e − x 2 xy cos y + y 2 − x 2 sin y ) ]
∂v
= e − x [2 y cos y (1) + sin y (0 − 2 x )]
∂x
[ ( )
+ 2 xy cos y + y 2 − x 2 sin y − e − x ]( )
[
= e− x 2 y cos y − 2 x sin y − 2 cos y − y sin y + x sin y
2
] 2

= e− x [cos y(2 y − 2 xy ) + sin y(x − y − 2 x )]


2 2

∂v
∂y
= e−x [2 x( y(− sin y ) + cos y + (y − x )cos y + sin y(2 y − 0)]
2 2

= e−x [− 2 xy − sin y + 2 x cos y + (y − x )cos y + 2 y sin y]


2 2

= e−x [sin y(2 y − 2 xy ) + cos y(2 x + y − x )]


2 2

BIET 90 DEPARTMENT OF MATHEMATICS


MA6251 MATHEMATICS - II

∂v ∂v
Now real part u = ∫ dx − dy
∂y ∂x
[
= ∫ e − x sin y (2 y − 2 xy ) + cos y (2 x + y 2 − x 2 ) dx ]
[ (
− ∫ e − x (cos y (2 y − 2 xy ) + sin y x 2 − y 2 − 2 x dy)]
[
= sin y ∫ e − x (2 y − 2 xy )dx + cos y ∫ e − x (2 x + y 2 − x 2 ) dx ]
[
− e − x ∫ cos y (2 y − 2 xy )dy − e − x ∫ sin y (x 2 − y 2 − 2 x ) dy ]
[
= sin y (2 y − 2 xy )(e − x ) − (0 − 2 y )(e − x ) ]
[
+ cos y (2 x + y 2 − x 2 )(e − x ) − (2 + 0 + 2 x )(e − x ) + (− 2 )(− e − x ) ]
− e − x [(2 y − 2 xy )(sin y ) − (2 − 2 x )(− cos y )]
[
− e − x (x 2 − y 2 − 2 x )(cos y ) − (0 − 2 y − 0)(− sin y ) + (− 2)(cos y ) + C ]
[
= sin y − 2 ye − x + 2 xye − x + 2 ye − x ]
[
+ cos y − 2 xe − x − y 2 e − x + x 2 e − x − 2e − x + 2 xe − x + 2e − x ]
e − x [2 y sin y − 2 xy sin y + 2 cos y − 2 x cos y ]
[
− e − x − x 2 cos y + y 2 cos y + 2 x cos y − 2 y sin y − 2 cos y + C ]
= sin y (2 xye − x ) + cos y (x 2 − y 2 )e − x − 2 ye − x sin y
+ 2 xye − x sin y − 2e − x cos y + 2 xe − x cos y
+ (x 2 − y 2 )(cos y )e − x − 2 x cos ye − x + 2 ye − x sin y + 2 xe − x cos y + C
= 2 xye − x sin y + (x 2 − y 2 )e − x cos y + C
[
⇒ u (x, y ) = e − x 2 xy sin y + (x 2 − y 2 )cos y + C ]
log(x 2 + y 2 ) is harmonic and find its conjugate
1
Example: 6 show that the function u =
2
log(x 2 + y 2 )
1
Solution: Let u =
2
∂u 1  1 
(2 x ) = 2
x
=  2 2 
∂x 2  x + y  x + y2
∂ 2 u 1 (x 2 + y 2 )(1) − x(2 x − 0 )
=
∂x 2 2 (x 2 + y 2 )2
x 2 + y 2 − 2x 2 y2 − x2
= =
(x 2
+ y2 )
2
(x 2
+ y2 )
2

∂u 1  1 
(2 y ) = 2
y
= 
∂y 2  x 2 + y 2  x + y2
∂ 2 u 1 (x 2 + y 2 )1 − y (2 y )
=
∂y 2 2 (x 2 + y 2 )2

BIET 91 DEPARTMENT OF MATHEMATICS


MA6251 MATHEMATICS - II

=
x2 + y2 − 2y2
=
x2 − y2
=
(
− y2 − x2 )
(x 2
+y )
2 2
(x + y ) (x 2
+y )
2 2 2 2 2

Now =
∂ u ∂ u
+
2
=
y −x 2

(y − x ) = 0 2 2 2 2

∂x ∂y (x + y ) (x + y )
2 2 2 2 2 2 2 2

∴ u is harmonic.
∂u ∂u
Now v = ∫ dx + dy
∂y dx
−y x
=∫ 2 dx + 2 dy
x +y 2
x + y2
xdy − ydx xdy − ydx
=∫ 2 =∫ 2
x +y 2
x 1 + ( y / x)
2
( )
d ( y / x)
=∫
1 + ( y / x)
2

v = tan −1 ( y / x ) + C

4.6 CONFORMAL TRANSFORMATION

Definition:
Conformal mapping
A mapping or transformation which preserves angles in magnitude and in
direction between every pair of curves through a point is said to be conformal at that point.

Isogonal Transformation
The transformation preserves the angle in magnitude but not in direction
between every pair of curves through a point is said to be a isogonal at that point.

Critical Point:
A point at which f ' ( z ) = 0 is called a critical point of the transformation.

ie) At the critical point, the transformation w = f (z ) is not conformal.

Example: Find the critical points for the transformation w 2 = ( z − α )( z − β )


Solution: w 2 = ( z − α )( z − β )

= ( z − α ) + ( z − β ) = 2 z − (α + β )
dw
2w
dz
= z − (α + β )
dw 1
∴w
dz 2

BIET 92 DEPARTMENT OF MATHEMATICS


MA6251 MATHEMATICS - II

dw
Critical points occur at =0
dz
1
∴ z − (α + β ) = 0
2
dw w
Also =
z − (α + β )
dz 1
2
dw
The critical points occurs at =0
dz

w
∴ =0
z − (α + β )
1
2
⇒w=0
⇒ ( z − α )( z − β ) = 0
⇒ z = α, z = β
1
∴ The critical points occur at ∴ z = (α + β ), α & β
2

Simple Translations
(i)Translation:
The transformation w = z + a where a is a complex constant,represents a translation.
Let z = x + iy, w = u + iv and a = a1 + ia 2 ,
Then u + iv = ( x + iy ) + (a1 + ia 2 )
= (x + a1 ) + i ( y + a 2 )
∴ u = x + a1 and v = y + a 2 are the equations of transformation,
The image of the point (x,y) in the z-plane is the point (x + a, y + b ) in the w-plane.
∴ The transformation w = z + a translates every point (x , y) through a constant vector
representing ‘a’ .
Every point in any region of the z-plane is mapped upon the w-plane in the same manner.
If the whole w-plane is superposed on the z-plane the figure is shifted through a distance
given by the vector ‘a’.
In particular this transformation maps circles into circles. Also the corresponding region in
the z-plane and w-plane will have the same shape, size and orientation.

(ii) Magnification:
The transformation w =az, where a is a complete constant, represents
magnification.
Now w = az
⇒ u + iv = a( x + iy )
⇒ u = ax, v = ay are the equators of transformation.

BIET 93 DEPARTMENT OF MATHEMATICS


MA6251 MATHEMATICS - II

It is clear that the image of any figure in the z-plane is magnified ‘a’ times. Without any
changes in its shape and orientation. The transformation maps circles into circles.

(iii) Magnification and Rotation:


If we consider ‘a’ is a complex number it represents both magnification and
rotation.
Let z = re iθ , α = αe iβ and w = Re iφ

Now w = az
⇒ Re iφ = αe iβ re iθ
⇒ R = rα , φ = β + θ
The image θ of any point p in z-plane is obtained from p by rotating op through an angle
α = arg ’a’ and magnifying op in the ration a .
The transformation w = az corresponds to a rotation together with a magnification.

Example 1: Find the image of the circle z =c by the transformation w = 5z


Solution: Given w = 5z
∴ u + iv = 5( x + iy )
= 5x + i5y
⇒ u = 5 x, v = 5 y
Given z =C
1
⇒ (x 2 + y ) =C
2 2

⇒ x2 + y = C2
2

2
u v 2
⇒  +  = C2
 5 5
⇒ u 2 + v 2 = 25C 2

BIET 94 DEPARTMENT OF MATHEMATICS


MA6251 MATHEMATICS - II

⇒ u 2 + v 2 = (5C )
2

∴ z = C Maps to a circle in w-plane with centre at the origin and radius 3C.

1
The Transformation w =
z
1
The transformation w = is conformal at all points of the z-plane except at z = 0.
z
Put z = x + iy and w = u + iv then
1
u + iv =
x + iy
1 u − iv
⇒ x + iy = = 2
u + iv u + v 2
u v
⇒ x + iy = 2 −i 2
u +v 2
u + v2
Equating real and imaginary parts,
u v
x= 2 &y=− 2
u +v 2
u + v2
1
Also x 2 + y 2 = 2
u + v2
Case (i)
The line x = 0 (ie) the imaginary axis in the z-plane.
When x = 0, u = 0 which is the imaginary axis in the w-plane.
∴ The imaginary axis in the z-plane is mapped to the imaginary axis in the w-plane.

Case (ii)
The line y = 0
When y = 0, v = 0
∴ The real axis in the z-plane is mapped to the real axis in the w-plane.

Case (iii)
The equation of any line parallel to x- axis in the z-plane (i.e.,) y = k
v
y=− 2
u + v2
v
∴k =− 2
u + v2
v
⇒ u2 + v2 + = 0
k

BIET 95 DEPARTMENT OF MATHEMATICS


MA6251 MATHEMATICS - II

2 2
 1   1 
⇒ u + v +
2
 = 
 2k   2k 
 −1
This is a circle in the w-plane with centre at  0,  and passes through the origin.
 2k 
∴ The line parallel to real axis in the z-plane map into a family of circle in the w-plane passing
through the origin and having centre on v-axis.

Case (IV)
The equation of a line parallel to y-axis in the z-plane (ie) x = c
u
Given x = 2
u + v2
u
c= 2
u + v2
u
⇒ u2 + v2 + = 0
c
2 2
 1   1 
⇒ u −  + v2 =  
 2c   2c 
 1  1
This is a circle with centre at  ,0  and radius is
 2c  2c

Hence the line parallel to y-axis in the z-plane map onto the family of circles in the w-plane
passing through the origin and having centre on axis.

Case (v)
Consider the line y-axis in the z-plane
u x
Now =
v y

u 1 x 1
Put y = mx ⇒ = − Q = 
v m y m
∴ V = − mu , which is a straight line in the w-plane passing through the origin.
Hence the line y = mx passes through the origin maps into the straight line v = - mu in the w-
plane.

Case (vi)
The equation of a circle centre at the origin x 2 + y 2 = r 2
1
We know that x 2 + y 2 = 2
u + v2
1
∴ u 2 + v 2 = 2 Which is a circle at the origin in the w-plane.
r

BIET 96 DEPARTMENT OF MATHEMATICS


MA6251 MATHEMATICS - II

1
∴ The circle x 2 + y 2 = r 2 maps into the circle u 2 + v 2 =
r2
∴ The circle x + y > 1 in the z-plane maps into the region x 2 + y 2 < 1 (ie) the exterior of unit
2 2

circle z = 1 maps into the interior of the unit circle w = 1 .Also the interior of the unit circle
z = 1 maps into the exterior.

1
Example 2: Find the image of z − 2i = 2 under the transformation w =
z
Solution: Given z − 2i = 2
x + iy − 2i = 2

x + i ( y − 2) = 2
x 2 + ( y − 2) 2 = 2
x 2 + ( y − 2) = 4
2

x2 + y2 + 4 − 4y = 4
x 2 + y 2 − 4 y = 0 ………….(1)
1
The given transformation is w =
z
u v
⇒x= 2 ,y =− 2
u +v 2
u + v2
Substitute the values of x and y in (1)

 u   −v   −v 
2 2

 2 2 
+ 2 2 
− 4 2 2 
=0
u +v  u +v  u +v 
⇒ u 2 + v 2 + 4v(u 2 + v 2 ) = 0
⇒ (u 2 + v 2 )(1 + 4v) = 0
⇒ (1 + 4v) = 0 which is the straight line equation in the w-plane.

Example 3: Find the image of the circle z − 1 = 1 in the complex plane under the mapping
1
w=
z
Solution: Given z − 1 = 1
x + iy − 1 = 1
(x − 1)2 + y 2 =1
⇒ x − 2x + 1 + y 2 = 1
2

⇒ x 2 − 2 x + y 2 = 0 …………(1)

BIET 97 DEPARTMENT OF MATHEMATICS


MA6251 MATHEMATICS - II

1
Given transformation is w =
z
u v
⇒x= ,y =− 2
u +v 2 2
u + v2
Substitute in (1)
 u   −v 
2 2
 u 
 2 2 
+ 2 2 
− 2 2 =0
u +v  u +v  u +v
2

( )
⇒ u 2 + v 2 − 2u u 2 + v 2 = 0
⇒ (u 2 + v 2 )(1 − 2u ) = 0
⇒ (1 − 2u ) = 0
Which is the straight line equation in the w-plane.

BILINEAR TRANSFORMATION

Definition: Bilinear transformation (or) Mobius transformation (or) linear


fractional
az + b
The transformation w = , ad − bc ≠ 0 where a, b, c, d are complex
cz + d
constants is called the bilinear transformation.

Fixed Point (or) Invariant Point:

A fixed of a mapping w = f (z ) is a point z whose image is the same point.


The fixed point or invariant points of the transformation w = f (z ) are obtained by
solving z = f (z ) .
az + b az + b
The fixed point of w = are obtained by z = ⇒ cz 2 − (a − d )z − b = 0
cz + d cz + d
This is a quadratic equation in z, giving two values of z unless a = d and b = c =0.
Hence, a bilinear transformation has (at most) two fixed points.

Cross Ratio:
(z1 − z 2 )(z 3 − z 4 )
If z1 , z 2 , z 3, z 4 are four complex numbers, then is called the cross
(z1 − z 4 )(z 3 − z 2 )
ratio of four points z1 , z 2 , z 3, z 4 .

Properties of Bilinear transformation:

BIET 98 DEPARTMENT OF MATHEMATICS


MA6251 MATHEMATICS - II

1. The bilinear transformation always transforms circles into circles with lines as limiting
cases.
2. The bilinear transformation preserves cross ratio of four points.

2z − 5
Example: 1 Find the fixed points for the transformation w =
z+4
Solution: Fixed points are obtained from
2z − 5
z=
z+4
⇒ z + 4z − 2z + 5 = 0
2

⇒ z 2 + 2z + 5 = 0
2 ± 4i
∴z = −
2

Example: 2. find the bilinear transformation mapping the points z = 1, i,−1 into the points
w = 2, i,−2 respectively.
Solution: The bilinear transformation is given by
(w − w1 )(w2 − w3 ) (z − z1 )(z 2 − z 3 )
=
(w − w3 )(w2 − w1 ) (z − z 3 )(z 2 − z1 )
(w − 2)(i + 2) = (z − 1)(i + 1)
(w + 2)(i − 2) (z + 1)(i − 1)
(w − 2) = (z − 1)(i + 1)(i − 2)
(w + 2) (z + 1)(i − 1)(i + 2)
=
(z − 1)(− 3 − i )
(z + 1)(i − 3)
=
(z − 1)(3 + i )
(z + 1)(3 − i )
w − 2 + w + 2 (z − 1)(3 + i ) + ( z + 1)(3 − i )
=
w − 2 − w − 2 ( z − 1)(3 + i ) − ( z + 1)(3 − i )
2w z (3 + i + 3 − i ) + 3 − i − 3 − i
=
− 4 z (3 + i − 3 − i ) − 3 − i − 3 + i
w 6 z − 2i
=
− 2 2iz − 6
− 6 z + 2i
w=
iz − 3

Example: 3 Find the mobius transformation that maps the points z = 0,1, ∞ into the
points z = −5,−1,3 respectively. What are the invariant points of this transformation?
Solution: The bilinear transformation is given by

BIET 99 DEPARTMENT OF MATHEMATICS


MA6251 MATHEMATICS - II

(w − w1 )(w2 − w3 ) (z − z1 )(z 2 − z 3 )
=
(w − w3 )(w2 − w1 ) (z − z 3 )(z 2 − z1 )
Since z 3 = ∞ , we simplify the transformation
(w − w1 )(w2 − w3 ) (z − z1 )
=
(w − w3 )(w2 − w1 ) (z 2 − z1 )

(w + 5)(− 4) = (z − 0)
(w − 3)(4) (1 − 0)


(w + 5 ) = z
(w − 3)
⇒ w + 5 = 3 z − wz
⇒ w(1 + z ) = 3z − 5

3z − 5
⇒ w= is the required transformation.
1+ z
To get the invariant points, put w = z
3z − 5
∴z =
z +1
⇒ z − 2z + 5 = 0
2

2 ± 4 − 20
z=
2
2 ± 4i
=
2
= 1 ± 2i
∴ The invariant points are z = 1 ± 2i

Example: 4. Find the bilinear transformation that maps the points z = 0,−1, i into the points
z = 1,0, ∞ respectively.
Solution: The bilinear transformation is given by
(w − w1 )(w2 − w3 ) (z − z1 )(z 2 − z 3 )
=
(w − w3 )(w2 − w1 ) (z − z 3 )(z 2 − z1 )
Since w3 = ∞ , we simply the transformation.
(w − w1 ) (z − z1 )(z 2 − z 3 )
=
(w2 − w1 ) (z − z 3 )(z 2 − z1 )
(w − 1) = (z − 0)(− 1 − i )
(0 − 1) (z − i )(− 1 − 0)
− z (1 + i )
w −1 = is the required transformation
( z − i)

BIET 100 DEPARTMENT OF MATHEMATICS


MA6251 MATHEMATICS - II

CHAPTER - 5
COMPLEX INTEGRATION

5.1 CAUCHY’S INTEGRAL THEOREM


Statement:
If f(z) is analytic and f’(z) is continuous at all points inside and
on a simple closed curve c, then ∫ f ( z )dz = 0 .
c

Proof: Let R be the region by c,


z = x + iy, dz = dx + idy, f ( z ) = u + iv
Now ∫ f ( z )dz = ∫ (u + iv)(dx + idy)
c c

= ∫ (udx − vdy) + i ∫ (vdx + udy )...........(1)

∂u ∂u ∂v ∂v
Since f’(z) is continuous, the four partial derivatives , , ,
∂x ∂y ∂x ∂y
are also exists and continuous in R and on c
By Green’s theorem in the plane
 ∂N ∂M 
∫c Mdx + Ndy = ∫∫R  ∂x − ∂y dxdy
∴ (1) Becomes,
 ∂v ∂u   ∂u ∂v 
∫c f ( z )dz = ∫∫R  − ∂x − ∂y dxdy + i ∫∫R  ∂x − ∂y dxdy ……(2)
Since f(z) = u+iv is analytic by the CR – Equation,

∴ From (2)

 ∂u ∂u   ∂u ∂u 

c
f ( z )dz = ∫∫  − dxdy + i ∫∫  − dxdy
R 
∂y ∂y  R 
∂x ∂x 
= 0 + i0 =0

CAUCHY’S INTEGRAL FORMULA (OR) CAUCHY’S


FUNDAMENTAL FORMULA
Statement:
If f(z) is analytic inside and on a simple closed curve c and if ‘a’ is
any point within c, then,
1 f ( z)
f(a) = ∫
2πi c z − a
dz , the integration around c being taken in the

positive direction.
Proof:

BIET 101 DEPARTMENT OF MATHEMATICS


MA6251 MATHEMATICS - II

f ( z)
Given f(z) is analytic inside and on c. Now is analytic inside
z−a
and on c except at z =a
Draw a circle c1 : z − a = r with center at z = a and radius r units such
that c1 lies entirely inside c.

f ( z)
Now φ ( z ) = is analytic in the region enclosed between c and c1
z−a
∴ ∫ φ ( z )dz = ∫ φ ( z )dz
c c1

f ( z) f ( z)
(ie) ∴ ∫ dz = ∫ dz
c
z−a c1
z−a
On c1 , any point z is given by z = a + re iθ , 0 ≤ θ ≤ 2π ∴ dz = ire iθ dθ


f ( z) f (a + re iθ ) iθ
∴∫ dz = ∫ iθ
ire dθ
c
z−a 0 re
= 2πif (a)

1 f ( z)
∴ f (a) = ∫
2πi c z − a
dz

CAUCHY’S INTEGRAL FORMULA FOR DERIVATIVES OF AN


ANALYTIC FUNCTION:
Statement:
If f(z) is analytic inside and on a simple closed curve c and z = a is any
interior point of the region R enclosed by c, then
1 f ( z)
f ' (a) = ∫
2πi c ( z − a ) 2
dz

2! f ( z)
f ' ' (a) = ∫
2πi c ( z − a ) 3
dz

In general,

BIET 102 DEPARTMENT OF MATHEMATICS


MA6251 MATHEMATICS - II

n! f ( z)
f n (a) = ∫
2πi c ( z − a ) n +1
dz

Problems:
z2 +1
1. Evaluate ∫c z 2 − 1 dz where c is circle
(i) z − 1 = 1 (ii ) z + 1 = 1 (iii ) z − i = 1

Solution:
z2 +1 z2 +1
Given ∫ 2 dz = ∫c ( z + 1)( z − 1) dz
c z −1

z2 +1 A B
Consider = +
( z + 1)( z − 1) z + 1 z − 1
z 2 + 1 = A( z − 1) + B( z + 1)
Put z = 1,B = 1, put z =-1 , A = -1

z2 +1 1 1
∴∫ dz = − ∫ dz + ∫ dz
c
( z + 1)( z − 1) z + 1 z − 1
= 2πi[− f (a ) + f (a)]
Here the point are z = -1 and z = 1
(i) c is the circle z − 1 = 1 . The point z = -1 lies outside and z = 1 lies inside the
circle z − 1 = 1
z2 +1
∴∫ dz = 2πi[0 + f (a)]
c
( z + 1)( z − 1)

= 2πi [since f(z) = 1, f(-1) = 1]


(ii) c is the circle z + 1 = 1 . The point z = -1 lies inside and z = 1 lies outside the
circle z + 1 = 1

z2 +1
∴∫ dz = 2πi[− f (a) + 0]
c
( z + 1)( z − 1)
= − 2πi [ since f(z) = 1]
(iii) c is the circle z − i = 1
When Z = 1, 1 − i = 2 > 1 lies outside c
When z = -1, − 1 − i = 2 > 1 lies outside c
∴ ∫ f ( z )dz = 0
c

BIET 103 DEPARTMENT OF MATHEMATICS


MA6251 MATHEMATICS - II

z+4
2. Evaluate ∫z c
2
+ 2z + 5
dz where c is the circle

(i) z + 1 + i = 2 (ii) z + 1 − i = 2 (iii) z = 1


Solution:
z+4
Consider
c
∫z
+ 2z + 5
dz2

The singular points of f(z) are given by


− 2 ± 4 − 20
Z= = −1 ± 2i
2
Z = − 1 + 2i, z = −1 − 2i

z+4 z+4
∴∫ dz = ∫ dz
c z + 2z + 5
2
c
( z − (− 1 + 2i )(− 1 − 2i ))
(i) z + 1 + i = 2 is the circle
When z = -1+ 2i, − 1 + 2i + 1 + i = 3i > 2 lies outside c
z = −1 − 2i, − 1 − 2i + 1 + i = − i = 1 < 2 lies inside c
∴By Cauchy’s Integral formula

z+4
∫ z − (−1 + 2i) / z − (−1 − 2i) dz = 2πif (a)
c

 3 − 2i  π
= 2πi  = [2i − 3]
 − 4i  2
z+4
Where f ( z ) =
z − (−1 + 2i )
− 1 − 2i + 4 3 − 2i
f (−1 − 2i ) = =
− 1 − 2i + 1 − 2i − 4i
(ii) z + 1 − i = 2 is the circle
When z = −1 + 2i, − 1 + 2i + 1 − i = i = 1 < 2 lies inside c
When z = −1 − 2i, − 1 − 2i + 1 − i = − 3i > 2 lies outside c

∴ By Cauchy’s Integral formula

z+4
∫ z − (−1 + 2i) / z − (−1 − 2i) dz = 2πif (a)
c

BIET 104 DEPARTMENT OF MATHEMATICS


MA6251 MATHEMATICS - II

 3 + 2i  π
= 2πi  = [3 + 2i ]
 4i  2

z+4
Where f ( z ) =
z − (−1 − 2i )
− 1 + 2i + 4 3 + 2i
f (−1 − 2i ) = =
− 1 + 2i + 1 + 2i 4i
(iii) z = 1 is the circle
When z = −1 + 2i, − 1 + 2i > 1 lies outside c
When z = −1 − 2i, − 1 − 2i < 1 lies outside c

zdz
3. Using Cauchy’s Integral formula, evaluate ∫ (z − 1)(z − 2)
c
2
where c is the circle

1
z−2 =
2
z
Solution: Consider
(z − 1)(z − 2)2
1
Hence the point z – 1 lies outside circle z − 2 = and the point z = 2 lies outside
2
1
the circle z − 2 = .
2
zdz f ( z) z
∴∫ =∫ dz where f(z) =
c ( z − 1)( z − 2 )
2
c ( z − 2)
2
z −1
= 2πif ' (a)
= 2πi (− 1)
= − 2πi
Using Cauchy’s integral formula
[Here a = 2
z
F(z) =
z −1
f ' ( z) =
(z − 1)(1) − z (1)
(z − 1)2
f ' ( z ) = −1 ]

5.2 TAYLOR AND LAURENT SERIES


Taylor’s Series
Statement: If f (z) is analytic inside a circle C with its centre at z = a then for all z
inside C,
f ' (a )
f ( z ) = f (a) + (z − a )
f ' ' (a)
+ ( z − a) 2 + ...............
1! 2!

BIET 105 DEPARTMENT OF MATHEMATICS


MA6251 MATHEMATICS - II

This is known as Taylors series of f(z) about z = a .This expression is valid at


all points interior to any circle. Having its centre at z = a and within which the
function is analytic.
The circle of convergence of the Taylor series is the largest circle that can be
drawn around z = a such that within which f(z) is analytic. The radius of this circle is
called the redius of convergence.

Maclaurin’s Series :
Taking a = 0 in the Taylor series for f(z), then we have
f ' (0) f ' ' (0) f ' ' ' (0)
f ( z ) = f (0) + + + + ..............
1! 2! 3!
The series is called Maclaurin’s series of f(z).

Laurent’s Series:
If C1 and C 2 are two concentric circles with centre at z = a and radii
r1 and r2 (r1 < r2 ) and if f(z) is analytic inside and on the circles and within the
annulus between C1 and C 2 , then for any in the annulus , we have ,

f ( z ) = ∑ a n ( z − a ) + ∑ bn (z − a )
n −n

n =0

1 f ( z )dz 1 f ( z )dz
Where a n = ∫
2πi C ( z − a ) n +1
and bn = ∫
2πi C ( z − a )− n +1
Where C is any circle lying between C1 and C 2 with centre at z = a for all n
and the integration being taken in positive direction.

Some Important Results: If z < 1, we have



• (1 + z )−1 = 1 − z + z 2 − z 3 +............. = ∑ (− 1)n z n
n =0

• (1 − z )−1 = 1 + z + z 2 + z 3 + ............. = ∑ z n
n =0

• (1 + z )−2 = 1 − 2 z + 3z 2 − 4 z 3 +............. = ∑ (− 1)n (n + 1) z n
n =0

• (1 − z )−2 = 1 + 2 z + 3z 2 + 4 z 3 +............. = ∑ (n + 1) z n
n =0

1
1. Expand at x = 1 in Taylor series
z−2
1
Solution: Given f (z) = f(1) = -1
z−2
−1
f ' ( z) = , f ' (1) = −1
( z − 2) 2

BIET 106 DEPARTMENT OF MATHEMATICS


MA6251 MATHEMATICS - II

2
f ' ' ( z) = , f ' ' (1) = −2
( z − 2) 3

−6
f ' ' ' ( z) = , f ' ' ' (1) = −6
( z − 2) 4
∴ The Taylor series at z = 1 is given by

f ' (a )
f ( z ) = f (a) + (z − a )
f ' ' (a)
+ ( z − a) 2 + ...............
1! 2!
( z − 1) 2 ( z − 1) 3
= − 1 + ( z − 1)(−1) + (−2) + (−6) + ........
2! 3!
[ ]
= − 1 + ( z − 1) + ( z − 1) 2 + ( z − 1) 3 + ..................

z2 −1
2. Expand f ( z ) = as a Laurent’s series if (i) z < 2 (ii) z > 3
(z + 2)(z + 3)
(iii) 2 < z < 3
Solution:
z2 −1
Let f ( z ) =
(z + 2)(z + 3)
− 5z − 7
=1 +
(z + 2)(z + 3)
− 5z − 7 A B
Now = +
(z + 2)(z + 3) z + 2 z + 3
− 5 z − 7 = A( z + 3) + B( z + 2)
Put z = -3 ⇒ 15 − 7 = − B ⇒ − B = 8 ⇒ B = −8
Put z = −2 ⇒ 10 − 7 = A ⇒ A = 3
3 8
∴ f ( z) = 1 + −
z+2 z+3

z z
(i) z < 2 ⇒ < 1 also <1
2 3
3 8
∴ f ( z) = 1 + −
 z  z
21 +  31 + 
 2  3
−1 −1
3 z 8 z
= 1 + 1 +  − 1 + 
2 2 3 3

BIET 107 DEPARTMENT OF MATHEMATICS


MA6251 MATHEMATICS - II

3  8  z  z 2 
2
z z
= 1 + 1 − +   − ......  − 1 − +   − ....... 
2  2  2   3 3  3
 

3 2
(ii) z > 3 ⇒ 3 < z ⇒ < 1,2 < 3 < z , < 1
z z

3 8
f ( z) = 1 + −
z +2 z +3
3 8
= 1+ −
 2  3
z 1 +  z 1 + 
 z  z

−1 −1
3 2 8 3
= 1 + 1 +  − 1 + 
z z z z
3  2  2   8  3  3 2 
2

= 1 + 1 − +   − ...... − 1 − +   − .......
z  z z  z z z
 

(iii) 2 < z < 3


⇒ 2 < z, z < 3
2 z
⇒ < 1, < 1
z 3

3 8
f ( z) = 1 + −
z +2 z +3
3 8
= 1+ −
 2  z
z 1 +  31 + 
 z  3
−1 −1
3 2 8 z
= 1 + 1 +  −  1 + 
z z 3 3
3 2 2  8  z  z 2 
2

= 1 + 1 − +   − ...... − 1 − +   − .......
z  z z  3 3 3
 


7z − 2
3. Find Laurent’s series expansion of f (z) = in 1 < z + 1 < 3
z ( z − 2)( z + 1)
7z − 2
Solution: Let f (z) =
z ( z − 2)( z + 1)

BIET 108 DEPARTMENT OF MATHEMATICS


MA6251 MATHEMATICS - II

7z − 2 A B C
= + +
z ( z − 2)( z + 1) z z − 2 z + 1
7 z − 2 = A( z − 2)( z + 1) + Bz ( z + 1) + Cz ( z − 2)
Put z = 0, A = 1
z = 2, B = 2
z = −1, C = −3

7z − 2
F (z) =
z ( z − 2)( z + 1)
1 2 3
f ( z) = + −
z z − 2 z +1

Given 1 < z + 1 < 3


Now put t = z + 1, then 1 < t < 3
⇒1< t, t < 3
1 t
⇒ < 1, < 1
t 3
1 2 3
∴ f ( z) = + −
t −1 t − 3 t
3 1 2
= − + −
t  1  t
t 1 −  31 − 
 t  3
−1 −1
3 1 1 2 t 
= − + 1 −  − 1 − 
t t t 3  3
3 1  1  1   2  t  t 2 
2

= − + 1 + +   + .....  − 1 + +   + ...... 
t t  t  t   3  3 3
 


1   2 z + 1  z +1 
2 2
3 1  1   
=− + 1+ +  + ..... − 1 + +  + ...... 
z + 1 z + 1  z + 1  z +1  3
  3  3  

5.4 CALCULUS OF RESIDUES

Definition: Zero of an analytic function


A point z = a is said to be a zero of an analytic function f (z) if f(z) is
zero at z = a.

BIET 109 DEPARTMENT OF MATHEMATICS


MA6251 MATHEMATICS - II

If f (a) = 0 and f’ (a) ≠ 0 then z = a is called a simple zero of f (z) (or) a


zero of the first order.
If f (a) = f ' (a) = ..... = f n −1 (a) = 0 & f n (a) ≠ 0 then z = a is a zero of
order n.
Example; Let f (z) = z 2
Then f ' ( z ) = 2 z , f ' ' ( z ) = 2
f (0), f ' (0) = 0, f ' ' (0) = 2 ≠ 0
∴ z = 0 is a zero of order 2.

Singular point:
A point z = a is said to be a singular point (or) singularity of f (z) is not
analytic at z =a.

Types of Singular Point:


Isolated Singular Point:
A point z = a is said to be an isolated singular point of f (z) if
(i) f (z) is not analytic at z = a
(ii) f (z) is analytic at all points for some neighbourhood of z = a

z
Example: f ( z ) =
(z − 1)( z − 2)
Then z = 1, 2 are isolated points.

Pole:
A point z = a is said to be a pole of f(z) of order n if we can find a positive
integer such that lim( z − a ) n f ( z ) ≠ 0
z →a

Essential singular point:


A singular point z = a is said to be an essential point f (z) if the Laurent’s
series of f (z) about z = a possesses the infinite number of terms in the principal
part (terms containing negative powers).

Example
Let f ( z ) = e1 2
Clearly z = 0 is a singular point

Also f ( z ) = e
1 z
= 1+
1z
+
1( )
z
2

+ .......
1! 2!
1 1 1 
= 1 + +  2  + .......
z 2!  z 
∴ z = 0 is an essential singular point

BIET 110 DEPARTMENT OF MATHEMATICS


MA6251 MATHEMATICS - II

Removable singular point:


A singular point z = a is said to be a removable singular point of f(z) if the
Laurent’s series of f(z) about z = a does not contain the principal part.

Example
sin z
Let f ( z ) =
z
Clearly z = 0 is a singular point

sin z 1  z3 z5 
f ( z) = = z − + − ........
z z 3! 5! 
2 4
z z
=1 − + − .........
3! 5!
Z = 0 is a removable singular point.

f (z ) =
tan z
1. Classify the nature of the singular point of
z

tan z
Solution : f (z ) =
z

1 z3 
=  z + + ...... 
z 3 

z2
= 1+ + .....
3
This is the Laurent’s series of f ( z ) about z=0 and there is no principal part.

∴ Z=0 is a removable singular point.

Also f ( z ) =
tan z sin z
=
z z cos z

Poles of f(z) are z cos z = 0

⇒ z = 0, z = nπ , n = 0,±1,....

⇒ z = 0, nπ are simple poles (pole of order 1)

BIET 111 DEPARTMENT OF MATHEMATICS


MA6251 MATHEMATICS - II

2. Consider the function f (z ) =


sin z
. Find the pole and its order.
z4

sin z
Solution: f ( z) =
z4

1  z3 z5 
= 4 z − + − ......
z  3! 5! 

1 1 z
= 3
− + − .......
z 6 z 120

∴ z = 0 is a pole of order 3.

Definition
A function f (z ) is said to be an entire function or integral function if it is
analytic everywhere in the finite plane except at infinity.

Example f ( z ) = e z , sin z , cos z

Definition
A function f (z ) is said to be a meromorphic function if it is analytic
everywhere in the finite plane except at finite number of poles.

Example
cos πz 2
Consider f ( z ) =
(z − 1)(z + 2)
Then f (z ) is not analytic at z=1,-2
∴ f (z 0 is a meromorphic function.

Definition
The residue of a function f (z ) at a singular point z = a is the coefficient

1
b1 of in the Laurent’s series of f (z ) about the point z = a
z−a

BIET 112 DEPARTMENT OF MATHEMATICS


MA6251 MATHEMATICS - II

5.5 EVALUATION OF RESIDUES

1. Suppose z = a is a pole of order 1

Then {Re sf ( z ) }z = a = Lim (z − a) f (z)


z→ a

2. Suppose z = a is a pole of order n

1 d n −1
Then {Re sf ( Z )}z = a = Lim ( z − a )n f ( z )
z =a (n − 1)! dz n −1

P(z )
3. Suppose z =a is a pole of order 1 and f ( z ) =
Q( z )

P(a )
Then { Res f ( z ) } z = a =
Q ' (a )

CAUCHY’S RESIDUE THEOREM

If f (z ) is analytic at all point inside and on a simple closed curve C


Except at a finite number of point z1, z 2, z3 .....zn inside C
Then
∫ f ( z )dz = 2πi [Sum of residues of f(z) at z
c
z z ....zn ]
1, 2, 3,

Proof
Given that f (z ) is not analytic
Only at z1 , z 2 , z3, ...z n
Draw the non intersecting small
Circles c1 , c2 , c3 ...cn with centre at
z1 , z 2 , z3 ,...z n and radii ρ1 , ρ 2 , ρ 3 ...ρ n
Then f (z ) is analytic in the region
Between c and c1 , c2 , c3 ...cn

BIET 113 DEPARTMENT OF MATHEMATICS


MA6251 MATHEMATICS - II

∫ f ( z )dz = ∫ f ( z )dz + ∫ f ( z )dz + ... + ∫ f ( z )dz


c c1 c2 cn
…..(1)

Now z1 , z 2 , z3 ...z n are the singular points of f (z ) .

1
∴ Res f ( z ) z = zi = the coefficient of in the Laurent’s series of
z − zi
f (z ) about z = zi (by definition of residues)

1 f (z )
= b1 = ∫
2πi c1 ( z − z1 )1−n
dz

 1 f ( z) 
Since  bn = ∫c (z − z1 )z −n 
dz
 2π i
 1 

1 f (z )
= ∫
2πi c1 ( z − z1 )0
dz

f ( z )dz
1
2πi c∫1
=

⇒ ∫ f ( z )dz = 2πi Re sf ( z ) z = z …..(2)


i
c1

From (1) and (2)


∫ f (z )dz = 2π Re sf (z ) z = z + 2π Re sf (z ) z = z + 2π Re sf (z ) z = z
c1
1 2 n

= 2π Re sf ( z ) z = z + Re sf ( z ) z = z + ... + Re sf ( z ) z = z
1 2 n

= 2πi {Sum of residues of f ( z ) at z = z1 , z2 , z3 ...zn }

z+2
Example 1 Find the residue of f(z) = about each singularity.
(z − 2)(z + 1)2
Solution: The poles of f (z) are given by
(z − 2) = 0, z + 1 = 0
⇒ z = 2, z = −1
∴ The poles of f (z) are z = 2 is a simple poles and z = -1 is a pole of order 2.
∴ [Re sf ( z )]z = 2 = lim( z − 2) f ( z )
z →2

BIET 114 DEPARTMENT OF MATHEMATICS


MA6251 MATHEMATICS - II

z+2
= lim( z − 2)
4
=
z →2 (z − 2)(z + 1) 9
2

∴ [Re sf ( z )]z = −1 = lim [( z − 2) f ( z )]


d 2
z → −1 dz

z+2
= lim [(z − 2)
d 2
]
z → −1 dz ( z − 2)( z + 1) 2
d  z + 2
= lim 
z → −1 dz z − 2 
 
 ( z − 2 )(1) − ( z + 2 )(1)  4
= lim  =−
z → −1
 (z − 2) 2
 9

dz
Example 2. Evaluate ∫ (z
c
2
+4 )
2
where c is the circle z − i = 2

1
Solution: Given f ( z ) =
+4 (z 2
) 2

z = ±2i is a pole of order 2


Here z = 2i lies inside the circle z − i = 2
And z = −2i lies outside the circle and is of order 2.
∴ [Re sf ( z )]z = zi = lim ( z − 2i ) f ( z )
d 2

dz
z →2i

= lim
d
(z − 2i)2 1

z→2i
dz (z − 2i) (z + 2i)2
2

= lim
d
(z − 2i)2
dz
z →2i

= lim
( z + 2i) (0) − 2( z + 2i)
2

z →2i (z + 2i)4
8i
=−
256

∴By Residue theorem


dz  − 8i  π
∫ (z = 2πi =
c
2
+4 )
4
 256 6

BIET 115 DEPARTMENT OF MATHEMATICS


MA6251 MATHEMATICS - II

CONTOUR INTEGRATION
TYPE: I

Integrals of the type ∫ f (cos θ , sin θ )dθ
0
where f (cos θ , sin θ ) is

A rational function of sin θ & cos θ . In this case we take unit circle
z = 1 as the contour. On z = 1 .

z = e iθ , cos θ =
1 iθ
2
( 1 1
e + e − iθ =  z + 
2 z
)
dz = ie iθ dθ , sin θ =
1 iθ
2i
( 1 1
e − e − iθ =  z − 
2i  z
)
dz
⇒ dθ =
iz
Also θ various from 0 to 2 π


 z 2 + 1 z 2 − 1  dz
∴ ∫ f (cos θ , sin θ )dθ = ∫ f  , 
0 c  2z 2iz  iz
Now applying Cauchy’s residue theorem, we can evaluate the
Integral on the right side.


Example 1. Evaluate ∫ 13 + 5 sin θ by using contour Integration.
0


Solution: I = ∫ 13 + 5 sin θ limit : 0 to 2 π
0

Contour: z = 1
Put z = e iθ
dz = ie iθ dθ , sin θ =
1 iθ
2i
( )1 1
e − e − iθ =  z − 
2i  z
dz
⇒ dθ =
iz
dz
I =∫
  z 2 −1  
iz 13 + 5
c
 
  2 iz 
dz
= 2∫ 2
c
5 z + 26iz − 5
1
Where f ( z ) =
5 z + 26iz − 5 2

∴ I = 2 ∫ f ( z )dz ………….(1)
c

BIET 116 DEPARTMENT OF MATHEMATICS


MA6251 MATHEMATICS - II

To find Residue:
The poles of f (z) are 5 z 2 + 26iz − 5
− 26 ± (26i )2 + 100
Z=
10
− 26 ± 24 − i
= = ,−5i
10 5
−i
The pole z = lies inside the circle z = 1 and z = -5i lies outside the circle z = 1
5
 i
Now ∴ [Re sf ( z )]z = −i = lim  z +  f ( z )

5 5
5 z→ i

 i 1
= lim  z + 
z →−i  5  i
5
5 z + ( z + 5i )
 5
1 1
= =
 −i  24i
5 + 5i 
 5 
∴ By Cauchy’s Residue theorem
∫ f ( z )dz = 2πi∑ R
c

1 π
= 2πi =
24i 12
(1) becomes

2π π
I= =
12 6


sin 2 θ
Example 2: Evaluate ∫0 a + b cosθ dθ , a > b > 0 using contour integration.
2π sin 2 θ
dθ ,
Solution: Let I = ∫ a + b cos θ
0


1 − cos 2θ
= ∫ 2a + 2b cosθ dθ
0

We can write cos 2θ = Real part of e 2iθ ,Q e 2iθ = cos 2θ + i sin 2θ


1 − e 2iθ
∴ I = R.P ∫ dθ
0
2a + 2b cos θ
Put z = e iθ

BIET 117 DEPARTMENT OF MATHEMATICS


MA6251 MATHEMATICS - II

dz z2 +1
dθ = , cos θ =
iz z

1− z 2
dz
∴ I = R.P ∫
 z + 1  iz
2
0
2a + b 
 z 

1 1− z2
i ∫0 bz 2 + 2az + b
= R.P dz


1
i ∫0
= R.P f ( z )dz ………(1)

1− z2
Where f ( z ) =
bz 2 + 2az + b
To find Residues:
Poles of f(z) are given by bz 2 + 2az + b = 0

− 2a ± 4a 2 − 4b 2
Z=
2b
− a ± a2 − b2
=
b
− a + a2 − b2 − a − a2 − b2
Let α = and β = , since a>b,
b b

α < 1& β > 1


∴ The simple pole z = α lies inside C,

∴ [Re sf ( z )]z =α = lim(z − α ) f ( z )


z →α

1− z2
= lim( z − α )
z →α b(z − α )( z − β )
1−α 2 a − a2 − b2
= =
b(α − β ) b2
a − a2 − b2
∴∑ R =
b2

∴ By Cauchy’s Residue theorem


∫ f ( z )dz = 2πi∑ R
c

 a − a2 − b2 
= 2πi 
 b2 
 

BIET 118 DEPARTMENT OF MATHEMATICS


MA6251 MATHEMATICS - II

(1) becomes


1
I= R.P ∫ f ( z )dz
i 0
 a − a2 − b2 
= R.P 2π  
 b 2 
 

= 2 a − a2 − b2
b
( )
Type II Integration around semi-circular contour
Consider the integral

P( x)
Improper integrals of the form ∫ dx ,where P(x) and Q(x)
−∞
Q( x)
Are polynomials in x such that the degree of Q exceeds that of P atleast by two
and Q(x) does not vanish for any x.


x2 − x + 2 5π
Example 1: Prove that ∫−∞ x 4 + 10 x 2 + 9 dx = 12 using contour integration.
z2 − z + 2
Solution: Let f ( z ) =
z 4 + 10 z 2 + 9
Consider ∫
C
f ( z )dz where C is the closed contour consisting of Γ, semi- large

circle of radius R and the real axis from –R to R.


R
Then ∫ f ( z )dz = ∫ f ( z )dz + ∫ f ( x)dx ………(1)
c Γ −R

z −z+2 2
Now f ( z ) = → 0 as z → ∞
z + 10 z 2 + 9
4

∴ lim zf ( z ) = 0
z →∞

Hence from (1) ∫ f ( x)dx = ∫ f ( z )dz
−∞ C

By using residue theorem, ∫ f ( z )dz = 2πi∑ R


C

∴ ∫ f ( x)dx = 2πi Re sf ( z )
−∞
The poles of f(z) are given by
z 4 + 10 z 2 + 9 = 0
(z 2
)(
+ 9 z2 +1 = 0 )
z = ±i, z = ±3i
The poles z = 3i, z = i lies in the upper half of the z – plane

BIET 119 DEPARTMENT OF MATHEMATICS


MA6251 MATHEMATICS - II

[Re sf ( z )]z =i = lim


z →i
(z − i ) f ( z )
z2 − z + 2
= lim( z − i )
z →i (
(z + i )(z − i ) z 2 + 9 )
1− i 1− i
= =
8(2i ) 16i

[Re sf ( z )]z =3i = lim


z → 3i
(z − 3i ) f ( z )
z2 − z + 2
= lim (z − 3i )
z → 3i (z − 3i )(z + 3i ) z 2 + 9 ( )
z −z+2 2
= lim
(z + 3i ) z 2 + 9
z → 3i ( )
− 7 − 3i 7 + 3i
= =
(6i )(− 8) 48i
1 − i 7 + 3i
∴ ∑ Re sf ( z ) = +
16i 48i
10
=
48i
5
=
24i


 5  5π
∫ f ( x)dx = 2πi 24i  = 12
−∞


x2
Evaluate ∫(
−∞
)(
x 2 + a 2 x 2 + b2 )
dx by using contour

Solution: Consider the integral ∫ f ( z )dz where


C
2
z
f ( z) =
(z
+ a z 2 + b2
2 2
)( )
And C is the closed contour consisting of Γ, the upper semi large
Circle z = R and the real axis from –R to R
R

∫ f ( z )dz = ∫ f ( z )dz + ∫ f ( x)dx


c Γ −R

When R → ∞, ∫ f ( z )dz
Γ

Hence ∫ f ( x)dx = ∫ f ( z )dz
−∞ C

BIET 120 DEPARTMENT OF MATHEMATICS


MA6251 MATHEMATICS - II

By using residue theorem, ∫ f ( z )dz = 2πi∑ R


C

Poles of f(z) are given by z 2 + a 2 = 0, z 2 + b 2 = 0


z = ± ai, z = ±bi
The pole z = ai and z = bi lies in the upper half plane

[Re sf ( z )]z =ai = lim


z → ai
(z − ai ) f ( z )
z2
= lim ( z − ai )
z → ai (z + ai )(z − ai )(z 2 + b 2 )
z2
= lim
z → ai ( z + ai ) z 2 + b 2 ( )
(ai ) 2
=
(
2ai − a 2 + b 2 )
a
=
(
2i a − b 2
2
)
[Re sf ( z )]z =bi = lim
z →bi
(z − bi ) f ( z )
z2
= lim ( z − bi )
z →bi (
(z + bi )(z − bi ) z 2 + a 2 )
−b 2
=
(
2bi − b 2 + a 2 )
−b
=
(
2i a 2 − b 2 )

2πi  a b 
∴ ∫ f ( z )dz = − 2
−∞
2i  a − b a − b 2 
 2 2

 a −b 
=π  
 (a + b )(a − b ) 
 1 
= π 
 (a + b ) 
Type III
∞ ∞
P ( x) P( x)
Integrals of the type ∫ sin nxdx and ∫ cos nxdx
−∞
Q( x) −∞
Q( x)


x sin mx
Example 1: Evaluate ∫x
0
2
+ a2
dx, m > 0, a > 0 by the method of residue

ze imz
Solution: Consider ∫
C
f ( z )dz where f ( z ) =
z 2 + a2

BIET 121 DEPARTMENT OF MATHEMATICS


MA6251 MATHEMATICS - II

And C is the contour consisting of Γ a semi circle z = R


In the upper half plane and the real axis from –R to R.
R
Then ∫
c
f ( z )dz = ∫ f ( z )dz +
Γ
∫ f ( x)dx
−R

When R → ∞, ∫ f ( z )dz
Γ

Hence ∫ f ( x)dx = ∫ f ( z )dz
−∞ C

By using residue theorem, ∫ f ( z )dz = 2πi∑ R


C

Hence ∫ f ( x)dx = 2πi∑ R
−∞

The pole of f(z) are given by z 2 + a 2 = 0 ⇒ z = ± ai


The simple pole z = ai lies in the upper half plane

[Re sf ( z )]z =ai = lim


z → ai
(z − ai ) f ( z )
ze imz
= lim ( z − ai )
z → ai (z + ai )(z − ai )
aie − ma e − ma
= =
2ai 2


e − ma
∴ ∫ f ( x)dx = 2πi = πie − ma
−∞
2

xeimx
⇒ ∫ 2 dx = πie − ma
−∞
x +a 2


x(cos mx + i sin mx)
∫ dx = πie − ma
−∞
x +a
2 2

Equating imaginary parts we get


x sin mx
−∞
∫ x2 + a2
dx = πie − ma


x sin mx π
⇒∫ dx = e − ma
0
x +a
2 2
2

BIET 122 DEPARTMENT OF MATHEMATICS


MA6251 MATHEMATICS - II FORMULAE

UNITWISE FORMULAE

Ordinary Differential Equation

1. ODE with constant coefficients: Solution y  C.F + P.I

Complementary functions:

Sl.No. Nature of Roots C.F


1. m1  m 2 ( Ax  B )e mx
2. m1  m2  m3  Ax 2
 Bx  c  e mx
3. m1  m 2 Ae m1 x  Be m2 x
4. m1  m2  m3 Ae m1 x  Be m2 x  Ce m3 x
5. m1  m2 , m3 ( Ax  B )e mx  Ce m3 x
6. m    i e x ( A cos  x  B sin  x )
7. m  i A cos  x  B sin  x

Particular Integral:

Type-I
If f ( x )  0

then P . I  0

Type-II
If f ( x )  e ax
1 ax
P .I  e
 ( D)

BIET DEPARTMENT OF MATHEMATICS


MA6251 MATHEMATICS - II FORMULAE
Replace D by a . If  ( D)  0 , then it is P.I. If  ( D)  0 , then diff. denominator
w.r.t D and multiply x in numerator. Again replace D by a . If you get denominator
again zero then do the same procedure.

Type-III

Case: i If f ( x )  sin ax (or ) cos ax


1
P .I  sin ax (or) cos ax
 ( D)
Here you have to replace only for D2 not for D . D2 is replaced by  a 2 . If the
denominator is equal to zero, then apply same procedure as in Type – I.

Case: ii If f ( x )  Sin2 x (or) cos 2 x (or) sin 3 x (or) cos 3 x

1  cos 2 x 1  cos 2 x
Use the following formulas Sin2 x  , cos 2 x  ,
2 2
3 1 3 1
sin 3 x  sin x  sin 3 x , cos 3 x  cos x  cos 3 x and separate P . I1 & P . I 2
4 4 4 4

Case: iii If f ( x )  sin A cos B (or ) cos A sin B (or ) cos A cos B (or ) sin A sin B
Use the following formulas:
1
( i ) s in A cos B   sin( A  B )  sin( A  B ) 
2
1
(ii) cos A sin B   Sin( A  B )  sin( A  B ) 
2
1
( iii ) cos A cos B   cos( A  B )  cos( A  B ) 
2
1
( iv ) sin A sin B   cos( A  B )  cos( A  B ) 
2

Type-IV
If f ( x )  x m
1
P.I  xm
 ( D)
1
 xm
1  g ( D)

BIET DEPARTMENT OF MATHEMATICS


MA6251 MATHEMATICS - II FORMULAE

 1  g ( D)  x m
1

Here we can use Binomial formula as follows:

i) 1  x   1  x  x 2  x3  ...
1

ii) 1  x   1  x  x 2  x3  ...
1

iii) 1  x   1  2 x  3x2  4 x3  ...


2

iv) 1  x   1  2 x  3x 2  4 x3  ...
2

v) (1  x) 3  1  3x  6 x 2  10 x3  ...
vi) (1  x) 3  1  3x  6 x 2  10 x3  ...

Type-V
If f ( x)  e axV where V  sin ax, cos ax, x m
1 ax
P.I  e V
 ( D)

First operate e ax by replacing D by D+a.

1
 eax V
 ( D  a)

Type-VI
If f ( x)  x nV where V  sin ax,cos ax

sin ax  I.P of eiax


cos ax  R.P of eiax

Type-VII (Special Type Problems)

If f ( x)  sec ax (or) cosecax (or) tan ax

1
P.I  f ( x)  eax  e ax f ( x)dx
Da

1. ODE with variable co-efficient: (Euler’s Method)

d2y dy
The equation is of the form x 2 2
 x  y  f ( x)
dx dx

BIET DEPARTMENT OF MATHEMATICS


MA6251 MATHEMATICS - II FORMULAE
Implies that ( x 2 D 2  xD  1) y  f ( x)
To convert the variable coefficients into the constant coefficients
Put z  log x implies x  e z
xD  D
d d
x 2 D 2  D( D  1) where D  and D 
dx dz
x 3 D 3  D( D  1)( D  2)

The above equation implies that  D(D 1)  D  1 y  f ( x) which is O.D.E

with constant coefficients.


2. Legendre’s Linear differential equation:
d2y dy
The equation if of the form (ax  b)2 2  (ax  b)  y  f ( x)
dx dx
Put z  log(ax  b) implies (ax  b)  e z
(ax  b) D  aD
d d
(ax  b) 2 D 2  a 2 D( D  1) where D  and D 
dx dz
(ax  b)3 D 3  a 3 D( D  1)( D  2)

3. Method of Variation of Parameters:

d2 y dy
The equation is of the form a 2
 b  cy  f ( x )
dx dx
C.F  Ay1  By2 and

P.I  Py1  Qy2


y2 f ( x )
where P    dx and
y1 y2  y1 y2
y1 f ( x)
Q dx
y1 y2  y1 y2

Vector Calculus
1. Vector differential operator is   i  / x  j  / y  k  / z

2. Gradient of     i  / x  j  / y  k  / z

3. Divergence of F    F

BIET DEPARTMENT OF MATHEMATICS


MA6251 MATHEMATICS - II FORMULAE

i j k
4. Curl of F  XF   / x  / y  / z
F1 F2 F3

5. If F is Solenoidal vector then  F  0


6.

7. If F is Irrotational vector the XF  0


8. Maximum Directional derivative  

 a
9. Directional derivative of  in the direction of a 
a

n1 n2
10. Angle between two normals to the surface cos  
n1 n2

Where n1   1 at ( x , y , z ) & n2   2 at ( x , y , z


1 1 1 2 2 2)


11. Unit Normal vector, n̂ 


12. Equation of tangent plane l ( x  x1 )  m( y  y1 )  n( z  z1 )  0

Where l, m,n are coefficient of i , j , k in  .


13. Equation of normal line
x  x1 y  y1 z  z1
 
l m n

14. Work Done =  F dr , where dr  dxi  dyj  dzk


C

15. If  F .dr be independent of the path is that curlF  0


C

dxdy dydz dzdx


16. In the surface integral dS  , dS  , dS  & dS  ndS
ˆ
nˆ.k nˆ.i nˆ. j

BIET DEPARTMENT OF MATHEMATICS


MA6251 MATHEMATICS - II FORMULAE

17. Green’s Theorem:


u v
If u, v, , are continuous and one-valued functions in the region R enclosed
y x

 v u 
by the curve C, then  udx  vdy     dxdy .
C R 
x y 

18. Stoke’s Theorem:

Let F be the vector point function, around a simple closed curve C and over the

open surface S having as its boundary, then  F dr    xF  nds


C S
ˆ

19. Gauss Divergence Theorem:

Let F be a vector point function in a region R bounded by a closed surface S,

then  F nds
S
ˆ    Fdv
V

Analytic Functions

1. Necessary condition for f(z) is analytic function


u v v u
Cauchy – Riemann Equations:  &  (C-R equations)
x y x y

u 1 v v 1 u
2. Polar form of Cauchy-Riemann Equations:  & 
r r  r r 
 2u  2u
3. Condition for Harmonic function:  0
x 2 y 2
4. If the function is harmonic then it should be either real or imaginary part of a
analytic function.
5. Milne – Thomson method: (To find the analytic function f(z))

i) If u is given f ( z)   ux ( z,0)  iu y ( z,0)  dz

ii) If v is given f ( z )   vy ( z,0)  ivx ( z,0)  dz

BIET DEPARTMENT OF MATHEMATICS


MA6251 MATHEMATICS - II FORMULAE

6. To find the analytic function


i) f ( z )  u  iv ; if ( z)  iu  v adding these two
We have (u  v)  i(u  v)  (1  i) f ( z )
then F ( z )  U  iV where U  u  v,V  u  v & F ( z )  (1  i) f ( z )
Here we can apply Milne – Thomson method for F(z).

7. Bilinear transformation:
 w  w1  w2  w3    z  z1  z2  z3 
 w1  w2  w3  w  z1  z2  z3  z 
Complex Integration
1. Cauchy’s Integral Theorem:
If f(z) is analytic and f ( z ) is continuous inside and on a simple closed curve C,

then  f ( z)dz  0 .
c

2. Cauchy’s Integral Formula:


If f(z) is analytic within and on a simple closed curve C and z 0 is any point inside

f ( z)
C, then  z  a dz  2 if (a)
C

3. Cauchy’s Integral Formula for derivatives:


If a function f(z) is analytic within and on a simple closed curve C and a is any
1 f ( z)
point lying in it, then f (a)  
2 i C  z  a 2
dz

2! f ( z) n! f ( z)
Similarly f (a)  
2 i C  z  a  3
dz , In general f ( n ) (a)  
2 i C  z  a n1
dz

4. Cauchy’s Residue theorem:


If f(z) be analytic at all points inside and on a simple closed cuve c, except for a
finite number of isolated singularities z1 , z2 , z3 ,...zn inside c, then

 f ( z)dz  2 i(sum of the residues of f ( z)) .


C

BIET DEPARTMENT OF MATHEMATICS


MA6251 MATHEMATICS - II FORMULAE
5. Critical point:
The point, at which the mapping w = f(z) is not conformal, (i.e) f ( z )  0 is called
a critical point of the mapping.
6. Fixed points (or) Invariant points:
az  b
The fixed points of the transformation w  is obtained by putting w = z in
cz  d
the above transformation, the point z = a is called fixed point.
7. Re s{ f ( z )}  Lt ( z  a) f ( z ) (Simple pole)
z a

8. Re s{ f ( z )} 
1 d m 1
Lt m 1
(m  1)! z  a dz
 z  a  m
f ( z)  (Multi Pole (or) Pole of order m)

P( z )
9. Re s{ f ( z )}  Lt
z a Q( z )

10. Taylor Series:


A function f ( z ) , analytic inside a circle C with centre at a, can be expanded in
the series
( z  a) ( z  a) 2 ( z  a)3 ( z  a) n ( n )
f ( z )  f (a)  
f (a )  
f (a )  
f (a )  ...  f (a )  ...
1! 2! 3! n!
Maclaurin’s Series:
Taking a = 0, Taylor’s series reduce to
z z2 z3
f ( z )  f (0)  f (0)  f (0)  f (0)  ...
1! 2! 3!
 
bn
11. Laurent’s Series: f ( z )   an ( z  a ) n  
n 1 ( z  a )
n
n0

1 f ( z) 1 f ( z)
where an  
2 i C1 ( z  a) n 1
dz & bn  
2 i C2 ( z  a)1n
dz , the integrals being

taken anticlockwise.
12. Isolated Singularity:
A point z  z0 is said to be isolated singularity of f ( z ) if f ( z ) is not analytic at

z  z0 and there exists a neighborhood of z  z0 containing no other singularity.

BIET DEPARTMENT OF MATHEMATICS


MA6251 MATHEMATICS - II FORMULAE
1
Example: f ( z)  . This function is analytic everywhere except at z  0 .
z
 z  0 is an isolated singularity.
13. Removable Singularity:
lim f ( z)
A singular point z  z0 is called a removable singularity of f ( z ) if
z  z0

exists finitely.
sin z
lim f ( z )  lim 1
Example: z (finite)  z  0 is a removable
z 0 z 0
singularity.
14. Essential Singularity:
If the principal part contains an infinite number of non zero terms, then z  z0 is

known as a essential singularity.

1/ z 1/ z 
1 2

Example: f ( z)  e  1 
z
  ... has z  0 as an essential
1! 2!
singularity.
CONTOUR INTEGRATION:
15. Type: I
2
The integrals of the form 0
f (cos  ,sin  )d Here we shall choose the contour

z2 1
as the unit circle C : z  1 or z  ei ,0    2 . On this type cos   ,
2z
z2 1 1
sin   and d  dz .
2iz iz
16. Type: II

P( x)
Improper integrals of the form  Q( x) dx , where P(x) and Q(x) are polynomials


in x such that the degree of Q exceeds that of P at least by two and Q(x) does not
R
vanish for any x. Here 
C
f ( z )dz  
R
f ( x)dx   f ( z )dz as R  
  f ( z)dz  0 .

BIET DEPARTMENT OF MATHEMATICS


MA6251 MATHEMATICS - II FORMULAE
17. Type: III
 
The integrals of the form 

f ( x) cos mxdx (or) 

f ( x) sin mxdx where

f (x) 0 as x .

Laplace Transform

1. Definition: L  f (t )    e  st f (t )dt
0

2.
Sl.No
L 1
1
1.
s
n ! (n  1)
2. L t n  
s n 1 s n 1
1
3. L eat 
sa
1
4. L e at 
sa
L sin at 
a
5.
s  a2
2

L cos at 
s
6.
s  a2
2

L sinh at 
a
7.
s  a2
2

L cosh at 
s
8.
s  a2
2

3. Linear Property: L af (t )  bg (t )  aL  f (t )  bL  g (t )

4. First Shifting property:


If L  f (t )  F (s) , then

i) L eat f (t )    F (s)ss a

ii) L e at f (t )    F (s)ss a

BIET DEPARTMENT OF MATHEMATICS


MA6251 MATHEMATICS - II FORMULAE
5. Second Shifting property:
 f (t  a), t  a
If L  f (t )  F (s) , g (t )   then L  g (t )  eas F (s)
0, ta

6. Change of scale:
1 s
If L  f (t )  F (s) , then L  f (at )   F 
a a

7. Transform of derivative:
d2
If L  f (t )  F (s) , then L tf (t )  
d
F ( s) , L t 2 f (t )   2 F ( s ) ,
ds ds
dn
In general L t n f (t )   (1) n n F ( s)
ds
8. Transform of Integral;

1 
If L  f (t )  F (s) , then L  f (t )    F ( s)ds
t  s
9. Initial value Theorem:
Lt f (t )  Lt sF ( s )
If L  f (t )  F (s) , then
t 0 s
10. Final value Theorem:
Lt f (t )  Lt sF ( s )
If L  f (t )  F (s) , then
t  s0
11.
Sl.No
1 
1. L1   1
s
 1 
2. L1  e at
 s  a 
 1 
3. L1  e  at
 s  a 
 s 
4. L1  2
 s  a 
2 cosat

 1  1
5. L1  2 sin at
 s  a 
2
a

BIET DEPARTMENT OF MATHEMATICS


MA6251 MATHEMATICS - II FORMULAE

 s 
6. L1  2
 s  a 
2 cosh at

 1  1
7. L1  2 sinh at
 s  a 
2
a
1 t n 1
8. L1  n 
s  ( n  1)!

12. Deriative of inverse Laplace Transform:


1 1
L1  F ( s)  L  F ( s)
t
t

13. Colvolution of two functions: f (t )  g (t )   f (u ) g (t  u)du


0

14. Covolution theorem:


If f(t) & g(t) are functions defined for t  0 then L  f (t )  g (t )  L  f (t ) L  g (t )

15. Convolution theorem of inverse Laplace Transform:

L1  F (s)G(s)  L1  F (s)  L1 G(s)

16. Solving ODE for second order differential equations using Laplace Transform
i) L  y(t )  sL  y(t )  y(0)
ii) L  y(t )  s2 L  y(t )  sy(0)  y(0)

iii) L  y(t )  s3 L  y(t )  s 2 y(0)  sy(0)  y(0) take y  L  y(t )

 t
y (t )dt   L  y (t ) 
1
17. Solving integral equation: L 
 
0  s
18. Inverse Laplace Transform by Contour Integral method
1
L1  F ( s )    F ( s )e st ds
2 i c

19. Periodic function in Laplace Transform:


If f(x+T) = f(x), then f(x) is periodic function with period T.
1
L  f (t ) 
T

1  e sT 
0
e st f (t )dt

BIET DEPARTMENT OF MATHEMATICS


MA6251 UNIVERSITY QUESTIONS

B.E./B.Tech. DEGREE EXAMINATION, MAY/JUNE 2009

Second Semester

Civil Engineering

MA2161 – MATHEMATICS – II

(Common to all branches of B.E./B.Tech. )

(Regulation 2008)

Time : Three hours Maximum : 100 marks

Answer ALL questions

PART A – (10 x 2 = 20 marks)

 
1. Find the particular integral of D2  2 D  1 y  e  x cos x .

2. Solve the equation x 2 y  xy  y  0 .

3. Find the values of a , b, c so that the vector F   x  y  az  i   bx  2 y  z  j 

  x  cy  2z  k may be irrotational.

4. State Green’s theorem in a plane.

5. State the Cauchy-Riemann equation in polar coordinates satisfied by an analytic function.

2z  6
6. Find the invariant points of the transformation w  .
z7

7. Evaluate  tan z dz where C is z  2 .


c


8. Find the Taylor series for f ( z )  sin z about z  .
4

1  cos t
9. Find the Laplace transform of .
t

1 k
10. Find the inverse Laplace transform of cot  .
s

BIET DEPARTMENT OF MATHEMATICS


MA6251 UNIVERSITY QUESTIONS
PART B – (5 x 16 = 80 marks)

11. (a) (i) Solve the equation D 2


 4 y  x 2 cos 2 x .

 
(ii) Solve the equation D2  a 2 y  tan ax by the method of variation of

parameters.

Or

 
(b) (i) Solve the equation x 2 D2  3 xD  5 y  x cos  log x  .

dx dy
(ii) Solve  y  sin t , x   cos t given that x  2 and y  0 at t  0 .
dt dt

12. (a) (i) Find the angle between the normals to the surface xy 3 z 2  4 at the points

 1, 1, 2 and  4,1, 1 .


(ii) Verity Stoke’s theorem for F  xyi  2 yzj  zxk where S is the open surface of the
rectangular parallelepiped formed by the planes x  0, x  1, y  0, y  2 and z  3 above the
XY plane.

Or

(b) (i) Find the directional derivative of   2xy  z 2 at the point 1, 1, 3 in the direction

of i  2 j  2k .

2 2
(ii) Verify Gauss divergence theorem for F  x 2 i  y j  z k where S is the surface of
the cuboid formed by the planes x  0, x  a, y  0, y  b, z  0 and z  c .

sin 2 x
13. (a) (i) Find the analytic function f ( z )  P  iQ , if P  Q  .
cosh 2 y  cos 2 x

(ii) Find the bilinear transformation which maps the points z  0,  i , 1 into
w  i ,1,0 respectively.

Or

 2 2  2 2
(b) (i) If f ( z ) is a regular function of z , prove that  2
 2 
f ( z )  4 f ( z ) .
 x y 

BIET DEPARTMENT OF MATHEMATICS


MA6251 UNIVERSITY QUESTIONS

1
(ii) Find the image of the half plane x  c , when c  0 under the transformation w  .
z
Show the regions graphically.

zdz 1
14. (a) (i) Evaluate   z  1 z  2
c
2
where c is the circle z  2 
2
using Cauchy’s integral

formula.

2
d
(ii) Evaluate  1  2 x sin  x 2
,  0  x  1 , using contour integration.
0

Or

z2  1
(b) (i) Find the Laurent’s series of f (z)  valid in the region 2  z  3.
z 2  5z  6

x 2 dx
(ii) Evaluate   x 2  a 2  x 2  b2  , using contour integration, where a  b  0 .

15. (a) (i) Find the Laplace transform of te 2 t cos 3t .

1
(ii) Find the inverse Laplace transform of .
 s  1  s 2  4 

 
(iii) Solve the equation y  9 y  cos 2t , y(0)  1 and y    1 using Laplace
2
transform. Or

t , in 0  t  a
(b) (i) Find the Laplace transform of f ( t )   and f ( t  2a )  f ( t ) .
 2a  t , in a  t  2a
t

 t sin 3t dt .
4 t
(ii) Find the Laplace transform of e
0

BIET DEPARTMENT OF MATHEMATICS


MA6251 UNIVERSITY QUESTIONS

B.E./B.Tech. DEGREE EXAMINATION, Nov/Dec 2009

Second Semester

MA2161 – MATHEMATICS – II

(Common to all branches of B.E./B.Tech. )

(Regulation 2008)

Time : Three hours Maximum : 100 marks

Answer ALL Questions

PART A – (10 x 2 = 20 marks)


1. Solve the equation D2  6 D  13 y  0 . 
2. Find the particular integral of  D  1 y  e  x cos x .
2

 
3. Find grad r n where r  xi  yj  zk and r  r .

4. Find the unit normal to the surface x 2  xy  z 2  4 at 1, 1, 2  .

5. State the basic difference between the limit of a function o a real variable and that of a complex
variable.

6. Prove that a bilinear transformation has at most two fixed points.

7. Identify the type of singularities of the following function: f ( z )  e z 1


.

e2z
8. Calculate the residue of f ( z )  at its pole.
( z  1)2

9. Find the Laplace transform of t cos at .

10. Verify initial value theorem for f (t )  1  e  t  sin t  cos t  .

BIET DEPARTMENT OF MATHEMATICS


MA6251 UNIVERSITY QUESTIONS
PART B – (5 x 16 = 80 marks)

11. (a) (i) Solve the equation D 2


 3 D  2 y  2cos  2 x  3   2e x .


(ii) Apply the method of variation of parameters to solve D2  4 y  cot 2 x . 
Or

 
(b) (i) Solve the differential equation x 2 D2  xD  4 y  x 2 sin  log x  .

dx dy
(ii) Solve the simultaneous differential equations  2 y  sin 2t ,  2 x  cos 2t .
dt dt

12. (a) (i) Prove that curl  u  v    v    u   u    v  udiv v  vdiv u .

x  xy  dx   x 2  y 2  dy where C is the square bounded by the


2
(ii) Evaluate
C

lines x  0, x  1, y  0 and y  1 .

Or

  
(b) (i) Verify Stoke’s theorem when F  2 xy  x 2 i  x 2  y 2 j and C is the 
boundary of the region enclosed by the parabolas y 2  x and x 2  y .

(ii) Evaluate   sin zdx  cos xdy  sin ydz  by using Stoke’s theorem, where C is the
C

boundary of the rectangle defined by 0  x   , 0  y  1, z  3 .

13. (a) (i) Verify that the families of curves u  c1 and v  c2 cut orthogonally, when u  iv  z 3 .

 
(ii) Find the analytic function u  iv , if u   x  y  x 2  4 xy  y 2 . Also find the
conjugate harmonic function v .

Or

1
(b) (i) Find the image of the circle z  1 in the complex plane under the mapping w  .
z

(ii) When the function f ( z )  u  iv is analytic, prove that the curves u  constant and
v  constant are orthogonal.

BIET DEPARTMENT OF MATHEMATICS


MA6251 UNIVERSITY QUESTIONS

zdz 1
14. (a) (i) Evaluate   z  1 z  2
c
2
where c is the circle z  2 
2
by using Cauchy’s integral

formula.

1
(ii) Evaluate f ( z )  in Laurent series valid for the regions z  3 and
 z  1 z  3
1  z  3.

Or

2
d
(b) (i) Evaluate  2  cos .
0


dx
(ii) Evaluate  , a0 using contour integration.
x  a2 
3
2
0

1
15. (a) (i) Using convolution theorem find the inverse Laplace transform of .
 s  1  s  1
2

f (t )  t , 0 t a
(ii) Find Laplace transform of with f ( t  2a )  f ( t ) .
 2a  t , a  t  2a

Or

(b) (i) Find the Laplace transform of square wave function defined by
1, in 0  t  a
f (t )   with period 2a .
 1, in a  t  2a

d2 y
(ii) Solve the differential equation  y  sin 2t ; y(0)  0, y(0)  0 by using
dt 2
Laplace transform method.

BIET DEPARTMENT OF MATHEMATICS


MA6251 UNIVERSITY QUESTIONS
B.E./B.Tech. DEGREE EXAMINATIONS, MAY/JUNE 2010

Regulations 2008

Second Semester

Common to all branches

MA2161 – Mathematics II

Time: Three Hours Maximum: 100 Marks

Answer ALL Questions

PART A – (10 x 2 = 20 Marks)

1. Transform the equation x 2 y   xy   x into a linear differential equation with constant


coefficients.

 
2. Find the particular integral of D2  1 y  sin x .

3. Is the position vector r  xi  yj  zk irrotational? Justify.

4. State Gauss divergence theorem.

5. Verify whether the function u  x 3  3 xy 2  3 x 2  3 y 2  1 is harmonic.

6. Find the constants a , b, c if f ( z )  x  ay  i (bx  cy ) is analytic.

 3z 2  7 z  1  1
7. What is the value of the integral c  z  1  dz where C is z  2 ?
1
8. If f ( z )   2 1  ( z  1)  ( z  1) 2  ...  , find the residue of f ( z ) at z  1 .
z 1

9. Find the Laplace transform of unit step function.


10. Find L1 cot 1 ( s ) .
PART B – (5 x 16 = 80 marks)

 
11. (a) (i) Solve the equation D2  4 D  3 y  e  x sin x .

 
(ii) Solve the equation D2  1 y  x sin x by the method of variation of

parameters.

BIET DEPARTMENT OF MATHEMATICS


MA6251 UNIVERSITY QUESTIONS
OR

 
(b) (i) Solve x 2 D2  2 xD  4 y  x 2  2log x .

(ii) Solve

dx
 2 x  3 y  2e 2 t ,
dt
dy
 3 x  2 y  0.
dt

     
12. (a) (i) Prove that F  6 xy  z 3 i  3 x 2  z j  3 xz 2  y k is irrotational vector and

find the scalar potential such that F    .

  3x  8 y 2  dx   4 y  6 xy  dy where C is the
2
(ii) Verity Green’s theorem for
C

boundary of the region defined by x  y 2 , y  x 2 .

OR

(b) Verify Gauss – divergence theorem for the vector function


f   x 3  yz  i  2 x 2 yj  2k over the cube bounded by x  0, y  0, z  0 and
x  a, y  a, z  a .

13. (a) (i) Prove that every analytic function w  u  iv can be expressed as a function z alone,
not as a function of z .

(ii) Find the bilinear transformation which maps the points z  0,1,  into
w  i ,1,  i respectively.

OR

1
(b) (i) find the image of the hyperbola x 2  y 2  1 under the transformation w  .
z

z
(ii) Prove that the transformation w  maps the upper half of z - plane on to the
1 z
upper half of w - plane. What is the image of z  1 under this transformation?

7z  2
14. (a) (i) Find the Laurent’s series of f ( z )  in 1  z  1  3 .
z ( z  1)( z  2)

BIET DEPARTMENT OF MATHEMATICS


MA6251 UNIVERSITY QUESTIONS

4  3z
(ii) Using Cauchy’s integral formula, evaluate  z( z  1)( z  2) dz , Where ‘ C ’ is the circle
C

3
z  .
2

OR


x2  x  2
(b) (i) Evaluate  x 4  10 x 2  9 dx

using contour integration.

2
d
(ii) Evaluate  2  cos
0
using contour integration.

 
s
15. (a) (i) Apply convolution theorem to evaluate L1  .
  s2  a2 2 
 

(ii) Find the Laplace transform of the following triangular wave function given by
t , 0 t 
f (t )   and f ( t  2 )  f ( t ) .
 2  t ,   t  2

OR

(b) (i) Verify initial and final value theorems for the function f ( t )  1  e  t (sin t  cos t ) .

(ii) Using Laplace transform solve the differential equation y  3 y  4 y  2e  t with
y(0)  1  y(0) .

BIET DEPARTMENT OF MATHEMATICS


MA6251 UNIVERSITY QUESTIONS

B.E./B.Tech. Degree Examination, November/December 2010

Regulations 2008

Second Semester

Common to all branches

MA2161 – Mathematics – II

Time : Three Hours Maximum : 100 Marks

Answer ALL Questions

PART A – (10 x 2 = 20 marks)

( )
1. Reduce the equation x 2 D 2 + xD + 1 y = log x into an ordinary differential equation with
constant coefficients.

( )
2. Find the particular integral of D 2 − 2 D + 2 y = e x cos x .

 
3. Prove that div r = 3 and curl r = 0 .

4. State Stoke’s theorem.

5. Verify whether the function u = x 3 − 3 xy 2 + 3 x 2 − 3 y 2 + 1 is harmonic.

6. Verify whether f ( z ) = z is analytic function or not.

ez
7. Evaluate ∫C z − 1 dz , if C is z = 2 .
−1
8. If f ( z ) = − 2 1 + ( z − 1) + ( z − 1)2 + ... , find the residue of f ( z ) at z = 1 .
z −1

9. Find Laplace transform of t sin 2t .

 1 
10. Find L−1  2 .
 s + 4s + 4 

BIET DEPARTMENT OF MATHEMATICS


MA6251 UNIVERSITY QUESTIONS

Part B – (5 x 16 = 80 marks)

( )
11. (a) (i) Solve D 2 + 16 y = cos 3 x .

d2 y
(ii) Solve by the method of variation of parameters, + 4 y = sec 2 x .
dx 2

OR

( )
(b) (i) Solve x 2 D 2 − 3 xD + 4 y = x 2 cos ( log x ) .

dx dy
(ii) Solve + 2 y = − sin t , −2x = cost given x = 1 and y = 0 at t = 0 .
dt dt

12. (a) (i) If r is the position vector of the point ( x , y , z ) , Prove that ∇ 2 r n = n( n + 1)r n − 2 .


∫ ( 3 x )
− 8 y 2 dx + ( 4 y − 6 xy ) dy  where C is
2
(ii) Verify Green’s theorem in plane for
C

the boundary of the region bounded by the lines x = 0, y = 0 , and x + y = 1 .

OR
   
(b) (i) Verify Gauss’s divergence theorem for F = 4 xzi − y 2 j + yzk over the cube bounded
by x = 0, x = 1, y = 0, y = 1, z = 0 and z = 1 .

13. (a) (i) Prove that u = e x ( x cos y − y sin y ) is harmonic and hence find the analytic function
f ( z ) = u + iv .

(ii) Find the bilinear transformation that transforms 1, i and −1 of the z – plane onto 0, 1
and ∞ of the w – plane. Also show that the transformation maps interior of the unit circle of
the z – plane on to upper half of the w – plane.

OR

−y
(b) (i) Prove that u = x 2 − y 2 and v = are harmonic but u + iv is not regular.
x + y2
2

1
(ii) Find the image of the half – plane x > c , c > 0 under w = . Sketch graphically. Also
z
find the fixed point of w .

BIET DEPARTMENT OF MATHEMATICS


MA6251 UNIVERSITY QUESTIONS

z+4
14. (a) (i) Evaluate ∫zc
2
+ 2z + 5
dz , where C is the circle z + 1 + i = 2 , using Cauchy’s integral

formula.

z2
(ii) Find the residues of f ( z ) = at its isolated singularities using Laurent’s
( z − 1) ( z + 2 )
2 2

series expansions. Also state the valid region.

OR



(b) (i) Evaluate ∫ 2 + cos θ .
0


dx
(ii) Evaluate
−∞
∫ (x 2
+ 1) ( x 2 + 4 )
using contour integration.

15. (a) (i) Find the Laplace transform of

f ( t ) =∈, 0≤t ≤a
and f ( t + 2a ) = f ( t ) for all t .
= − ∈, a ≤ t ≤ 2a

s2
(ii) Find the inverse Laplace transform of using convolution theorem.
(s 2
+ a2 )(s 2
+ b2 )
OR

(b) (i) Verify initial and final value theorems for the function f ( t ) = 1 + e − t (sin t + cos t ) .

(ii) Using Laplace transform solve the differential equation y ′′ − 3 y′ − 4 y = 2e − t with


y(0) = 1 = y′(0) .

BIET DEPARTMENT OF MATHEMATICS


 
MA6251 UNIVERSITY QUESTIONS

B.E./B.Tech. DEGREE EXAMINATION, APRIL/MAY 2011 

Second Semester 

Civil Engineering 

MA2161 – MATHEMATICS – II 

(Common to all branches of B.E./B.Tech. ) 

(Regulation 2008) 

Time : Three hours                Maximum : 100 marks 

Answer ALL questions 

PART A – (10 x 2 = 20 marks) 

1. Transform the equation  x 2 y′′ + xy′ = x  into a linear differential equation with constant 


coefficients. 

( )
2. Find the particular integral of D 2 − 4 y = cosh 2 x . 

3. Find the value of  m so that the vector  F = ( x + 3 y ) i + ( y − 2 z ) j + ( x + mz ) k   is 


   

solenoidal. 


4. State the physical interpretation of the line integral  F ⋅ dr . 
 
A

5. Show that  u = 2 x − x 3 + 3 xy 2 is harmonic. 

6. Find the map of the cirlce  z = 3  under the transformation  w = 2 z . 

7. Define Singular point. 

8. Expand  f ( z ) = sin z  in a Taylor series about origin. 

 
9. State the conditions under which Laplace transform of  f ( t ) exists. 

{ }
 
10. Find  L−1 cot −1 ( s ) . 
 

 
BIET DEPARTMENT OF MATHEMATICS
 
MA6251 UNIVERSITY QUESTIONS

PART B – (5 x 16 = 80 marks) 

( )
11.   (a)    (i)  Solve the equation   D 2 + 5 D + 4 y = e − x sin 2 x . 

d2 y
          (ii)  Solve the equation  + y = cos ecx  by the method of variation of                                                 
dx 2

    parameters. 

      Or 

d2 y dy
(b)    (i)  Solve  (1 + x )2 2
+ (1 + x ) + y = 2sin [ log(1 + x )] . 
 
dx dx

dx dy
       (ii)  Solve  − y = t   and   + x = t 2 . 
 
dt dt

12.   (a)  (i)  Find the angle between the normals to the surface  xy = z 2 at the points   

              ( 1, 4, 2 )  and  ( −3, −3, 3 ) .              (6) 

∫ ( )
       (ii)  Verify Green’s theorem in a plane for   3 x 2 − 8 y 2 dx + ( 4 y − 6 xy ) dy  , Where C is 
C

the boundary of the region defined by the lines  x = 0, y = 0  and  x + y = 1 .    (10) 

        Or 

(b)  Verify Gauss Divergence theorem for  F = 4 xzi − y 2 j + yzk  over the cube bounded by  


   

x = 0, x = 1, y = 0, y = 1, z = 0, z = 1 .              (16) 

dw ∂w ∂w
13.  (a)  (i)  If  w = f ( z ) is analytic, prove that = = −i . 
dz ∂x ∂y

log ( x 2 + y 2 )  is harmonic. Determine its analytic function. Find also its 
1
       (ii) Show that  u =
2
conjugate. 

        Or 

 ∂2 ∂2 
(b)  (i)  If  f ( z ) is an analytic function prove that   2 + 2  f ( z ) = 4 f ′( z ) . 
2 2

 ∂x ∂y 

          (ii)   Find the image of  z = 2 under the mapping (1)  w = z + 3 + 2i   (2)  w = 3 z . 

BIET DEPARTMENT OF MATHEMATICS


 

MA6251 UNIVERSITY QUESTIONS

z2 − 1
14. (a)  (i) Expand  f ( z ) =  as a Laurent’s series in the region  2 < z < 3 . 
( z + 2)( z + 3)

( z + 1)
        (ii) Evaluate ∫ dz  where  C  is  z + 1 + i = 2 using Cauchy’s integral formula. 
( z 2 + 2z + 4)
2
C

        Or 


x2 − x + 2
(b)  (i)  Evaluate  ∫ 4 dx  using contour integration. 
−∞
x + 10 x 2
+ 9


        (ii)   Evaluate  ∫ 2 + cos θ   using contour integration. 
0

t , for 0 < t < a


15. (a)  (i)  Find the Laplace transform of  f ( t ) =  ,  f ( t + 2a ) = f ( t ) . 
 2a − t , for a < t < 2a

 1 
        (ii)  Using Convolution theorem  L−1   . 
 ( s + a ) ( s + b ) 

        Or 

 cos at − cos bt 
(b)  (i) Find  L   . 
 t 

d2x dx dx
      (ii) Solve  2
− 3 + 2 x = 2 , given  x = 0  and  = 5  for  t = 0  using Laplace transform 
dt dt dt
method. 

BIET
  DEPARTMENT OF MATHEMATICS
MA6251 UNIVERSITY QUESTIONS

B.E./B.Tech. DEGREE EXAMINATION, JUNE 2011


Common to all B.E./B.Tech
Second Semester
181202 – MATHEMATICS – II
(Regulation 2010)
Time : Three hours Maximum : 100 marks
Answer ALL questions
PART A – (10 x 2 = 20 marks)
1. Solve : ( D 3 + 1) y = 0 .
2. Reduce the equation x 4 y′′′ − x 3 y′′ + x 2 y′ = 1 into linear equation with constant coefficients.
3. Find the unit vector normal to the surface of the sphere x 2 + y 2 + z 2 = 1 .
( )
4. Show that F = y 2 − z 2 + 3 yz − 2 x i + ( 3 xz + 2 xy ) j + ( 3 xy − 2 xz + 2 z ) k is both solenoidal
   

and irrotational.
5. Find the analytic function w = u + iv whose imaginary part is given by
v = e x ( x sin y + y cos y ) .
1
6. Find the image of z + 1 = 1 under the mapping w = .
z
1− e 2z
7. Find the residue of at its pole.
z4
1
8. Expand at z = 1 as a Taylor’s series.
z+2
e −2 s
9. Find the inverse Laplace transform of .
s−3
1
10. If L [ f ( t )] = , find Lt f ( t ) and Lt f ( t ) .
s ( s + a2 )
2 t →0 t →∞

PART B − ( 5 x 16 = 80 marks )
( )
11. (a) (i) Solve : D + 3 D + 2 y = sin x + x 2 .
2 (8)

(ii) Solve for x from the equations D 2 x + y = 3e 2 t , Dx − Dy = 3e 2 t . (8)

Or
d2 y
(b) Solve, by the method of variation of parameters, the equation 2
+ a 2 y = tan ax . (16)
dx

12. (a)
∫ (3x − 8 y 2 ) dx + ( 4 y − 6 xy ) dy where C is
2
Verify Green’s theorem in the plane for
C
(16)

BIET DEPARTMENT OF MATHEMATICS


MA6251 UNIVERSITY QUESTIONS

the boundary of the region define by x = 0, y = 0, x + y = 1 .

Or
(b) (i) Using Stoke’s theorem prove that curl grand φ = 0 . (8)

     
(ii) Evaluate
∫∫ F i n ds
s
where F = 2 xyi + yz 2 j + xzk and S is the surface of the

parallelepiped bounded by x = 0, y = 0, z = 0, x = 2, y = 1 and z = 3 . (8)

13. (a) (i) Prove that u = e − y cos x and v = e − x sin y satisfy Laplace equations, but that u + iv is
(8)
not an analytic function of z .
(ii) Show that the families of curves r n = a sec nθ and r n = b cos ecnθ cut orthogonally. (8)

Or
(b) Find the bilinear transformation which maps the points z = 1, i , −1 into the points
w = i , 0, − i . Hence find the image of z < 1 . (16)

z
14. (a) (i) Using Cauchy’s integral formula evaluate ∫ dz , where C is the circle z + i = 1 . (8)
c
z +1
2

z −1
2
(ii) Expand the function f ( z ) = in Laurent’s series for the region z > 3 . (8)
( z + 2)( z + 3)
Or


(b) Evaluate, by contour integration, the integral ∫ ,0 < a < 1. (16)
0
1 − 2a sin θ + a 2

15. (a) Using convolution theorem, find the inverse Laplace transform of
s2
.
( s 2 + a 2 ) ( s 2 + b2 ) (16)

Or
d2 y dy
Solve, by Laplace transform method, the equation 2
+ 2 + 5 y = e − t sin t ,
(b) dt dt
y(0) = 0, y′(0) = 1 . (16)

BIET DEPARTMENT OF MATHEMATICS


MA6251 UNIVERSITY QUESTIONS
B.E./B.Tech. DEGREE EXAMINATION, NOV./DEC. 2011

Regulation 2008

Second Semester

Common to all branches

MA2161 – MATHEMATICS – II

Time : Three Hours Maximum : 100 marks

Answer ALL questions

PART A – (10 x 2 = 20 marks)

1. Transform the equation x 2 y′′ + xy′ = x into a linear differential equation with constant
coefficients.

( )
2. Find the particular integral of D 2 + 4 y = sin 2 x .

3. Prove that F = yzi + zxj + xyk is irrotational.


   

4. Prove by Green’s theorem that the area bounded by a simple closed C curve is
1
( xdy − ydx ) .
2 ∫C

5. Show that an analytic function with constant imaginary part is constant.

1+ z
6. Find the invariant points of the transformation w = .
1− z

z dz
7. Evaluate ∫ ( z − 1)( z − 2) , where C is the circle z = 1 / 2 .
C

e2z
8. Calculate the residue of f ( z ) = at its pole.
( z + 1)
2

(
9. Find L e −3 t sin t cos t . )
e − as
10. Find inverse Laplace Transform of .
s

BIET DEPARTMENT OF MATHEMATICS


MA6251 UNIVERSITY QUESTIONS

PART B – (5 x 16 = 80 marks)

( )
11. (a) (i) Solve the equation D 2 − 3 D + 2 y = 2cos(2 x + 3) + 2e x .

( )
(ii) Apply the method of variation of parameters to solve D 2 + 4 y = cot 2 x .

Or

d2y dy
(b) (i) Solve: (1 + x )2 2
+ (1 + x ) + y = 4cos [ log(1 + x )] .
dx dx

dx dy
(ii) Solve − y = t and + x = t 2 given x(0) = y(0) = 2 .
dt dt

12. (a) Evaluate ∫(x


2
) ( )
+ xy dx + x 2 + y 2 dy where C is the square bounded by the lines
C

x = 0, x = 1, y = 0 and y = 1 .

Or

( )
(b) Verify Gauss Divergence theorem for the vector function f = x 3 − yz i − 2 x 2 yj + 2k
   

over the cube bounded by x = 0, y = 0, z = 0 and x = a , y = a , z = a .

13. (a) (i) Find the analytic function w = u + iv when v = e −2 y ( y cos 2 x + x sin 2 x ) and find u .

1
(ii) Show that the map w = maps the totality of circles and straight lines as circles or
z
straight lines.

Or

(b) (i) If u ( x , y ) and v ( x , y ) are harmonic functions in a region R, prove that the function

 ∂u ∂v   ∂u ∂v 
 − + i +  is an analytic function of z = x + iy .
 ∂y ∂x   ∂x ∂y 

1
(ii) Prove that the transformation w = maps the family of circles and straight lines into
z
the family of circles or straight lines.

BIET DEPARTMENT OF MATHEMATICS


MA6251 UNIVERSITY QUESTIONS

z+4
14. (a) (i) Using Cauchy’s integral formula evaluate ∫z
C
2
+ 2z + 5
dz where C is the circle

z +1+ i = 2 .

z2 − 1
(ii) Find the Laurent’s series of f ( z ) = valid in 2 < z < 3 .
( z + 2)( z + 3)

Or

sin π z 2 + cos π z 2
(b) (i) Evaluate using Cauchy’s residue theorem, ∫ dz , where C: z = 3 .
C
( z − 1)( z − 2)

x2
(ii) Evaluate using contour integration ∫ dx .
(x + 1)
2
2
−∞

 1 
15. (a) (i) Find L−1   using convolution theorem.
 s ( s 2
+ 4 ) 

(ii) Find the Laplace transform of a square wave function given by


 a
 E for 0 ≤ t ≤ 2
f (t ) =  , and f ( t + a ) = f ( t ) .
 − E for a ≤ t ≤ a
 2

Or

∫ te
−2 t
(b) (i) Evaluate cos t dt using Laplace transforms.
0

d2 y dy dy
(ii) Solve 2
+ 4 + 4 y = sin t , if = 0 and y = 2 when t = 0 using Laplace
dt dt dt
transforms.

BIET DEPARTMENT OF MATHEMATICS


MA6251 UNIVERSITY QUESTIONS

B.E./B.Tech. DEGREE EXAMINATION, NOVEMBER/DECEMBER 2011


Common to all B.E./B.Tech
Second Semester
181202 – MATHEMATICS – II
(Regulation 2010)
Time : Three hours Maximum : 100 marks
Answer ALL questions
PART A – (10 x 2 = 20 marks)
1. Find the particular integral of ( D 2 − 4 D + 4 ) y = x 2 e 2 x .

Transform the differential equation ( x 2 D 2 + 4 xD + 2 ) y = x +


1
to a differential equation with
2. x
constant coefficients.
3. Find the directional derivative of φ ( x , y , z ) = xy 2 + yz 2 at the point ( 2, −1,1) in the direction of
the vector i + 2 j + 3k .
 
4. Evaluate
∫∫∫ ∇
v
. F dV where F = x 2 i + y 2 j + z 2 k and V is the volume enclosed by the cube

0 ≤ x , y, z ≤ 1 .
5. Are z , Re( z ), Im( z ) analytic? Give reason.
6. Define Conformal.
sin π z 2 + cos π z 2 π
7. Using Cauchy’s integral formula, evaluate ∫C ( z + 1) ( z + 2 ) dz , where C is z = 2 .
1

8. Classify the singularity of f ( z ) = e e z2 .

 2π
 0, t<
3
9. Find the Laplace transform of f ( t ) =  .
cos  t − 2π 
t>

  ,
3  3
10. Verify the final value theorem for f ( t ) = 3e − t .

PART B − ( 5 x 16 = 80 marks )

( )
11. (a) (i) Solve D 2 + 4 D + 3 y = 6e −2 x sin x sin 2 x .

(ii) Using variation of parameters, solve ( 2 D 2 − D − 3 ) y = 25e − x .

Or
2
2 d y dy
(b) (i) Solve x dx 2 − 3 x dx + 4 y = x ln x .
2

BIET DEPARTMENT OF MATHEMATICS


MA6251 UNIVERSITY QUESTIONS

dx dy
(ii) Solve + 2 x − 3 y = t and − 3 x + 2 y = e 2t .
dt dt
 
12. (a) (i) Determine f ( r ) , where r = xi + yj + zk , if f ( r )r is solenoidal and irrotational. (6)
(ii) Verify Stoke’s thorem for the vector F = ( y − z )i + yzj − xzk , where S is the
surface bounded by the planes x = 0, y = 0, z = 0, x = 1, y = 1, z = 1 and C is the
square boundary on the xoy -plane. (10)

Or

( ) ( )
 
(b) (i) If F is a vector point function, prove that curl curlF −1 = ∇ ∇ i F −1 − ∇ 2 F . (6)

(ii) Vrify Gauss’s theorem for F = ( x 2 − yz ) i + ( y 2 − zx ) j + ( z 2 − xy ) k over the




rectangular parallelepiped formed by 0 ≤ x ≤ 1, 0 ≤ y ≤ 1 and 0 ≤ z ≤ 1 . (10)

13. (a) (i) If f ( z ) is analytic function of z in any domain, prove that


 ∂2 ∂2  p− 2
 2 + 2  f ( z ) = p f ′( z ) f ( z ) .
p 2 2

 ∂x ∂y 
(ii) 1
Show that the transformation w = transforms, in general, circles and straight lines
z
into circles and straight lines that are transformed into straight lines and circles
respectively.
Or
(b) (i) Verify that the families of curves u = c and v = c cut orthogonally, when w = e z 2 .
1 2

(ii) Find the Bilinear transformation that maps the points 1 + i , − i , 2 − i of the z - plane into
the points 0,1, i of the w - plane
14. 1
Find the Laurants’s series expansion of f ( z ) = valid in the regions
(a) (i) z (1 − z )
z + 1 < 1, 1 < z + 1 < 2 and z + 1 > 2 .


(ii) Evaluate ∫ a + b cos θ ( a > b > 0 ) , using contour integration.
0

Or
3z + 7 z + 1
2
If f (a ) = ∫ dz , where C is the circle z = 2 , find the values of
(b) (i) C
z−a (6)
f (3), f ′(1 + i ) and f ′′(1 + i ) .

BIET DEPARTMENT OF MATHEMATICS


MA6251 UNIVERSITY QUESTIONS


cos x dx
(ii) Evaluate ∫ (x
−∞
2
+ a 2 ) ( x 2 + b2 )
using contour integration, where a > b > 0 . (10)

15. (a) (i) t , 0 < t < 1


Find the Laplace transform of f ( t ) =  and f ( t + 2) = f ( t ) for t > 0 .
 0, 1 < t < 2 (6)

(ii) Solve y′′ − 3 y′ + 2 y = 4e 2 t , y(0) = −3, y′(0) = 5 , using Laplace transform. (10)
Or

−1  1  s +a 2 2
 
(b) (i) Find L  ln  2  .
 s  s + b
2
 
 1 
(ii) Using convolution theorem, find L−1  
 ( s + 1) ( s + 1) 
2

BIET DEPARTMENT OF MATHEMATICS


MA6251 UNIVERSITY QUESTIONS
B.E./B.Tech. DEGREE EXAMINATIONS, MAY/JUNE 2012

Second Semester

Common to all branches

MA2161 – Mathematics II

(Regulations 2008)

Time: Three hours Maximum: 100 marks

Answer ALL questions

PART A – (10 x 2 = 20 Marks)

d2 y dy
1. Transform the equation (2 x + 3)2 2
− 2(2 x + 3) − 12 y = 6 x into a differential equation
dx dx
with constant coefficients.

2. Find the particular integral of ( D − 1) y = e x sin x .


2

   
3. Find λ such that F = (3 x − 2 y + z )i + (4 x + λ y − z ) j + ( x − y + 2 z )k is solenoidal.

4. State Gauss divergence theorem.

5. State the basic difference between the limit of a function of a real variable and that of a
complex variable.

6. Prove that a bilinear transformation has atmost two fixed points.

7. Define singular point.

4
8. Find the residue of the function f ( z ) = at a simple pole.
z ( z − 2)
3

9. State the first shifting theorem on Laplace transforms.

10. Verify initial value theorem for f ( t ) = 1 + e − t (sin t + cos t ) .

PART B – (5 x 16 = 80 marks)

( )
11. (a) (i) Solve D 2 + a 2 y = sec ax using the method of variation of parameters.

( )
(ii) Solve: D 2 − 4 D + 3 y = e x cos 2 x .

BIET DEPARTMENT OF MATHEMATICS


MA6251 UNIVERSITY QUESTIONS
Or

( )
(b) (i) Solve the differential equation x 2 D 2 − xD + 4 y = x 2 sin(log x ) .

dx dy
(ii) Solve the simultaneous differential equations + 2 y = sin 2t , − 2 x = cos 2t .
dt dt

( ) ( )
12. (a) (i) Show that F = y 2 + 2 xz 2 i + ( 2 xy − z ) j + 2 x 2 z − y + 2 z k is irrotational and
   

hence find its scalar potential.

∫ ( 3 x )
− 8 y 2 dx + ( 4 y − 6 xy ) dy  , where C
2
(ii) Verity Green’s theorem in a plane for
C

is the boundary of the region defined by x = 0, y = 0 and x + y = 1 .

Or
     
∫ = + − +
2 2
(b) (i) Using Stokes theorem, evaluate F i dr , where F y i x j ( x z ) k and ‘C’ is
C

the boundary of the triangle with vertices at ( 0, 0, 0 ) , ( 1, 0, 0 ) , ( 1,1, 0 ) .

(ii) Find the work done in moving a particle in the force field given by

F = 3 x 2 i + (2 xz − y ) j + zk along the straight line from ( 0, 0, 0 ) to ( 2,1, 3 ) .


   

13. (a) (i) Prove that every analytic function w = u + iv can be expressed as a function z alone,

not as a function of z .

(ii) Find the bilinear transformation which maps the points z = 0,1, ∞ into w = i ,1, − i

respectively.

Or

 ∂2 ∂2 
(b) (i) If f ( z ) is an analytic function of z , prove that  +  log f ( z ) = 0 .
 ∂x ∂y 2 
2

1
(ii) Show that the image of the hyperbola x 2 − y 2 = 1 under the transformation w = is
z

the lemniscates r 2 = cos 2θ .

BIET DEPARTMENT OF MATHEMATICS


MA6251 UNIVERSITY QUESTIONS

zdz 1
14. (a) (i) Evaluate ∫ ( z − 1)( z − 2)
C
2
where C is z − 2 =
2
by using Cauchy’s integral formula.

1
(ii) Evaluate f ( z ) = in Laurent series valid for the regions z > 3 and
( z + 1)( z + 3)

1 < z < 3.

Or

z −1
(b) (i) Evaluate ∫ ( z + 1) ( z − 2) dz , where C is the circle
C
2
z − i = 2 using Cauchy’s

residue theorem.


cos mx
(ii) Evaluate ∫x
0
2
+ a2
dx , using contour integration.

 
 s . −1
15. (a) (i) Apply convolution theorem to evaluate L
 ( s2 + a2 )2 
 

(ii) Find the Laplace transform of the following triangular wave function given by

t , 0≤ t ≤π
f (t ) =  and f ( t + 2π ) = f ( t ) .
 2π − t , π ≤ t ≤ 2π

OR

e at − e − bt
(b) (i) Find the Laplace transform of . (4)
t

−2 t
(ii) Evaluate ∫ te
0
cos t dt using Laplace transform. (4)

d2 y dy
(ii) Solve the differential equation 2
− 3 + 2 y = e − t with y(0) = 1 and y′(0) = 0 ,
dt dt

using Laplace transform. (8)

BIET DEPARTMENT OF MATHEMATICS


MA6251 UNIVERSITY QUESTIONS

BIET DEPARTMENT OF MATHEMATICS


MA6251 UNIVERSITY QUESTIONS

BIET DEPARTMENT OF MATHEMATICS

You might also like